GkToday 16 To 31 January 2020 (Upscpdf - Com)

You might also like

Download as pdf or txt
Download as pdf or txt
You are on page 1of 114

GKTODAY

Current Affairs [PDF] - January 16-31, 2020

Last Updated: February 6, 2020


Published by: GKTODAY.IN
GKToday © 2020 | All Rights Reserved

The authors and publisher have made every effort to ensure that the information in this E-book is
correct. However, GKToday does not assume and hereby disclaims any liability to any party for any loss,
damage, or disruption caused by errors or omissions, whether such errors or omissions result from
negligence, accident, or any other cause.
This document is a property of GKToday. Unauthorized Duplication is not allowed.
Current Affairs [PDF] - January 16-31, 2020

Contents
......................... ...... _ ........ ...... ...... ....................................................
News Headlines: January 1-15, 2020 .......................................................................................................................................................... 3
Current Affairs – News Headlines: January 16, 2020 ................................................................................................................... 3
Current Affairs – News Headlines: January 17, 2020 ................................................................................................................... 4
Current Affairs – News Headlines: January 18, 2020 ................................................................................................................... 5
Current Affairs – News Headlines: January 19, 2020 ................................................................................................................... 6
Current Affairs – News Headlines: January 20, 2020 .................................................................................................................. 6
Current Affairs – News Headlines: January 21, 2020 ................................................................................................................... 7
Current Affairs – News Headlines: January 22, 2020 ................................................................................................................... 8
Current Affairs – News Headlines: January 23, 2020 ................................................................................................................... 9
Current Affairs – News Headlines: January 24, 2020 ................................................................................................................. 10
Current Affairs – News Headlines: January 25, 2020 .................................................................................................................. 11
Current Affairs – News Headlines: January 26, 2020 ................................................................................................................. 12
Current Affairs – News Headlines: January 27, 2020 ................................................................................................................. 13
Current Affairs – News Headlines: January 28, 2020 ................................................................................................................. 13
Current Affairs – News Headlines: January 29, 2020 ................................................................................................................. 14
Current Affairs – News Headlines: January 30, 2020 ................................................................................................................. 15
Current Affairs – News Headlines: January 31, 2020 .................................................................................................................. 15
January 16, 2020 .......................................................................................................................................................................................... 16
January 15: Indian Army Day ............................................................................................................................................................. 16
WEF: Global Risks Report, 2020 ...................................................................................................................................................... 17
RAISINA DIALOGUE: Ind-Fin MoU; Russia on India’s UNSC membership and Indo-Pacific .......................................... 17
Henley Passport Index launched ........................................................................................................................................................ 18
January 17, 2020 .......................................................................................................................................................................................... 18
First Session of India-Norway Dialogue held in New Delhi ....................................................................................................... 18
“Globalizing Indian Thought”, an International Conclave held at Kozhikode ....................................................................... 19
Food Processing Summit held in Ladakh ........................................................................................................................................ 19
Historic Bru-Reang Refugee agreement signed-34,000 displaced to settle in Tripura ....................................................... 20
India-World Bank sign 88 million USD to implement Assam Inland Water Transport Project ........................................ 20
Sahyog-Kaijin-India-Japan joint exercise for Coast Guards ........................................................................................................ 21
Bangladesh: Regularly submerging Island to house 100,000 Rohingya refugees .................................................................. 21
Winged Raider: Indian Army’s largest airborne exercise-500 troops participate .................................................................. 22
National Information Centre sets up Centre of Block Chain Technology in Bengaluru ..................................................... 22
January 18, 2020 .......................................................................................................................................................................................... 23
NITI Aayog and Ladakh UT sign MoU to develop Infrastructure projects ............................................................................ 23
“APNA UREA-SonaUgle” brand of HURL launched-To increase Urea Production ............................................................. 23
Russia: Production of S-400 missiles begins; China-India-Russia meet in March 2020 ..................................................... 24
Amazon to invest 1 billion USD in India; Create 1 million jobs ................................................................................................. 24
J&K Development: Rs 2000 crore for Z-Morh Tunnel; 36 Union Ministers’ visit ............................................................... 25
Tuberculosis: India renames its programme; WHO endorses India developed tech-TrueNAt MTB ............................... 25
Firsts of Census 2021: Cereals eaten, transgender run houses .................................................................................................. 26
Industrial Meet of ‘Wings India’-Asia’s largest Civil Aviation Event held at Bengaluru ....................................................... 26
ELECRAMA-Indian Electrical Industry Showcase inaugurated ................................................................................................. 27
GoI launches “Seismic Hazard Microzonation” to minimize earthquake disasters .............................................................. 27
January 19, 2020 .......................................................................................................................................................................................... 28
World Gold Council report: RBI ranks 6th in buying gold abroad .......................................................................................... 28
Myanmar-China sign 33 deals of BRI: Increases China’s ease of access of Indian Ocean .................................................. 28
January 20, 2020 ......................................................................................................................................................................................... 28
K-4 Nuclear capable Underwater missile test fired by India ..................................................................................................... 28
National Ganga Mission Begins Conservation of wet lands along the Ganges ..................................................................... 29
National Immunization Day observed in India .............................................................................................................................. 29
India to set up Maritime Research Coordination Centre in Sri Lanka ..................................................................................... 30
Group of Ministers report on legal framework to prevent sexual harassment ...................................................................... 30
Report on Central and State Water Departments based on Efficiency Targets: Gujarat leads ........................................... 31
1 trillion USD required to achieve UN’s Decarbonization Target of Shipping Industry ....................................................... 31
Delhi Government releases Guarantee card on free bus services, water & electricity .......................................................... 31

© 2020 GKToday | All Rights Reserved | https://www.gktoday.in 2


Current Affairs [PDF] - January 16-31, 2020

National Agricultural Export Authority, APEDA launched 186 product testing laboratories ............................................. 32
India prepares for Corona Virus-Steps of Health Ministry ......................................................................................................... 32
NIC organizes Tech Conclave in New Delhi on “Technologies for NextGen Governance” ............................................. 33
January 21, 2020 .......................................................................................................................................................................................... 34
UNCTAD releases Global investment Trend report: India among top 10 recipients of FDI ............................................. 34
“TIGERSHARKS”, First Squadron of Sukhoi-30 MKI aircraft inducted by IAF in Tamil Nadu ...................................... 34
Carbon Disclosure Project: India ranks 5th .................................................................................................................................... 35
GoI launches paperless licensing of petroleum service stations ................................................................................................. 35
Global Social Mobility Index launched at World Economic Forum: India ranks 76th ........................................................ 36
January 22, 2020 ......................................................................................................................................................................................... 36
Claims against India under Bilateral International Treaty dismissed ........................................................................................ 36
Women Power at Republic Day parade: Tania Shergil and CRPF bikers ............................................................................... 37
Start Up India-first time in Republic Parade; Indian Navy .......................................................................................................... 37
India Nepal inaugurate new check post, Jogbani-Biratnagar at the border ........................................................................... 38
32nd interaction of PRAGATI held: Rs 24,000 crores worth projects discussed ................................................................ 38
India joins Reskilling Revolution Initiative at World Economic Forum as a Founding Member ........................................ 39
World Economic Forum Survey: World loses hopes in Climate Science; Indians are most-trusting ................................ 39
World Economic Forum: 1 trillion trees and report on Password role in Cyber Crime ...................................................... 40
January 23, 2020 ......................................................................................................................................................................................... 40
Facts Box: “Three Capital Bill” of Andhra Pradesh ..................................................................................................................... 40
National Startup Advisory Council setup to boost Startups and Innovations ........................................................................ 40
India-Brazil: BIT to be signed; Cabinet approves agreements on legal assistance, Oil ......................................................... 41
India participates at United Nations AIDS Programme at World Economic Forum ............................................................. 41
Democracy Index released: India ranks 51; Slips 10 positions ................................................................................................... 42
January 23: Birth Anniversary of Subhash Chandra Bose celebrated ...................................................................................... 42
Ghana inspired by India’s Ujjwala; India to assist; MoU signed ................................................................................................ 43
Facts Box: ILO’s Labour Report, 2020 ........................................................................................................................................... 43
Defence Ministry approved procurement of 5,100 crores of hardware ................................................................................. 44
January 24, 2020 ......................................................................................................................................................................................... 44
Phase Out of Ozone Depleting Chemicals achieved by India .................................................................................................... 44
“Vyom Mitra”-ISRO’s half Humanoid-An experimental mission of Gaganyaan ................................................................... 45
January 24: National Girl Child Day ................................................................................................................................................ 45
National Data and Analytics Platform released by NITI Aayog ................................................................................................ 46
January 25, 2020 ......................................................................................................................................................................................... 46
25 January – National Voters Day ................................................................................................................................................... 46
January 26, 2020 ......................................................................................................................................................................................... 46
Ganga-Volga Dialogue held with Russia to enhance Connectivity ........................................................................................... 46
Department of Biotechnology held International Summit on Women in STEM .................................................................. 47
India & World Bank sign Agreement of 210 million USD in agriculture and rural transformation ................................. 47
India’s first Super Fab Lab inaugurated in Kerala ........................................................................................................................ 48
Indian Railway’s first Waste to Energy Plant uses “PolyCrack” Technology ......................................................................... 48
January 27, 2020 ......................................................................................................................................................................................... 49
Bharat Parv, a festival of culture being held at Red Fort ............................................................................................................ 49
India-Brazil ink 15 agreements during Brazil President’s visit on Republic Day ................................................................... 49
Padma Awards announced on Republic Day ................................................................................................................................ 49
DRDO displays A-SAT weapon system at Republic Day Parade ............................................................................................. 50
Shiv Bhojan Tali Scheme launched in Maharashtra ..................................................................................................................... 50
Kobe Bryant, a retired NBA star dies in Helicopter Crash ......................................................................................................... 50
Global Potato Conclave to be held in Gujarat ................................................................................................................................ 51
Andhra Pradesh Cabinet nods to abolish Legislative Council ..................................................................................................... 51
GoI signs “Historic Accord”, Bodo Agreement ............................................................................................................................ 52
January 27: International Holocaust Remembrance Day ............................................................................................................ 52
GoI new Target: Indian Railways to run 100% on electricity by 2024 ................................................................................... 52
January 28, 2020 ......................................................................................................................................................................................... 53
Corona virus: WHO increases global risk from moderate to high ........................................................................................... 53
India’s first underwater Metro: To begin in March 2022 in Kolkata ...................................................................................... 53
Bengal becomes the fourth state to pass resolution against CAA ............................................................................................. 53
US: 25,000 Child porn cases in 5 months in India; Rajya Sabha panel report on child porn ............................................ 54

© 2020 GKToday | All Rights Reserved | https://www.gktoday.in 3


Current Affairs [PDF] - January 16-31, 2020

January 28: Lala Lajpat Rai Birth Anniversary Celebrated .......................................................................................................... 54


Resolution against CAA in European Parliament; India’s Measures ......................................................................................... 55
Britain releases new 50 pence coin marking BREXIT ................................................................................................................. 55
Passport Renewal Reminder Services begun by Passport Seva Kendras ................................................................................. 56
Report Card on Smart Cities to be released by June 2020 ........................................................................................................ 56
MGNREGA scheme faces funds shortage ...................................................................................................................................... 56
January 29, 2020 ......................................................................................................................................................................................... 57
Supreme Court: African Cheetah permitted to be introduced in India ................................................................................... 57
10 more Wet lands declared Ramsar Sites: Maharashtra gets its first Ramsar site ............................................................... 57
Bhuvan Panchayat 3.0 launched by ISRO ...................................................................................................................................... 57
Composite Regional Centre launched in Port Blair ...................................................................................................................... 58
“Operation Vanilla” launched by Indian Navy for Disaster Relief at Madagascar ................................................................. 58
National Commission for Homeopathy bill, 2019 approved by Cabinet ................................................................................ 58
Cabinet: National Commission for Indian System of Medicine Bill, 2019 approved ............................................................ 59
The Medical Termination of Pregnancy (Amendment) Bill, 2020 approved by Cabinet ................................................... 59
Cure for Corona Virus: AYUSH recommends Unani Medicines .............................................................................................. 60
“Madhavpur Mela” of North East reviewed; Cabinet approves 30% NEC allocation to new projects ........................... 60
January 30, 2020 ......................................................................................................................................................................................... 60
NITI Aayog’s Aspirational District Ranking released: Chandauli of UP tops ......................................................................... 60
World Sustainable Development Summit organized by TERI first time in New Delhi ........................................................ 61
War Memorial built by Assam Rifles in Nagaland ........................................................................................................................ 61
Surajkund Mela-International Crafts Fair organized by Haryana with HP as theme state .................................................. 61
UN Development Programme: Telangana tops sustainable growth goals performer list ................................................... 62
January 30: World Neglected Tropical Diseases Day .................................................................................................................. 62
Transparency International: Assam tops in budget formulation ............................................................................................... 63
SAMPRITI-IX: India-Bangladesh Joint Military Exercise to be held in Meghalaya ............................................................... 63
Indian Railways to roll out Automatic Identification and Data Collection .............................................................................. 63
Indian National Commission for Cooperation with UNESCO held in New Delhi ............................................................... 64
January 31, 2020 .......................................................................................................................................................................................... 64
WHO: International Health Emergency declared over Corona Virus ...................................................................................... 64
World Games Athlete of the Year, 2019-Title won by Rani Rampal ....................................................................................... 65
CII celebrates 125 years; Tamil Nadu CM launches new scheme at the event ..................................................................... 65
Google: Tangi Application; AI Model for Killer Whales; 1 million USD Grant ..................................................................... 65
TERI: India’s Steel Demand to increase carbon emissions by 600 MT ................................................................................. 66
Economic Survey 2020 tabled in the Parliament by the Finance Minister ............................................................................. 66
Amazon partnering with Eastern Railways sets up pick-up kiosk in Kolkata after Mumbai ............................................. 67
ISRO preparing 500 low cost satellite launch vehicles in 5 years ............................................................................................ 68
Vidya Bal-Social Activist, Feminist passed away ........................................................................................................................... 68
Indian Railways: First Banana Container Train to boost exports; First High Capacity Parcel Van ................................... 68
First indigenous bio-jet fuel powered IAF aircraft lands successfully in Leh ......................................................................... 69
Current Affairs Quiz: January 16, 2020 ................................................................................................................................................. 69
Current Affairs Quiz: January 17, 2020 ................................................................................................................................................. 72
Current Affairs Quiz: January 18, 2020 ................................................................................................................................................. 75
Current Affairs Quiz: January 19-20, 2020 .......................................................................................................................................... 78
Current Affairs Quiz: January 21, 2020 .................................................................................................................................................. 81
Current Affairs Quiz: January 22, 2020 ................................................................................................................................................. 84
Current Affairs Quiz: January 23, 2020 ................................................................................................................................................. 87
Current Affairs Quiz: January 24, 2020 ................................................................................................................................................ 90
Current Affairs Quiz: January 25, 2020 ................................................................................................................................................. 93
Current Affairs Quiz: January 26-27, 2020 .......................................................................................................................................... 96
Current Affairs Quiz: January 28, 2020 ................................................................................................................................................ 99
Current Affairs Quiz: January 29, 2020 ............................................................................................................................................... 102
Current Affairs Quiz: January 30, 2020 ............................................................................................................................................... 105
Current Affairs Quiz: January 31, 2020 ................................................................................................................................................ 108

© 2020 GKToday | All Rights Reserved | https://www.gktoday.in 4


Current Affairs [PDF] - January 16-31, 2020

News Headlines: January 1-15, 2020


Current Affairs – News Headlines: January 16, 2020
Army Day
Army Day celebrated on January 15
On January 15, 1949, Lt Gen K M Cariappa became the first Indian commander-in-chief of the Indian
Army
General Francis Butcher was the last British commander-in-chief of the Indian Army
India
222 squadron to be operationalised with Sukhoi-30 fighter jets at Air Force Station in Thanjavur on
January 20
Science Film Festival of India (SCI-FFI 2020) being held at Panaji (Goa): January 15-18
Centre accepts Justice Dhingra committee report on Delhi Police’s role in 1984 anti-Sikh riots
Annual bull taming event ‘Jallikattu’ begins in Madurai, Tamil Nadu on the occasion of Pongal
Crossword Book Awards: Popular Awards
Business & Management — Anju Sharma For ‘Corporate Monk’
Health & Fitness — Dr Jaishree Sharad For ‘Skin Rules’
Non-Fiction — Gaur Gopal Das For ‘Life’s Amazing Secrets’
Children’s Books — Sudha Murty For ‘The Upside-Down King’
Fiction — Twinkle Khanna For ‘Pyjamas Are Forgiving’
Biography — Krishna Trilok For ‘Notes Of A Dream: The Authorized Biography Of A.R. Rahman’
Crossword Book Awards: Jury Awards
Indian Language Translation: N. Prabhakaran For ‘Dairy Of A Malayali Madman’ (Translated By
Jayasree Kalathil)
Children’s Writing — Richa Jha For ‘Machher Jhol’ (Illustrated By Sumanta Dey)
English Fiction — Madhuri Vijay For ‘The Far Field’
English Non-Fiction — Shanta Gokhale For ‘One Foot On The Ground’
New Frog Species
Three new species of tiny frogs have been discovered in Arunachal Pradesh by scientists of
Zoological Survey of India.
These were discovered from Talle Valley Wildlife Sanctuary (WLS) in Lower Subansiri district
These were named as ‘Liurana himalayana’, ‘Liurana indica’ and ‘Liurana minuta’
Economy & Corporate
Merchandise trade data for April-December 2019-20 — Exports: USD 239.29 billion, Imports: USD
357.39 billion, Trade Deficit: USD 118.10 billion
RBI asks banks and other card-issuing companies to provide facility to customers to switch on and
off their debit or credit cards
‘Saksham’, the month-long fuel conservation campaign of PCRA (Petroleum Conservation Research
Association), to begin from January 16
National Health Authority signs MoU with ITC e-Choupal to help farmers avail benefits of Ayushman
Bharat health insurance scheme
Amazon founder Jeff Bezos announces USD 1 billion investment in India to help bring small and
medium businesses online
World
Annual Global Risks Report of Geneva-based WEF identifies 5 global risks: extreme weather events,
human-made environmental disasters, major biodiversity loss, natural disasters, and failure of climate
change mitigation and adaptation.
© 2020 GKToday | All Rights Reserved | https://www.gktoday.in 3
Current Affairs [PDF] - January 16-31, 2020

Sri Lanka: India constructs new buildings for five schools in Tamil-majority Northern Province
China’s first and oldest female tractor driver Liang Jun, whose image features on Renminbi currency
notes, dies at 91
Russian Prime Minister Dmitry Medvedev resigns
Icc Awards
Virat Kohli: Captain of 2019 ODI, Test Teams of the Year
Rohit Sharma: 2019 ODI Cricketer of the Year
Pat Cummins: Test Cricketer of the Year
Ben Stokes: 2019 ICC’s Cricketer of The Year (Sir Garfield Sobers Trophy)
Virat Kohli: 2019 Spirit of Cricket Award
Marnus Labuschagne: 2019 ICC Men’s Emerging Cricketer
Current Affairs – News Headlines: January 17, 2020
India
Conference of CPA (Commonwealth Parliamentary Association) India Region being held at UP
Assembly in Lucknow: January 16-19
Coast Guards of India and Japan conduct ‘Sahyog-Kaijin’ exercise off the Chennai coast
Singer Suman Kalyanpur and music director Kuldeep Singh win MP govt.’s National Lata Mangeshkar
Awards for 2017 and 2018, respectively
Agreement signed to settle 34,000 Bru-Reang refugees from Mizoram in Tripura
Arunachal becomes first state to sign MoU with IUCN (International Union for Conservation of
Nature); to start red-listing of orchids
IIT-Madras researchers develop device to generate hydrogen fuel from seawater
WHO endorses ‘TrueNat TB test’ technology developed by Indian scientists to detect Tuberculosis
International conclave on “Global Indian Thought” organised at IIM Kozhikode; PM unveils Swami
Vivekananda’s statue
IAF rescues trekkers from frozen Zanskar River in the UT of Ladakh; were part of ongoing ‘Chadar
Trek’
Month-long exhibition ‘Indian Heritage in Digital Space’ being organised in New Delhi from January 15
by DST (Department of Science & Technology) & IIT-Delhi
International Heritage Symposium organised in New Delhi on January 15-16 by DST & IIT-Delhi
Central Adoption Resource Authority (CARA) celebrates 5th Annual Day on January 15
World Bank, centre sign $88 million loan agreement for implementing Assam Inland Water Transport
Project to help modernize the state’s passenger river ferry sector
Ladakh Food Processing Summit “Building Partnerships for Inclusive Growth in Food Processing
Sector” organised in Leh
to organise mega-science exhibition Vigyan Samagam in New Delhi from January 21 to March 20
Mega-science exhibition Vigyan Samagam to be organised in New Delhi from January 21 to March 20
jointly by the Department of Atomic Energy (DAE) and DST
Sanrakshan Kshamta Mahotsav – 2020 organised by Petroleum Conservation Research Association
(PCRA) in New Delhi to promote conservation and efficient use of petroleum products
Economy & Corporate
51st K-9 Vajra-T gun rolled out at L&T Armoured Systems Complex at Hazira in Gujarat
Reliance Jio is largest mobile service company with 36.9 crore mobile subscribers: TRAI
Liquidator will have to deposit unclaimed dividends and undistributed proceeds in Corporate
Voluntary Liquidation Account (CVLA) before seeking dissolution of a corporate debtor: Insolvency

© 2020 GKToday | All Rights Reserved | https://www.gktoday.in 4


Current Affairs [PDF] - January 16-31, 2020

and Bankruptcy Board of India (IBBI)


Newly appointed Deputy Governor Michael Patra to head Monetary Policy Department at RBI
Hallmarking of gold jewellery and artefacts compulsory from January 15, 2021; sales permitted in
only three grades of gold — 14, 18 and 22 carats
Bank of Baroda and Kotak Mahindra Bank exit Equifax Credit Information Services
UN WESP 2020
Global economy grew 2.3% in 2019: United Nations World Economic Situation and Prospects (WESP)
2020 report
GDP growth estimate for India is 5.7% in the current fiscal and 6.6% in the next fiscal
World
Qatar scraps mandatory exit permits for migrant domestic staff and others
Germany announces compensation to power companies for shutdown of their coal-fired power plants
by 2038
US Census: Sikhs included as a separate Ethnic Group for the first time
Former WWE (World Wrestling Entertainment) wrestler Rocky Johnson dies at 75 in US
Sports
Stamp on Chuni Goswami, captain of India’s last Asian Games gold winner football team (1962),
issued
Mohun Bagan merges with RP-Sanjiv Goenka Group’s ATK FC, to be known as ATK-Mohun Bagan
Kerala govt. awards GV Raja awards to Muhammad Anas (athletics) and PC Thulasi (badminton) in
male and female categories respectively
Indian GM Abhijeet Gupta wins Delhi Open International Grand Masters Chess Championship
Current Affairs – News Headlines: January 18, 2020
India
ISRO’s communication satellite GSAT-30 launched onboard Ariane 5 rocket of Arianespace from
French Guiana
Aadhaar required to get benefits under scheme for civilian victims of terror, communal violence
Gujarat CM Vijay Rupani flags off the inaugural run of the Ahmedabad-Mumbai Tejas Express from
Ahmedabad
Travellers from China to be screened for novel coronavirus (nCoV) at Indian airports
Economy & Corporate
Foreign exchange reserves reach a life-time high of $461.21 billion in the week to January 10
NHIDCL (National Highways and Infrastructure Development Corporation) awards 6.5 km long Z-Morh
Tunnel project in J&K to APCO Amarnathji Tunnelway Pvt Ltd to be built at a cost of Rs 2,379 crore
Logo and brand ‘Apna Urea Sona Ugle’ of Hindustan Urvarak and Rasayan Ltd (HURL) unveiled by
Fertiliser Minister D V Sadananda Gowda
China’s Great Wall Motors (GWM) to acquire manufacturing facility of General Motors (GM) in
Talegaon, Maharashtra
TRAI directs telecom service providers to deposit all unclaimed refunds of customers in the
Telecommunication Consumers Education and Protection Fund (TCEPF)
NPCI (National Payment Corporation of India) launches blockchain-based Vajra platform for secure
online transactions
BCCL (Bennett Coleman & Company Limited) Chairperson Indu Jain conferred ICSI Lifetime
Achievement Award of the Institute of Company Secretaries of India (ICSI)
World
53-year old bureaucrat Mikhail Mishustin appointed new Prime Minister of Russia
© 2020 GKToday | All Rights Reserved | https://www.gktoday.in 5
Current Affairs [PDF] - January 16-31, 2020

Ukraine: Prime Minister Oleksiy Honcharuk resigns after less than six months in office.
Sports
12-day, 7,800-km Dakar Rally held in Saudi Arabia ends: Spain’s Carlos Sainz wins car rally; Ricky
Brabec of US wins motorbike rally
India (340/6 in 50 overs) beat Australia (304 in 49.1 overs) by 36 runs in 2nd ODI at Rajkot; level
series 1-1
Former India all-rounder Bapu Nadkarni dies in Mumbai at 96; made 1414 runs and took 88 wickets in
41 Tests
‘The Winning Sixer’, book written by former Indian batsman W.V. Raman, released
Current Affairs – News Headlines: January 19, 2020
India
Lieutenant General SK Saini appointed new Vice Chief of Army Staff
Former bureaucrat Nripendra Misra appointed chairperson of the executive council of Nehru
Memorial Museum and Library (NMML)
Election Commission’s ‘cVIGIL’ and ‘Voter Helpline App’ win 2019 eGovernance ‘Award of Excellence’
given by the Computer Society of India (CSI), Special Interest Group on e-Governance (CSI SIG e-Gov)
Ashwini Kumar Chopra, editor of Hindi newspaper Punjab Kesari and BJP MP from Karnal in Haryana
in 2014-19, dies at 63 in Gurugram
National Immunization Day will be observed on January 19
Merck Young Scientist Award 2019 in Chemical Sciences awarded to Dr Sakya Singha Sen from
National Chemical Laboratory (CSIR-NCL), Pune
Economy & Corporate
RBI holds 625.2 tonnes of gold (6.6% of its forex reserves): World Gold Council (WGC)
WEF Meeting
Annual meeting of the World Economic Forum (WEF) will be organised at Davos, Switzerland from
January 20 to 24
The theme is: “Stakeholders for a Cohesive and Sustainable World”
Union Minister of Commerce and Industry & Railways Piyush Goyal will lead the Indian delegation
World
National Security Advisor Ajit Doval calls on Sri Lankan President Gotabaya Rajapaksa in Colombo
Tropical Cyclone Tino batters Fiji
Khagendra Thapa Magar, who measured 67.08 cm and was verified by the Guinness World Records
as ‘world’s shortest man who could walk’, dies at 27 in Pokhara, Nepal
Sports
Rome Ranking Series wrestling tournament: Vinesh Phogat wins gold in women’s 53 kg, Anshu Malik
silver in women’s 57 kg
‘Fit India Cyclothon’ organised at Panaji, Goa as part of Youth Affairs and Sports Ministry’s ‘Fit India
Mission’
India’s Sania Mirza and Nadia Kichenok of Ukraine win women’s doubles title at Hobart International
tennis
Australia’s Ashleigh Barty wins women’s singles title at ATP-WTA Adelaide International tennis
Union Minister Arjun Munda becomes President of suspended Archery Association of India (AAI)
Current Affairs – News Headlines: January 20, 2020
India
India successfully test-fires the 3,500 km strike range nuclear-capable K-4 submarine-launched
ballistic missile off the coast of Andhra Pradesh
© 2020 GKToday | All Rights Reserved | https://www.gktoday.in 6
Current Affairs [PDF] - January 16-31, 2020

Nationwide programme to administer polio drops to children organized on National Immunization Day
(January 19)
Classical singer Sunanda Patnaik dies at 85 in Kolkata; singer of Gwalior gharana, she won 2012
Sangeet Natak Academy Award
Chhattisgarh CM Bhupesh Baghel launches employment mobile app ‘Rozgar Sangi’ to connect
employment providing institutions and skilled trained youth
Supreme Court to appoint its retired judge Justice A. M. Sapre as the chairman of its committee on
road safety
World
Britain’s Prince Harry and his wife Meghan to give up their royal titles of Duke and Duchess of Sussex
Over half the world’s total GDP is dependent on nature and exposed to risks from nature loss says
WEF’s ‘Nature Risk Rising Report’
Sports
Indonesia Masters 2020 badminton tournament: Thailand’s Ratchanok Intanon (women’s singles) and
Indonesia’s Anthony Sinisuka Ginting (men’s singles) win titles
India (289 for 3 in 47.3 overs) beat Australia (286 for 7 in 50 overs) by 7 wickets in 3rd and final ODI
at M. Chinnaswamy stadium in Bengaluru; win series 2-1
India’s medal winners at Rome Ranking Series wrestling tournament on January 18: Bajrang Punia:
gold in men’s 65kg freestyle, Ravi Kumar Dahiya: gold in men’s 61kg freestyle, Gurpreet Singh: gold in
men’s 82kg Greco-roman, Sunil Kumar: silver in men’s 97kg Greco-roman and Sajan Bhanwal: bronze
in men’s 77kg Greco-roman.
Tata Mumbai Marathon
Ethiopia’s Derara Hurisa wins men’s race in 2:08:09, women’s race won by Ethiopia’s Amane Beriso in
2:24:51
Srinu Bugatha (2:18:44) and Sudha Singh (2:45:30) finished first among Indian runners in men’s and
women’s races respectively.
Current Affairs – News Headlines: January 21, 2020
India
IAF inducts its first squadron of Sukhoi-30 MKI aircraft equipped with BrahMos missiles at its airbase
in Thanjavur (TN); named ‘Tigersharks’ 222 squadron
AP to have three capitals: Visakhapatnam: executive capital, Kurnool: judicial capital and Amaravati:
legislative capital
Former Congress Haryana MLA and ex-Rajya Sabha MP Shamsher Singh Surjewala dies at 87
Economy & Corporate
DPIIT (Department for Promotion of Industry and Internal Trade) launches paperless licensing
process for petroleum service stations
IMF lowers India’s economic growth estimate in 2019-20 to 4.8% from 6.1% in its World Economic
Outlook (WEO) Report
Challa Sreenivasulu Setty appointed as Managing Director of SBI for a period of three years
SC accepts Centre’s proposal to take over management control of real estate firm Unitech
Ashish Garg elected President of the Institute of Company Secretaries of India (ICSI)
UNCTAD Report On 2019 Fdi
Global Investment Trend Monitor report released by UNCTAD (United Nations Conference on Trade
and Development)
India among top 10 recipients of Foreign Direct Investment (FDI) in 2019, attracting USD 49 billion
(16% increase from 2018)
© 2020 GKToday | All Rights Reserved | https://www.gktoday.in 7
Current Affairs [PDF] - January 16-31, 2020

Global FDI in 2019 was USD 1.39 trillion (1% decline from 2018)
US remained the largest recipient of FDI at USD 251 billion in 2019
World
World’s 2,153 billionaires have more wealth than the 4.6 billion people who make up 60% of the
planet’s population, said NGO Oxfam in its ‘Time to Care’ report
Number of international tourist arrivals was 1.5 billion in 2019 (4% more than 2018): Madrid-based
UNWTO (United Nations World Tourism Organisation)
IMF revises downwards growth estimate for global economy in 2019 to 2.9% from 3% in its World
Economic Outlook (WEO) Report
India ranked 76th out of 82 countries on WEF’s Social Mobility Index topped by Denmark
Berlin Summit on Libya fails to initiate talks between the warring parties — strongman Khalifa Haftar
and the head of Tripoli’s UN-recognised government Fayez al-Sarraj
Sports
Former Test batsman and National selector Man Mohan Sood dies at 80
Current Affairs – News Headlines: January 22, 2020
India
PM Modi, Nepal’s PM K. P. Sharma Oli jointly inaugurate 2nd integrated border check post at Jogbani-
Biratnagar; first ICP was built in Raxaul-Birgunj in 2018
Jharia (Jharkhand) is India’s most polluted city; Lunglei (Mizoram) least polluted: Greenpeace India
report on air pollution
Manipur, Meghalaya and Tripura celebrated their Statehood Day on January 21
Uttarakhand becomes first state to implement agricultural land leasing policy
Upgraded ‘Sharang’ artillery guns to be inducted into Army: Gun Carriage Factory, Jabalpur (MP)
Indian Navy signs MoU with Geological Survey of India (GSI) for sharing of offshore data
Indian Council for Child Welfare (ICCW) announces winners of National Bravery Awards
Economy & Corporate
PSB appointments: Sanjiv Chadha: MD & CEO in Bank of Baroda; Lingam Venkata Prabhakar: MD and
CEO in Canara Bank, Atanu Kumar Das: MD and CEO in Bank of India
Yudhvir Singh Malik appointed CMD of crisis-hit real estate firm Unitech by the govt.
ICICI Bank launches cardless cash withdrawal facility from its ATMs using its mobile banking
application ‘iMobile’
India fourth best market after US, China and Germany for global companies looking for growth
abroad: PwC CEO survey
Centre notifies structure of National Startup Advisory Council; to be headed by Commerce & Industry
Minster
World
India opens its first Mahatma Gandhi Convention Centre in West African nation Niger at capital
Niamey
Australia confers its highest civilian honour, the Order of Australia, on Biocon founder Kiran
Mazumdar-Shaw
Guyana takes over chairmanship of the Group of 77 for 2020
WEF, partners launch 1t.org to support efforts to grow, conserve 1 trillion trees globally
Number of people registered as unemployed is expected to rise to 190.5 million in 2020: World
Employment and Social Outlook report of ILO
Nation’s Cup Women’s Boxing
India win 6 medals at 9th Nation’s Cup women’s boxing tournament in Sombor, Serbia
© 2020 GKToday | All Rights Reserved | https://www.gktoday.in 8
Current Affairs [PDF] - January 16-31, 2020

Silver (4) – M. Meena Kumari (54kg), Monika (48kg), Ritu Grewal (51kg) and Bhagyabati Kachari
(75kg)
Bronze (2): Pavitra (60kg) and Pwilao Basumatary (64kg).
AICS
Wrestler Yogeshwar Dutt and Paralympian Deepa Malik among new inductees in govt.’s All India
Council of Sports (AICS)
Sachin Tendulkar and chess wizard Viswanathan Anand omitted for not being “active” enough
Formed in 2015, AICS is an advisory body to the Ministry of Youth Affairs and Sports
Veteran sports administrator V K Malhotra is President of AICS.
Sports
Meyton Cup tournament in Innsbruck, Austria: Indian shooters Divyansh Singh Pawar (men’s 10m air
rifle) and Apurvi Chandela (women’s 10m air rifle) win gold medals
Current Affairs – News Headlines: January 23, 2020
India
Cabinet approves additional term of reference (ToR) to the Other Backward Classes (OBC) sub-
categorisation commission headed by Justice G. Rohini; extends its term till July 31, 2020
ISRO to send lady robot ‘Vyommitra’ in unmanned Gaganyaan; Vyoma: Space, Mitra: Friend
PM reviews status of several projects through PRAGATI — the ICT based multi-modal platform for
Pro-Active Governance and Timely Implementation, involving Central and State governments.
Daman designated as headquarters of newly-carved out Union Territory of Dadra and Nagar Haveli
and Daman and Diu
approves a model pact with foreign countries for recognition of seafarers’ competency certificates
UP: ‘Mukhyamantri Krishak Durghatna Kalyan Yojana to provide financial assistance to farmers
suffering accidents
Economy & Corporate
Cabinet approves closure of Hindustan Fluorocarbons Ltd.
Cabinet approves Rs 4371.90 crore for permanent campuses of new National Institutes of
Technology (NITs)
Indian Oil to assist Ghana in implementation of National LPG Promotion Policy in the African nation
India completes phasing out of ozone-depleting chemical Hydrochlorofluorocarbon (HCFC)-141b
Steel Minister Dharmendra Pradhan launches SERVICE (SAIL Employees Rendering Volunteerism &
Initiatives for Community Engagement)
SKOCH Governance Rankings
Gujarat tops SKOCH State of Governance 2019 ranking
Gujarat, Maharashtra, West Bengal, Andhra Pradesh and Tamil Nadu were the top five “Star States”
Star States are those where projects have performed the best in terms of rankings across different
parameters
Democracy Index
India was ranked 51st (down 10 places) in 2019 Democracy Index – a ranking of 165 countries by
Economist Intelligence Unit
The index was topped by Norway, followed by Iceland and Sweden
India was classified in the ‘flawed democracy’ category
GTCI
India was ranked 72nd (up 8 places) in 2020 Global Talent Competitiveness Index (GTCI)
Switzerland topped the list of 132 countries, followed by the US and Singapore.
Launched by INSEAD, GTCI ranks countries on the basis of their ability to grow, attract and retain
© 2020 GKToday | All Rights Reserved | https://www.gktoday.in 9
Current Affairs [PDF] - January 16-31, 2020

talent
World
Greece’s parliament elects 63-year-old Ekaterini Sakellaropoulou as the country’s first woman
president
Welsh comic legend actor, writer and co-founder of ‘Monty Python’ Terry Jones dies at 77
Sports
Maharashtra retain Khelo India Youth Games champions trophy in Guwahati with 256 medals
including 78 gold
Rowing Federation of India (RFI) de-recognised by the Sports Ministry for violating the Sports Code
during polls in Dec 2019
Current Affairs – News Headlines: January 24, 2020
India
Subhash Chandra Bose Aapda Prabhandan Puruskar awarded to Disaster Mitigation And
Management Centre, Uttarakhand (institution category) and IPS officer Kumar Munnan Singh
(individual category) for their commendable work in disaster management
EC hosts first Sukumar Sen Memorial Lecture in New Delhi as tribute to India’s first Chief Election
Commissioner
Poet Arvind Krishna Mehrotra receives 5th Mahakavi Kanhaiyalal Sethia Award at Jaipur Literature
Festival
MP, Maharashtra make reading of Preamble of Constitution mandatory in schools
Kerala Tourism wins UNWTO’s Accessible Destination Award for its disabled-friendly ‘Barrier-Free
Tourism Project’
Maharashtra Cabinet approves proposal to keep Mumbai open for 24 hours from January 27; bars
and pubs excluded
Election Commission successfully tests facial recognition technique in Telangana
Bal Shakti Puraskar 2020
49 children in 5-18 years age group were given the Bal Shakti Puraskar 2020 by President Ram Nath
Kovind
Awards were given in the fields of innovation, social service, scholastic, sports, art and culture and
bravery
Award comprised a medal, a cash prize of ₹1 lakh, a certificate and a citation.
Enemy Properties
A Group of Ministers (GoM) headed by Union Home Minister Amit Shah will monitor the disposal of
over 9,400 enemy properties across the country
Enemy properties were those left behind by the people who took citizenship of Pakistan and China.
Economy & Corporate
RBI raises short-term investment limit for FPIs (foreign portfolio investors) in government and
corporate bonds from 20% to 30%
4th special open market operation (OMO): RBI buys Rs 10,000 crore long-term securities and sells Rs
2,950 crore short-term government bonds
NITI Aayog releases Vision Document for the National Data and Analytics Platform (NDAP); to make
all govt. data accessible to stakeholders in a user-friendly manner
2 from India – IIM, Bangalore and XLRI- Xavier School of Management, Jamshedpur — ranked among
top 30 global B-Schools in Positive Impact Rating (PIR) 2020 released at WEF meeting in Davos
Tata Motors launches premium hatchback Altroz priced at ₹5.29 lakh for petrol version and ₹6.99
lakh for diesel
© 2020 GKToday | All Rights Reserved | https://www.gktoday.in 10
Current Affairs [PDF] - January 16-31, 2020

World
India delivers 30,000 doses of Measles and Rubella (MR) vaccine to Maldives
WEF Centre for Cybersecurity announces new internet security principles, developed by a group of
leading ISPs and multilateral organisations, to help protect up to one billion users
World Holocaust Forum organised in Jerusalem, 75th anniversary of the liberation of Auschwitz camp
commemorated
Swissmint mints world’s smallest gold coin of 2.96-millimeter diameter and 0.063 gm weight
Pakistan tests nuclear-capable surface-to-surface ballistic missile ‘Ghaznavi’ of 290-km range
Travel ban imposed in China’s Wuhan city after coronavirus outbreak
Spain’s eastern and southern coastline hit by Storm Gloria; nine dead
Mahatma Gandhi’s bust will be set up at IMO (International Maritime Organisation) headquarters in
London said Shipping Minister Mansukh Lal Mandaviya
Sports
World Archery lifts suspension on the Archery Association of India (AAI)
Current Affairs – News Headlines: January 25, 2020
India
CEC Sunil Arora takes over as the new Chair of the Forum of the Election Management Bodies of
South Asia (FEMBoSA) for 2020
National Girl Child Day celebrated on January 24; Govt announces to set up ten chairs in various
universities in the name of eminent women to encourage women to carry out research activities
UP Diwas celebrated on January 24; CM Yogi Adityanath lays foundation of 18 Atal residential
schools for children of labourers
Scholars from India and Russia participate in the first Ganga-Volga Dialogue of Civilizations held in
New Delhi
Department of Biotechnology holds International Summit on Women in STEM (science, technology,
engineering and math) – “Visualizing the Future: New Skylines” on January 23-24 in New Delhi
Latest edition of Oxford Advanced Learner’s Dictionary features 26 new Indian English words,
including Aadhaar, chawl, dabba, hartal and shaadi
Dipa Chaudhury & Puneet Gupta awarded Romain Rolland Prize for Translation for their Hindi
translation of French cartoon series “Asterix”
India successfully test-fires K-4 submarine-launched missile of 3500 km range off Visakhapatnam
coast
India, Bangladesh sign agreement to upgrade 50 km Ashuganj (river port)-Akhaura (land port) road in
Bangladesh into 4-lane highway.
644 extremists from eight banned outfits surrender in Assam
Economy & Corporate
India’s foreign exchange reserves touch a life-time high of USD 462.16 billion in the week ended
January 17
Government of India and the World Bank sign USD 210 million loan agreement for State of
Maharashtra’s Agribusiness and Rural Transformation Project.
IndusInd Bank launches wealth management platform Pioneer Banking for high net-worth customers
West Bengal was the top state in vegetable production in 2018-19 at 29.55 million tonne (MT)
Tata Steel Kalinganagar honoured at WEF meeting in Davos for admittance into the Global Lighthouse
Network
World
India’s ranked 80th on Corruption Perceptions Index (CPI-2019) of Transparency International; New
© 2020 GKToday | All Rights Reserved | https://www.gktoday.in 11
Current Affairs [PDF] - January 16-31, 2020

Zealand and Denmark are least corrupt, Somalia most corrupt


Ruling Nepal Communist Party wins 16 seats out of 18 in National Assembly, the upper house of
parliament
China: Death toll in coronavirus outbreak climbs to 25; authorities lock down 8 cities to prevent its
spread
WEF announces the launch of Global Consortium for Digital Currency Governance that will design a
framework for the governance of digital currencies
International Day of Education observed on January 24; theme: ‘Learning for people, planet, prosperity
and peace’
Sports
India beat host NZ by 6 wickets in the first T20I of the five-match series at the Eden Park, Auckland
India’s Sonia Lather (women’s 57kg) wins bronze at Strandja Memorial Boxing Tournament in Sofia,
Bulgaria
Current Affairs – News Headlines: January 26, 2020
Padma Awards
141 recipients of the 2020 Padma awards announced by the government
The 7 winners of Padma Vibhushan: George Fernandes (posthumous), Arun Jaitley (posthumous), ex-
PM of Mauritius Anerood Jugnauth, boxer M. C. Mary Kom, Hindustani classical singer Chhannulal
Mishra, Sushma Swaraj (posthumous), Sri Vishveshateertha Swamiji Sri Pejavara Adhokhaja Matha
Udupi (posthumous)
The 16 winners of Padma Bhushan include badminton player P. V. Sindhu, industrialists Venu
Srinivasan and Anand Mahindra, former Nagaland Chief Minister S. C. Jamir, architect Balkrishna
Doshi, Hindustani classical vocalist Ajoy Chakrabarty, former Bangladeshi diplomat Syed Muazzem
Ali (posthumous), former Goa CM Manohar Parrikar (Posthumous), legal education pioneer N. R.
Madhava Menon (posthumous), Peoples Democratic Party (PDP) leader Muzaffar Hussain Baig
The 118 winners of Padma Shri include film producers Karan Johar and Ekta Kapoor, actress
Kangana Ranaut, singers Suresh Wadkar and Adnan Sami, former cricketer Zaheer Khan, Rani Rampal
(hockey), former hockey player M. P. Ganesh, shooter Jitu Rai, boxer Bembem Oinam and archer
Tarundeep Rai
Army Gallantry Awards
Shaurya Chakra (6): Lt Col. Jyoti Lama, Major Konjengbam Bijendra Singh, Naib Subedar Narender
Singh, Naib Subedar Sombir (posthumous), Naik Naresh Kuma, and Sepoy Karmdeo Oraon
151 Sena Medals, including 10 Param Vishisht Seva Medal, 32 Ati Vishisht Seva Medal and 8 Yudh
Seva Medal (YSM) announced
Police Awards
President of India approved the conferment of service/gallantry medals on 1040 police personnel
President’s Police Medal for Gallantry (PPMG): 4
Police Medal for Gallantry (PMG): 286
President’s Police Medal for Distinguished Service: 93
Police Medal for Meritorious Service: 657
Jeevan Raksha Padak Awards
President approved the conferment of Jeevan Raksha Padak Series of Awards – 2019 on 54 persons
Sarvottam Jeevan Raksha Padak to 07, Uttam Jeevan Raksha Padak to 08 and Jeevan Raksha Padak
to 39 persons.
India
10th National Voters’ Day celebrated on January 25 to mark the foundation day of Election
© 2020 GKToday | All Rights Reserved | https://www.gktoday.in 12
Current Affairs [PDF] - January 16-31, 2020

Commission; theme: ‘Electoral Literacy for Stronger Democracy’


Brazilian President Jair Bolsonaro visits India
Noted artist and sculptor Sher Singh Kukkal dies at 86 in New Delhi
Filmmaker Kabir Khan’s web series “The Forgotten Army” creates Guinness World Record for being
the ‘biggest team of musicians to perform live ever’
to reward people visiting 15 tourist spots in a year by funding their expenses
Economy & Corporate
TVS Motors launches electric scooter iQube at Rs 115,000
World
Turkey: 20 people killed in 6.8 magnitude earthquake in eastern Elazig province
Strandja Memorial Boxing
India return from Strandja Memorial Boxing Tournament in Sofia, Bulgaria with three medals.
Silver was won by Mohammed Hussamuddin (men’s 57kg)
Bronzes were won by Sonia Lather (women’s 57kg) and Shiva Thapa (men’s 63kg)
Sports
Russian Grand Master Pavel Ponkratov wins Chennai Open international GM chess tournament
Current Affairs – News Headlines: January 27, 2020
India
Tourism Ministry organising Bharat Parv 2020 at Red Fort in New Delhi from January 26 to 31
Brazilian President Jair Messias Bolsonaro attends Republic Day Parade at Rajpath in New Delhi as
Chief Guest
Maharashtra: Maha Vikas Aghadi government of Shiv Sena, NCP and Congress launches ‘Shiv Bhojan
Thali’ scheme for providing meals to the poor for just Rs 10
NIRDPR (National Institute of Rural Development and Panchayati Raj) ties up with UNICEF for
communication resource unit
Himachal Pradesh celebrated its 50th Statehood Day on January 25
Economy & Corporate
Road transport and highways ministry launches ‘GATI’ portal to enable contractors to raise project
related issues
World
of Netherlands issues apology for the persecution of Jews during the second world war
Sports
Hong Kong’s Ng Ka-long (men’s) and Japan’s Akane Yamaguchi (women’s) win singles titles at
Thailand Masters badminton in Bangkok
Fabiano Caruana of US wins Tata Steel Masters chess tournament at Wijk Aan Zee, The Netherlands
India beat NZ by wickets in 2nd T20 at Eden Park, Auckland to take 2-0 lead in 5 match series
Current Affairs – News Headlines: January 28, 2020
India
71st Republic Day Parade in New Delhi: IAF wins first prize among the marching contingents of the
three services; CISF is first among paramilitary forces
Centre signs peace agreement with banned organisations demanding a separate Bodoland state
carved out of Assam
Andhra Pradesh Assembly passes resolution seeking to abolish the state Legislative Council
Theatre personality Sanjna Kapoor to get ‘Knight of the Order of Arts and Letters’ from French
government for her outstanding contribution to theatre
West Bengal assembly passes resolution against Citizenship Amendment Act
© 2020 GKToday | All Rights Reserved | https://www.gktoday.in 13
Current Affairs [PDF] - January 16-31, 2020

The Hindu Group Chairman N. Ram wins national award instituted by Kerala Media Academy
Sneha Pamneja wins Oxford Bookstore Book Cover Prize for designing the cover of “Tiffin: Authentic
Recipes Celebrating India’s Regional Cuisine”
Economy & Corporate
invites bids to divest its entire stake in Air India, and the airline’s subsidiary Air India Express along
with its joint venture Air India SATS Airport Services Private Limited
Horticulture production in India at 310.74 million tonnes (MT) in 2018-19; Fruits: 97.97 MT,
Vegetables: 183.17 MT
WORLD
62nd edition of Grammy Awards in Los Angeles: Billie Eilish wins in the awards for Album Of The
Year, Record Of The Year, Best New Artist, Song Of The Year, and Best Pop Vocal Album.
International Customs Day (ICD) celebrated by the World Customs Organisation (WCO) on January 26;
slogan: “Customs fostering Sustainability for People, Prosperity and the Planet”
Slovenian Prime Minister Marjan Sarec resigns and calls for fresh elections
International Day of Commemoration in Memory of the Victims of the Holocaust observed on January
27
Sports
Fares Dessouky of Egypt beats India’s Saurav Ghosal in final to win Pittsburgh Open squash
tournament
Anil Khanna nominated Life President of Asian Tennis Federation
41-year old American basketball legend Kobe Bryant among nine people killed in a helicopter crash in
US
Former Indian women’s hockey team captain Sunita Chandra dies at 76 in Bhopal
Current Affairs – News Headlines: January 29, 2020
Tableau Awards
Tableau of Assam (theme: ‘Land of Unique Craftsmanship and Culture’) was the best among
States/UTs in Republic Day Parade at Rajpath on January 26
Tableaux of Ministry of Jal Shakti (theme: ‘Jal Jeevan Mission’) and NDRF (theme: ‘Aapda Sewa
Sadaiv’) were the joint among those from different ministries/departments.
India
SC allows Centre to introduce the African cheetah from Namibia to a suitable habitat in India
Ramsar declares 10 more wetland sites from India as sites of international importance
Bhuvan Panchayat V 3.0 web portal launched by govt. for use by the public and Panchayati Raj
Institutions (PRIs)
Oxford University Press names Samvidhaan (Constitution) as its Hindi Word of 2019
2019 Nobel Economics laureate Abhijit Banerjee conferred honorary D Litt by Calcutta University
Former Bihar Minister and RJD MLA Abdul Gafoor dies at 60
Centre inaugurates Composite Regional Center (CRC) in Port Blair for Skill Development,
Rehabilitation and Empowerment of Persons with Disabilities
EAM S Jaishankar lays foundation stone of skill development centre to train tribals of Kevadiya
village in Gujarat’s Narmada district
Indian Railways commissions country’s first governmental waste to energy plant in Bhubaneswar
Economy & Corporate
Indian Banks Association appoints Sunil Mehta as its new chief executive
UK launches 4-mn pound Innovation Challenge Fund in India to support collaborative research into
emerging technologies
© 2020 GKToday | All Rights Reserved | https://www.gktoday.in 14
Current Affairs [PDF] - January 16-31, 2020

Togo appoints NTPC as its project management consultant (PMC) for 300 MW of solar power
projects
India’s crude steel output increased 1.8% to 111.2 million tonnes (MT) in 2019: World Steel
Association
3rd Global Potato Conclave-2020 being organised at Gandhinagar from January 28 to 31
Tata Motors launches electric variant of its compact sports utility vehicle Nexon for Rs 13.99 lakh
World
Taranjit Singh Sandhu appointed as India’s new Ambassador to the US
Indian Navy launches ‘Operation Vanilla’ to help cyclone-hit Madagascar
China is the second-largest arms producer in the world, behind the US: Stockholm International Peace
Research Institute (SIPRI)
Google releases hashtag #AndroidHelp to assist those facing smartphone issues
Sports
Pankaj Advani wins Senior National Billiards Championship in Pune
Current Affairs – News Headlines: January 30, 2020
India
Cabinet approves raising upper limit for permitting abortions to 24 weeks; Medical Termination of
Pregnancy Act, 1971 to be amended
Chandauli (UP) tops list of 112 aspirational districts ranked by NITI Aayog in December
TERI organising World Sustainable Development Summit in New Delhi on January 29-31; theme:
‘Towards 2030: Making the Decade Count’
Assam Rifles builds war memorial in Nagaland for 357 personnel martyred in anti-insurgency
operations
Indian Coast Guard commissions new interceptor boat C-448 at New Mangalore Port; built by L&T
Shipyard
Eminent social worker Tushar Kanjilal dies in Kolkata at 85; had won Padma Shri in 1996 and
Jamnalal Bajaj Award in 2008
Haryana govt. to organise 34th Surajkund International Crafts Mela from February 1 to 16; Himachal
Pradesh is ‘theme state’
Economy & Corporate
RBI appoints its Executive Director Janak Raj as member of the 6-member Monetary Policy
Committee (MPC)
Union Cabinet approves allocation of 30% of funds of the North-Eastern Council for new projects
Assam is the top state in terms of best practices followed in budget formulation: Transparency
International
World
US House of Representatives passes bill on sanctions against Chinese officials for meddling in Dalai
Lama’s succession
The World Social Report 2020, published by UN-DESA (UN Department of Economic and Social
Affairs) praises India for using digital technologies for furthering inclusive development
Sports
India beat NZ in 3rd T20 International at Seddon Park in Hamilton via Super Over to take an
unassailable 3-0 lead in 5-match series
Saina Nehwal joins BJP, last match was 1st round defeat at Thailand Masters Badminton on Jan 22
Current Affairs – News Headlines: January 31, 2020

© 2020 GKToday | All Rights Reserved | https://www.gktoday.in 15


Current Affairs [PDF] - January 16-31, 2020

India
India-Bangladesh joint military training exercise ‘SAMPRITI-IX’ to be held in Meghalaya from February
3 to 16
J&K journalist Yusuf Jameel wins 2019-2020 PEN Gauri Lankesh Award for democratic idealism
Martyr’s Day, the 72nd death anniversary of Mahatma Gandhi, observed on January 30
Department of Personnel and Training’s (DoPT) online certificate course on “Relevance of Gandhi in
the Contemporary World” launched by Union Minister Jitendra Singh
Access to casinos for the locals in Goa will be banned from February 1: CM Pramod Sawant
MP govt. to develop infrastructure along ‘Ram Van Gaman Path’, the route believed to be taken by
Lord Ram headed for his exile in forest.
Noted Urdu poet Ajmal Sultanpuri dies in Sultanpur (UP) at 97
MP: BJP MLA from Agar Malwa assembly constituency, Manohar Untwal, dies in Gurugram at 54
Economy & Corporate
GDP growth estimates at 5% in 2019-20, set to recover to 5.5% in 2020-21: FICCI Economic Outlook
Survey
India’s gold demand fell 9% to 690.4 tonnes in 2019: World Gold Council
DoT takes over process to issue and manage IMEI from private body MSAI
MSAI: Mobile Standard Alliance of India, IMEI-International Mobile Equipment Identity
Khadi wrist watches manufactured jointly by Titan and KVIC (Khadi and Village Industries
Commission) launched
World
Vinay Mohan Kwatra appointed as the next Ambassador of India to Nepal
UNEP (UN Environment Programme) Goodwill Ambassador Pavan Sukhdev wins 2020 Tyler Prize for
Environmental Achievement
2019 Global Go To Think Tank Index: Observer Research Foundation (ORF) is best ranked Indian
institution at 27th
European Parliament approves terms of the U.K.’s departure from the European Union
Sports
Canada’s Christine Sinclair becomes all-time leading scorer in international football with 185 goals
National Table Tennis Championships in Hyderabad: Petroleum Sports Promotion Board (PSPB) win
men’s title; Railways win women’s title
Ngangom Bala Devi becomes first Indian woman to earn a professional football contract abroad after
signing for Scottish club Rangers FC
Former India basketball captain Matthew Satya Babu dies in Chennai at 79.
January 16, 2020
January 15: Indian Army Day
On January 15, 2020, India celebrated its 72nd Army Day. It is being celebrated every year to honour soldiers
who sacrificed their lives for the country. The Army Day Parade was held at the Parade Ground in Delhi
Cantonment as a part of the celebrations. It is important to remember such great soldiers as they serve with
the motto “Service before Self”
Highlights
On January 15, the first General of Indian Army General K M Cariappa took charge in 1949. The transfer of
power from British to India is being celebrated on the day in the form of parades. The power was transferred
to Mr Cariappa from the last British General Bucher.
The parade was held at the Parade Ground of Delhi Cantonment. The Parade Ground was also named after

© 2020 GKToday | All Rights Reserved | https://www.gktoday.in 16


Current Affairs [PDF] - January 16-31, 2020

him and is called Cariappa Ground. Gallantry awards were distributed on the occasion.
General Cariappa retired on January 14, 1953. His retirement day is being celebrated as Armed Forces
Veterans Day every year
First Woman Officer
In 2020, Tania Sher Gill became the first woman Army Officer to lead the parade. She led an all-men
contingent at the parade. She joined the army in 2017. In 2019, Captain Bhavna Kasturi became the first
woman officer to lead an all men contingent during the Republic Day celebrations.
WEF: Global Risks Report, 2020
On January 15, 2020, the World Economic Forum released its Global Risks Report. Under the report, the
Forum has identified the global risks.
Top Risks
The top 5 Global risks according to the report are as follows
The Forum reported that Climate-related risks had the greatest likelihood of severe impacts.
The other threats include geopolitical turbulence. The retreat from multilateralism has threatened the
ability of the world to deal the global risks.
The current economic stagflation according to the report will increase. This might even lead to
increased social issues, protests, increased populists movements.
Though the Digital Technology is in its peak of evolution, the unequal access of the technology
across the globe will increase.
The social demographic, environmental and technological pressure on health systems is growing.
Top Highlights of the Report
The Report has listed 5 top risks in the terms of Likelihood and in terms of Impact. The global risks in terms
of likelihood includes extreme weather, climate action failure, natural disasters, biodiversity losses and
human-made environmental disasters. For the first time in the history of the report, all the five risks of
likelihood is climate-related.
The Global Risks in terms of impacts include climate action failure, weapons of mass destruction,
biodiversity losses, extreme weather and water crisis.
RAISINA DIALOGUE: Ind-Fin MoU; Russia on India’s UNSC membership and Indo-Pacific
The Raisina Dialogue was held at New Delhi for four days recently. Leaders from several countries
participated in the dialogue. It included foreign ministers from 13 countries including Iran, Australia and
Russia. During the Raisina Dialogue, India and Finland signed a defence agreement. Also, Russia announced
that it supports India for the latter’s permanent membership at the UNSC (United Nations Security Council)
Russia at Raisina Dialogue
Indo-Pacific
Russia at the Raisina Dialogue questioned the idea of “Indo-Pacific”. It said that the concept of Indo-Pacific
is divisive. Russia also believes that the concept of Indo-Pacific isolates China and also aims to move away
from ASEAN centred building model.
UNSC Membership
Russia at the Raisina Dialogue also announced that it backs India for the latter’s permanent UNSC
membership. Along with India, Brazil is also seeking permanent membership from UNSC. Russia extended
its support to Brazil as well.
India-Finland
On the sidelines of the Raisina Dialogue, India and Finland signed a Defence MoU. The agreement aimed to
increase cooperation between the countries in terms of procurement, production, R&D of Defence related
equipment. The agreement was under discussion between the countries since DefExpo 2018. It has now

© 2020 GKToday | All Rights Reserved | https://www.gktoday.in 17


Current Affairs [PDF] - January 16-31, 2020

been formalized to run up to DefExpo 2020. The talks for the agreement was initiated by PM Modi at the
first India-Nordic Summit held at Stockholm.
India-Nordic Summit
The Nordic countries are the countries in Northern Europe and North Atlantic. It mainly includes Finland,
Denmark, Iceland, Sweden and Norway. At the India-Nordic Summit, the areas of economic growth, global
security, climate change and innovation were focused.
Henley Passport Index launched
Recently, Henley Passport Index was launched. The index was launched based on the data collected from
International Air Transport Association (IATA). Around 199 different passports and 227 different travel
destinations were considered to launch the index.
The Index is based on the number of destinations their holders can access without visa.
Highlights
According to the index, Japan has the world’s strongest passport. Japan has been holding top rank position
for the past 3 years. According to 2020 index, the citizens can access 191 destinations without the
necessity of obtaining visa in advance.
Japan was followed by Singapore, Germany, South Korea at second, third and fourth places respectively. US
and UK have been falling places for two year are now in eighth position.
India ranked closer to the bottom at rank 84.
History of performances
UAE was the historic performer of Henley Passport Index 2020. UAE has jumped 47 places in the past 10
years. It is currently at 47th position.
India was in Rank 71 as the index was released in 2006. Today Indian citizens can access only 60
destinations in the world without obtaining prior Visa.
Visa Free Destinations
There are 58 destinations in the world that does not require prior Visa. 20 of the 58 are in Africa, 11 in Asia.
There is no developed country or a major power in the world where Indians can travel without Visa. Serbia is
the only European country where Indians are allowed to travel without visa.
What is Passport Index?
Passport index is an online tool that provides insights about visa-free travel option. The score also says
how welcoming the countries are. The Passport index includes Visa-Frees score, World Openness score,
Welcoming score and Global Mobility score. When these scores are high, passport index is also high and
such a passport is said to be highly powerful.
January 17, 2020
First Session of India-Norway Dialogue held in New Delhi
Between January 15, 2020 and January 16, 2020, the first session of India-Norway trade and investment
dialogue was held in New Delhi. It was the first meeting of the countries after they signed an agreement in
2019.
Highlights
India and Norway signed Dialogue of Trade and Investment when Norway Prime Minister visited India in
2018. Based on the agreement, the countries held their first meeting to discuss industrial trade. The
discussions included on various areas such as renewable energy, ICT, fisheries, MSME, maritime, shipping.
The countries exchanged their views on investment opportunities.
The Delhi Mumbai Industrial Corridor Development Corporation, FICCI (Federation of Indian Chambers of
Commerce and Industry), CII (Confederation of Indian Industries) made presentations. The Department of
Economic Affairs and Department of Promotion of Industry and Internal Trade (DPIIT) highlighted several
© 2020 GKToday | All Rights Reserved | https://www.gktoday.in 18
Current Affairs [PDF] - January 16-31, 2020

policy initiatives of the government.


Significance
At every bilateral meeting, it is important to involve the industries and their clusters as this would help in
understanding and exchanging the dynamics.
India-Norway
The trade between India and Norway was 1127 million USD in 2014. It is far less as compared to its
potential. There are 100 Norwegian companies that have invested in India. It includes hydropower, clean
energy, petroleum related services, drilling equipment, etc.
Apart from trade, diplomacy ties with Norway is important for India for its Arctic researches. India has 2
important research stations in Norway. They are Himadr and IndARC.
Himadri-Polar Research Station
India, in 2008, opened its first Arctic Research Station in Norway. It is called Himadri. The main research of
the station is on food-web dynamics, space weather, aerosol radiation, glaciers, microbial communities,
carbon recycling and sedimentology.
IndARC
IndARC is the first underwater observatory of India in the Arctic Region. It is midway between Norway and
North Pole. The observatory aims to study Arctic climate and how it influences the monsoon.
“Globalizing Indian Thought”, an International Conclave held at Kozhikode
On January 17, 2020, the International Conclave on “Globalizing Indian Thought” was held in IIM, Kozhikode.
PM Modi addressed the conclave. The conclave aimed at revealing India’s thoughts in becoming one of top
three economies of the world.
Highlights
India is stepping towards the goal of becoming a 5 trillion Dollar economy by 2025. This will make India one
of the top three economies in the world after US and China. It is essential for India to reveal its path while it
is achieving such goals. The Conclave is an initiative to achieve this.
The Conclave focuses on India’s thought on development. The thoughts are as follows
Satyam-Truth
Nithyam-Sustainability
Purnam-Wholeness.
The conversations on globalizations have so far been synonymous with westernization. India does not
contribute to such conversations. Through these conclaves India says that India has always stick to its
intellectual heritage, innovative knowledge system and rich traditions.
According to India, the three thoughts Satyam, Nithyam and Purnam influence the world greatly.
Significance
The Conclave is important as it reveals India and its path towards development to the global market.
Ethically, it is important to choose the right path of development. With the world marching towards
development, it is important to get hold of traditional routes to sustain mankind. The conclave is important
to convey this message to the world. The Conclave also offers opportunities to thinkers across different
professions to converge their ideas into creative platforms.
Food Processing Summit held in Ladakh
On January 16, 2020, the Department of Industries and Commerce, functioning under Ministry of Food
Processing Industries organized food processing summit in Ladakh. The other stakeholders of the summit
include Invest India and National Investment Promotion and Facilitation agency.
Highlights
The Summit was attended by food producers of Ladakh. It included producers of milk, barley, meat, organic

© 2020 GKToday | All Rights Reserved | https://www.gktoday.in 19


Current Affairs [PDF] - January 16-31, 2020

vegetables, apples and apricots. The Ladakh Action Plan for Food Processing to promote food processing
was presented at the summit.
The summit focused on chain mapping of food processing and recommended strategies to integrate local
population towards agriculture and food processing. It also focused primarily on Leh and Kargil regions.
Current Scenario
Though Ladakh has the potential for growing cereals, vegetables and food grains, agriculture has now taken
back seat in the rapid growth of the region. Agriculture has lost its place to the growing tourism in the
region. Also, the other reasons for decreased participation of agricultural activities include low cropping
intensity, short agriculture season and less productivity.
Government measures
The Government of India has been focusing on the following to develop agriculture in the region
GoI adopts improved green houses, poultry farming, lambing shed, value addition and marketing of
horticultural products and compost pits.
It introduces high yielding crop varieties, seed production techniques, organic farming
The Union Government is also motivating farmers to adopt cold weather technologies to increase
production.
Historic Bru-Reang Refugee agreement signed-34,000 displaced to settle in Tripura
On January 17, 2020, Government of India, Bru-Reang representatives, Government of Tripura and Mizoram
signed an agreement in New Delhi to end the refugee crisis of Bru-Reang community. Home Minister Amit
Shah presided over the event.
Highlights
Under the agreement, around 34,000 Bru Refugees are to settle in Tripura. The Centre will offer their
rehabilitation helps with an allocated budget of Rs 600 crores. With this, the refugees are now eligible to
enjoy social welfare schemes of state and central governments.
According to the agreement, the displaced families are to be provided with land of 40 by 30 square feet. The
land is to be provided by the Tripura Government. The Centre will provide aid of Rs 1.5 lakhs to build their
houses. The Centre will also provide fixed deposit of Rs 4 lakhs and Rs 5,000 cash aid per month and free
ration for 2 years.
What is the issue?
Due to ethnic tension in 1997, around 5,000 families of 30,000 Bru-Reang tribes were forced out of Mizoram
to seek shelter in Tripura. In 2010, GoI began its efforts to permanently rehabilitate them as they constantly
demanded to settle in Tripura. In 2018, GoI signed an agreement with the State Governments of Tripura and
Mizoram in order to help the families and increase the aid substantially.
Bru-Reang Tribes
The Bru Tribes are locally called Reang and hence the name Bru-Reang. They are ethnic tribes living in India,
Laos, Vietnam and Thailand. They speak Katuic language. They believe in Animism religion and some others
believe in Buddhism. The Bru-Reang are one of the 21 ethnic tribes of Tripura that are included int the
Scheduled Tribe list. Some of them are Tibeto-Burmese origin.
India-World Bank sign 88 million USD to implement Assam Inland Water Transport Project
On January 16, 2020, the Government of Assam, Government of India and World Bank signed 88 million USD
agreement. The agreement aims to modernize Assam’s Ferry transport service on the Brahmaputra river.
Highlights
There are over 361 ferry routes across Brahmaputra river in Assam. These routes act as a crucial means of
transport to 1000s of commuters in both rural and urban areas of the valley.
The aim of Assam Inland Water Transport Project (AIWTP) is to help Assam improve its ferry infrastructure

© 2020 GKToday | All Rights Reserved | https://www.gktoday.in 20


Current Affairs [PDF] - January 16-31, 2020

and strengthen institutions operating inland water transport. It also aims at introducing new and energy
efficient vessels with least disturbance to nature. The project also includes cargo transport vessels in the
route.
The inland water transport is the most sustainable water transport for Assam. This is mainly because it
provides low-cost and low-carbon options.
The project will adopt “Working with Nature” principles to design new infrastructure.
Inland Waterways
Around 15 waterways are operating in Assam. They operate on rivers such as Barak, Beki, Lohit, Doyang,
Kopili, Subansiri, Aai, Dihing, Dikihow, Puthimari, Gangadhar, Jinjiram and Tiwang. It is important to develop
these water ways as they are connected to Bay of Bengal through the Brahmaputra and will help increase
exports of locally produced goods of north east. The IBP (India-Bangladesh Protocol), National Waterway-2
that was recently launched in November 2019 makes this possible. Assam Inland Water Transport
Development is the implementing authority of these routes.

Sahyog-Kaijin-India-Japan joint exercise for Coast Guards


On January 16, 2020, India and Japan took part in a joint exercise called “Sahyog-Kaijin”. The exercise was
held between the coastal guards of the countries at Chennai Port.
Highlights
The Japanese ship Ehigo and 4 ships of Indian Coast Guard participated in the exercise. The coast guards
conducted the scenario of capture of a hijacked ship and rescue of crew in a joint operation. They also
demonstrated search and rescue operations and displayed their fire-fighting skills.
The exercise was conducted based on the agreement signed by the Coast Guards of the countries in 2006.
Significance
The exercise is important for the countries to handle piracy in the Indian Ocean region. Also, both the
countries being part of the Quad grouping, it is important for them to refresh their skills, learning from each
other and stay connected to ease operations at sea.
Other exercises
Apart from Coastal Guard Exercises, Indian and Japan hold other military exercises as well. The “Shinyuu
Maitri”is the joint Air Force Exercise that was held in October 2019 for the first time.
The India-Japan Maritime Exercise called JIMEX was conducted in 2013 for the first time. It focuses on
maritime security cooperation.
Dharma Guardian is the military exercise that is conducted between the armies of the countries.
Bangladesh: Regularly submerging Island to house 100,000 Rohingya refugees
The Bangladesh Government has planned to accommodate 100,000 Rohingya refugees in its Bhasan Char
Island.
Highlights
The island, Bhasan Char that is regularly submerged in water during monsoon has been fortified with
embankments and flood protection to relocate Rohingyas here. However, the date of relocation has not
been announced. Hospitals, mosques and houses have been built. There are more than 700,000 Rohingyas
that fled to Bangladesh from Myanmar as the latter began ethnic cleansing of Rohingyas in their country.
Bhasan Char
The Island is also known as Hatiya Upazila. It is located at 37 miles from the coast. The Island has been
fortified with Himalayan silt in 2006. It is under water between June and September due to monsoon rains.
This is because Bangladesh is a low-lying Delta country formed by the two unification of two mighty
Himalayan rivers Ganges and Brahmaputra.

© 2020 GKToday | All Rights Reserved | https://www.gktoday.in 21


Current Affairs [PDF] - January 16-31, 2020

Criticizations
The United Nations Refugee Agency termed the move of the Bangladeshi government as logistically
challenging. Several other organizations like the Human Rights Watch criticized the move as humanitarian
disaster.
India
India has been deporting Rohingyas to Myanmar. In 2017, the Union Ministry announced that there are
around 40,000 Rohingyas living in India. Around 16,000 of them have been registered under United Nations
as Refugees. They are spread across Delhi, Rajasthan, Andhra Pradesh, Haryana and Tamil Nadu.
Winged Raider: Indian Army’s largest airborne exercise-500 troops participate
The Indian Army recently conducted the largest air borne exercise called the Winged Raider. Around 500
troops participated in the exercise. The exercise was conducted in the north-eastern theatre.
Highlights
The exercise was conducted to demonstrate the operational readiness of the paratroopers and air warriors.
The C-130 Hercules, Dhruv Helicopters and C-17 globe master transport aircraft participated in the exercise.
The exercise was telecasted live. It focused on encountering China in the border region. The exercise also
included capacity building programmes and habitats. It also focused on moving certain advanced weapon
system to the eastern side.
Significance
The Army is preparing itself to act against threats posed by China. Indian Army is also developing
infrastructure in the Eastern borders. These exercises help to self-assess the readiness of the troops. The
exercise will also help army to rebalance its western and northern fronts.
Him Vijay
Similar exercise called Him Vijay was conducted in Arunachal Pradesh in October 2019. It was a mountain
combat exercise. The exercise tested the Integrated Battle Groups (IBG) in Indian army. The IBG consists of
tanks, infantry, air defence, logistics and signals. The IBG is a restructure combat tactics of Indian Army.
This helps to meet emerging threats in the Eastern borders.
The exercise also helped the Indian Army to learn lessons. According to the feedback of Him Vijay exercise,
communication skills of the army should be developed
National Information Centre sets up Centre of Block Chain Technology in Bengaluru
On January 18, 2020, the National Informatics Centre (NIC) will set up block chain technology centre in
Bengaluru. The centre is to be inaugurated by Union Minister Shri Ravi Shankar Prasad.
Highlights
The centre aims to provide Block Chain as service to all the stake holders who are interested in shared
experiences, learning and resources. The centre will share its block chain-based proof of concepts with
selected government entities. The centre will also act to help different wings of the government understand
the potential of the technology.
NIC
National Informatics Centre has always been the premier technology advisor and also a pioneer in
introducing technologies such as National Knowledge Network, NICNET, etc. It was founded in 1976. NIC
operating under Ministry of Electronic and Information Technology helps government in embracing
Information Technology. Since 1990s. It is also helping to disseminate E-Governance to the citizens.
NICNET is the ICT (Information and Communication Technology) network. It provides linkages between
different departmental and ministerial administrations in India.
National Knowledge Network
It is a project that aims to create robust network for the country. It integrates multidisciplinary research and

© 2020 GKToday | All Rights Reserved | https://www.gktoday.in 22


Current Affairs [PDF] - January 16-31, 2020

development by acting as a communication platform. The network aims to connect all the universities,
libraries, research institutes, laboratories, agricultural institutions and health care across the country.
January 18, 2020
NITI Aayog and Ladakh UT sign MoU to develop Infrastructure projects
On January 17, 2020, NITI Aayog and Ladakh Union Territory signed a Memorandum of Understanding.
According to the agreement, NITI Aayog will support the administration of the Ladakh through its initiative
“Development Support Services to States for Infrastructure Projects” (DSSS)
Highlights
NITI Aayog will identify high-impact projects. It will expedite project implementations, create unique models
of development and will identify high-impact priority projects. A team of experts associated with NITI Aayog
are to be stationed in Ladakh to identify and develop the territory. The team will predominantly focus on
solar energy, tourism and basic infrastructure.
What are DSSS projects of NITI Aayog?
The main objective of DSSS projects is creating PPP (Public Private Partnership) projects. In 2018, phase I
of the project was completed. In phase I, 10 projects were selected from 400 projects that were submitted
by the states. In phase II, that was completed in the year 2018-19, the selected projects entered transaction
stage. Preparation activities were carried out. The think tank also created techno-economic feasibility
report.
The following were other steps taken
The think tank organized investor consultations for the state governments
Integrated post-harvest supply chain management and integrated community based micro-irrigation
infrastructure were generated.
“APNA UREA-SonaUgle” brand of HURL launched-To increase Urea Production
The APNA UREA-SonaUgle brand of HURL (Hindustan Urvarak & Rasayan Limited) was launched by the
Union Minister of Chemicals and Fertilizers Shri D V Sadananda Gowda.
Recent developments in Urea Production
In order to make India self-sufficient in Urea, the Union Government approved revival of three Urea plants
located in Sindri, Gorakhpur and Barauni. The other two major fertilizer plants at Talcher and Ramagundam
are also to be reopened. With these measures GoI projects that the annual production of Urea is to increase
by 63.5 lakh metric tonnes per annum.
What is HURL?
HURL is a joint venture of Indian Maharatna companies namely IOCL (Indian Oil Corporation Limited), NTPC
(National Thermal Power Corporation) and Coal India Limited. The objective of HURL is to make India self-
reliant in Urea. The following are the targets set for HURL
Revival of three Urea plants by 2021
Increase neem coated urea production capacity to 38.1 lakh metric tonnes per annum
To start operations of energy efficient and environment friendly natural gas-based fertilizer plants by
2021. The feedstock (Natural Gas) is to be supplied by GAIL.
Current Scenario of Urea Production
According to National Urea Policy 2015, there are totally 31 Urea producing plants in India. Out of these, 28
units Natural Gas as feedstock. The rest of the units use Naphtha as feedstock.
Concerns
The production of urea is water consuming process. The current annual water consumption of urea
manufacturing plants as of June 2019 is 191 million cubic metres. Some of the urea plants in India are yet
to abide the 2003 Corporate Responsibility for Environment Protection Guidelines, 2003 that was issued by
© 2020 GKToday | All Rights Reserved | https://www.gktoday.in 23
Current Affairs [PDF] - January 16-31, 2020

CPCB (Central Pollution Control Board).


Russia: Production of S-400 missiles begins; China-India-Russia meet in March 2020
On January 17, 2020, Russia announced that the production of S-400 air defence missile system for India
has begun. The missiles are to be delivered by 2025. Russia also announced that the foreign Ministerial
meet of Russia, India and China is to be held in March 2020.
Highlights: Missiles
The upgraded version of the S-400 missiles that were previously available only to Russian forces are to be
delivered to India by 2025.
China, Russia and India
China was the third country in the world to hold S-400 missiles after Russia and Belarus. India was the
fourth country to possess S 400 missiles only after China. Also, it has been announced that China will be
the first country to get S500 missile system from Russia.
S500
The S500 missile is still under development and testing phase in Russia. S500 is very much similar to the
US THAAD (Terminal High Altitude Area Defence) system. S500 is a surface to air missile system with a
range of 600 km.
China-India-Russia meet
Russia also announced that the foreign ministers of China, India and Russia is to be held in March 2020.
This is the 17th time the ministers of the countries are meeting. Russia is also keen that Kashmir issue will
not be brought up at the meet.
Kashmir issue and UNSC
China has lately been raising the Kashmir issue and abrogation of Article 370 in global organizations.
Recently, China tried to bring up the Kashmir issue at the UNSC (United Nations Security Council) through a
closed-door meet. However, 15-member body voted against China and the issue couldn’t be brought for
discussion. China had earlier tried to conduct similar closed-door discussions in August 2019 and couldn’t
succeed.
Amazon to invest 1 billion USD in India; Create 1 million jobs
On January 17, 2020, one of the world’s largest retailer Amazon announced that it has planned to invest 1
billion USD in India and generate 1 million jobs. Globally Amazon has increased its employee base four
times.
Highlights
The retailer has been constantly facing rage of the small and medium enterprises in India for its discounts.
In order to involve small and medium businesses, a mega summit called the smBhav was organized by
Amazon in the capital on January 14, 2020. Around 3000 small and medium entrepreneurs attended the
summit.
Analysis
According to the National E-Commerce Policy, 2019, the Indian E-Commerce market is estimated to reach
200 billion USD by 2026. As India is one of the largest markets in the world, the foreign E-Commerce giants
are competing with each other to capture Indian market. This 1 trillion USD and smBhav moves of Amazon
are best examples.
While it is important to allow the growth of the market, it is also equally important keep the factors
influencing the growth at check. It includes
Regulation of customer data
Stimulating domestic digital economy
Regulatory issues

© 2020 GKToday | All Rights Reserved | https://www.gktoday.in 24


Current Affairs [PDF] - January 16-31, 2020

Export Promotion through E-Commerce. This is the most important factor as India should neither be
used as dumping yard for foreign products nor be used as a service provider for foreign E-Commerce
giants. Rather, local Indian goods and products should grow in terms of production making India a
net exporter.
Legislation
The principal legislation that is governing e-commerce in India is the IT act.
J&K Development: Rs 2000 crore for Z-Morh Tunnel; 36 Union Ministers’ visit
After Abrogation of Article 370, Government of India is rapidly working on developing the territories of
Ladakh and Jammu Kashmir. In order to hasten the process, 36 Union Ministers are to visit the Union
Territory. Also, the concession agreement for the Z-Morh tunnel was signed on January 17, 2020.
Highlights
A concession agreement to complete the 6.5 kilo metre Z-Morh Tunnel was signed. According to the
agreement, a two-lane bi-directional tunnel is to be constructed at a cost of Rs 2000 crores. A parallel
escape tunnel is to be completed in 3.5 years. The tunnel will open NH-1 Srinagar-Sonmarg-Gumri highway
throughout the year. The highway is seldom closed in winter due to snow.
About the Tunnel
The tunnel is capable of allowing 1,000 vehicles an hour at a speed of 80 km per hour. The Tunnel
construction process began in 2012. Border Road Organization (BRO) was the previous implementation
agency of the project. The project was however transferred to IL&FS in 2016. In 2019, on request of IL&FS
due its financial crisis, the project was foreclosed. NHIDCL (National Highway and Infrastructure
Development Corporation) invited for a new concessionaire through bidding process. APCO won the
contract and is to complete the balance work.
Visit of 36 Union Ministers
All the union ministers are to pay a visit to Jammu and Kashmir. The aim of the visit is to spread
information about the policies of the Union Government towards the development of J&K. This will help to
win hope and trust of the people of Jammu and Kashmir.
Tuberculosis: India renames its programme; WHO endorses India developed tech-TrueNAt MTB
On January 17, 2020, Government of India renamed its National TB Control Programme to National
Tuberculosis Elimination Programme. Also, the WHO (World Health Organization) recently endorsed on
technology developed by Indian scientists to detect the disease called TrueNat MTB in terms of accuracy.
Renaming
GoI has renamed the programme to National Tuberculosis Elimination Programme. The name change is in
line with the goal of India in eliminating the disease by 2025. This is five years ahead of the goal of United
Nations Sustainable Development target.
This will give big thrust to people working towards elimination of tuberculosis. Also, with the WHO stating
that TruNat MTB, the molecular test developed by Indian Scientists has greatest accuracy, has increased
India’s confidence in eliminating the disease as planned
TRueNat MTB
The TrueNat test a new molecular test that can diagnose Tuberculosis in one hour. The test uses
polymerase chain reaction to detect bacteria. The device that conducts the test is battery operated.
India’s plans
The TrueNat MTB kit is to be available at community health centres as a first step. Slowly, it is to be
extended to the primary health centres as well. There are around 5,500 to 6,000 primary health centres in
the country.

© 2020 GKToday | All Rights Reserved | https://www.gktoday.in 25


Current Affairs [PDF] - January 16-31, 2020

Leprosy and Polio


Previously renaming was also done for leprosy and polio programmes. In 1983, National Control
Programme was renamed to National Leprosy Eradication Programme. Also, in case of Polio, National
Programme for Control of Polio myelitis was renamed to Polio eradication Programme.
Firsts of Census 2021: Cereals eaten, transgender run houses
For the first time, the Census will collect data of transgenders headed houses. Also, the census will enquire
about “Main cereal”, which is also included for the first time in the history of census of India.
Highlights
The Census 2021 is to ask 31 questions to the citizens. For the first time of Indian history, the following are
to be included
The information about households run by transgender is to be collected. Under the question of sex of
household head, male, female and transgender is to be collected.
The Census 2021 is to be conducted through mobile phone application
Inquiry about main cereal consumed
Earlier it was announced that the census for the first time will collect data on OBC. However, this has
not been included. Rather there is provision of SC/ST in the census.
The questions to be asked are decided by the Ministry of Home Affairs. The census is to begin by
September 2020
What are the questions asked?
During the Census 2021, the following questions are to be asked to the citizens
Questions one to five are related to building numbers, floor material of the house, house number, wall
and roof of the house, condition of the census house and use of census house
Questions six and seven are about total number of persons normally residing, household number
The other questions include caste details, ownership status of the house, access to latrine, LPG
connection, smart phone, car, drinking water availability, main source of lighting, type of latrine, waste
water outlet, main cereal consumed, etc.
Previous Census
The 2011 census clubbed the employment of transgender, their caste and literacy under “Males” category.
Industrial Meet of ‘Wings India’-Asia’s largest Civil Aviation Event held at Bengaluru
On January 18, 2020, the Industrial meet of the Asia’s largest Civil Aviation was held in Bengaluru. The Civil
Aviation event is to be held at Begumpet airport, Hyderabad in March 2020. The theme of the event will be
Theme: Flying for All
Highlights
The Industrial Meet was held at Bengaluru as the city is the IT Capital of India and has the potential to
become an Aviation Capital of the country. The aim of the meet is to unite all the regulators and facilitators
of the aviation sector. It was announced at the event that GoI has planned to set up a world class training
facility for astronauts and pilots together.
Aviation Sector in India
India is the third largest domestic civil aviation market in the world. Also, it is the second fastest aviation
industry in the world. Currently, India ranks 9 in terms of the civil aviation market.
The aviation sector currently contributes 72 billion USD to GDP. It provides job opportunities to 8 million
Indians.
Targets set for aviation sector
India aims to become the third largest aviation market globally by 2024
The number of Indian carriers is to be increased to 1,100 by 2027

© 2020 GKToday | All Rights Reserved | https://www.gktoday.in 26


Current Affairs [PDF] - January 16-31, 2020

The freight traffic at the airports of India is to cross 11.4 MT by 2032.


Government measures
The GoI has taken several measures to improve the sector. It includes permitting 100% FDI in sea planes,
helicopter services, group handling services and non-scheduled air transport services
ELECRAMA-Indian Electrical Industry Showcase inaugurated
On January 18, 2020, the ELECRAMA 2020 was held in Noida. The event showcases innovations and
achievements of Indian Electrical Industry
Significance
The “ELECRAMA” platform is organized by Indian Electrical and Electronics Manufacturers’ Association
(IEEMA). It is supported by other ministries such as ministry of renewable energy, power, MSME (Micro
Small and Medium Enterprises). It acts a platform to connect Indian industries with the world. The event
also helps the world leaders share their ideas and develop cost effective solutions.
About the event
The ELECRAMA is one of the flagship events of Indian electrical industry. It brings complete package of
solutions to every power problem. It includes transmission, generation, distribution, renewables, power
electronics, electro mobility, power storage and automation.
Bottom line the event is important to develop the electrical energy sector in India.
Electrical Energy in India
India is the third largest producer and consumer of electricity. The National Grid of the country has the
capacity of 368.79 GW as of December 2019. Of this 41.16% is used for industrial purposes, 24.76% is used
for residential purposes, 17.69% is used for agricultural purposes and commercial sector consumes 8.24%
of the total electrical energy generated in the country.
Of the 368.79 GW, the private sector production is 46%.
The above data was provided by Ministry of Power.
GoI launches “Seismic Hazard Microzonation” to minimize earthquake disasters
The Ministry of Earth Science has launched Seismic Hazard Microzonation project in major cities of India.
The project aims to reduce hazards of seismic activities by putting in adequate measures
What is Seismic Hazard Microzonation?
It is the process of dividing earth quake prone areas based on certain factors. The factors include
geological and geophysical characteristics of sites. They are ground shaking, landslide, liquefaction
susceptibility, rock fall hazard and flooding caused due to earth quakes. Based on the division, mitigation
measures are prepared. Hence, it is a preventive method to minimise the losses.
The factors in the microzonation method are estimated based on response from soil layers.
Microzonation in India
So far, the mapping has been done in the State of Sikkim and in other eight cities. The cities are Delhi,
Bengaluru, Kolkata, Guwahati, Jabalpur, Dehra Dun, Ahmedabad and Gandhidham.
Japan
Japan is the best example where microzonation techniques are well implemented. The Civil engineering
practice in Japan is based on microzonation laws. This helps to design infrastructure with seismic
resilience.
Need
The technique gives better mitigatory measures as it helps to understand the impact of seismic activities
precisely. In certain places, where no reliable prediction model can be implemented, it is essential to adopt
preventive measures.

© 2020 GKToday | All Rights Reserved | https://www.gktoday.in 27


Current Affairs [PDF] - January 16-31, 2020

January 19, 2020


World Gold Council report: RBI ranks 6th in buying gold abroad
The World Gold Council (WGC) recently released its report, “World Gold Outlook 2020”. The report says that
gold has performed the best since 2010 in 2019. In 2019, the gold trade in USD has increased by 18.4% as
compared to 2010.
Highlights: India
According to the WGC, RBI (Reserve Bank of India) currently holds 625.2 tonnes of gold. This is 6.6% of the
country’s foreign exchange. With this, India ranks sixth in the world among other central banks.
The report says that higher taxes have exacerbated the local gold price consumption. the recently
introduces new hallmarking reforms are expected to increase consumer trust, remove inefficiencies,
promote growth and improve confidence.
Highlights: World
The other 5 countries that have bought more gold than India include China, Russia, Kazakhstan, Turkey and
Poland. The Exchange Traded Funds (ETF) of institutional investors were the second largest buyers of gold
in 2019. The report predicts that they will continue to buy more gold in 2020 as well.
In 2019, the largest gold sellers were Uzbekistan and Venezuela. They sold 16.6 tonnes and 30.3 tonnes
respectively.
World Gold Council
The WGC is a market development organization. The headquarters of WGC is located in London. The offices
of WGC are located in China, India, US and Singapore. The WGC office in India is located in Mumbai.
Myanmar-China sign 33 deals of BRI: Increases China’s ease of access of Indian Ocean
On January 18, 2020, China and Myanmar signed 33 new deals under BRI (Belt and Road Initiative) to
implement China-Myanmar Economic Corridor. Also, a concession agreement was signed by he countries to
implement the deep seaport project called Kyaukphyu Special Economic Zone.
Highlights
The agreements were signed to strengthen the collaboration in infrastructure sector that includes rail
connectivity, roads and power interconnection projects. However, the controversial Myitsone dam project
that is backed by China was not discussed.
India’s Concerns
The project allows China to have easy of access to Indian Ocean. This is because this route bypasses Strait
of Malacca through which China imports most of its oil requirements. Strait of Malacca connects Andaman
Sea and South China Sea. This route is on the international boundary and is far from India’s territories as
compared to the Myanmar route. With China of Ease of access to Indian Ocean, India has to strengthen its
Navy to safeguard its interests in the region. Also, Indian Navy is already witnessing increased Chinese
presence in the IOR.
This will aid in China’s plans of String of Pearls.
Myitsone Dam
The project was proposed across Irawaddy river. It was suspended and the Chinese tried to revive it at an
estimated cost of 3.6 billion USD. The project is controversial as it is located at a 60-mile distance from the
Sagaing fault line.
January 20, 2020
K-4 Nuclear capable Underwater missile test fired by India
On January 19, 2020, India test fired K4, a 3,500 km range nuclear capable missile successfully. The missile
is meant for Arihant-class submarines. Arihant is the first indigenously manufactured nuclear submarine.

© 2020 GKToday | All Rights Reserved | https://www.gktoday.in 28


Current Affairs [PDF] - January 16-31, 2020

Highlights
The K4 missile was test fired from the East Coast of Andhra Pradesh from an underwater pontoon. India
already operates Submarine Launched Ballistic Missile, K-15. However, the range of K-15 is limited to 750
km. The other under water missile being developed by India is BO-5, whose range is 700-km. India has also
begun its research on K-5 whose target is 5,000 km.
K series missiles
The K-4 is an intermediate-range submarine launched ballistic missile. The K-series missiles are named
after former president APJ Abdul Kalam. They are much faster than Agni missiles. The other K-series
missiles are K-15, K-5 and K-6. The K-5 and K-6 are under development by DRDO.
The K-15 missiles also known as Sagarika missiles is a replica to the land-based Shaurya missile. The only
difference is that K-15 are compatible to be deployed from submarines. The K-15 missile gets help from
Indian Regional Navigation Satellite System (IRNSS).
India’s Nuclear Arsenal
India, today, holds 130 to 140 nuclear weapons. It holds 8,300 kg of plutonium that is capable of producing
1000 more nuclear weapons.
India has not signed the Nuclear-Test-Ban treaty and Nuclear Non-Proliferation Treaty.
National Ganga Mission Begins Conservation of wet lands along the Ganges
The National Mission of Clean Ganga (NMCG) has taken up the initiative under Namami Gange Project to
conserve wetlands of the Ganges basin. The primary aim of Namami Gange project is to rejuvenate the river
by increasing its flow. It also includes recharging aquifers and conservation of wetlands.
Highlights
The NMCG along with the State Wetland authorities will identify and prepare plans in conserving wetlands.
Conditions of Wetlands along Ganges
Some of the wetlands are degraded through the drainage, landfill and over exploitation. For instance, the
recently closed Sisamau Nala drainage in Kanpur contaminated the underground water and also river
ecosystem in the region.
Significance of Wetlands
The wetlands provide nutrient recycling, flood and drought mitigation, surface and groundwater recharge.
Wetland Conservation
Ramsar Convention is an intergovernmental treaty that was established in 1971 by UNESCO. It is a treaty
that was signed for the conservation of wetlands. There are 27 Ramsar sited in India.
Steps of India
India differentiates its wetlands into 8 categories based on their regional presence. It includes Deccan
Plateau in the south, saline expanses in Rajasthan, Gulf of Kutch and Gujarat, delta wetlands of India’s East
Coast, lakes and rivers in mountain regions of Ladakh and Kashmir, marshes and swamps in northeast India
and Himalayan foothills and mangroves of Andaman and Nicobar Islands.
National Immunization Day observed in India
On January 19, 2020, the National Immunization Day (NID) was observed all over India as a part of Pulse
Polio Programme, 2020. Around 17.4 crore children of age less than 5 hears were administered polio drops
to sustain polio eradication in India.
Highlights
India has eradicated polio completely in 2012. In order to sustain the eradication, GoI organizes Pulse Polio
immunization campaigns on the standards set by World Health Organization (WHO). The next round of
National Immunization Day is to be held in March 10.

© 2020 GKToday | All Rights Reserved | https://www.gktoday.in 29


Current Affairs [PDF] - January 16-31, 2020

National Immunization Day


The National Immunization Day are one of the four strategies that are recommended by the WHO to
eradicate polio completely. According to WHO, the NID has to be conducted twice a year at a gap of 2 to 4
weeks.
There are two types of vaccines that are being administered in the country. It includes IPV (Inactivated Polio
Vaccine) and OPV (Oral Polio Vaccine).
What are IPV and OPV?
IPV is a wild-type poliovirus strain that is administered in combination of other vaccines. It includes
Diptheria, pertussis, tetanus, haemophilus, hepatitis B and influenza.
IPV is more effective than OPV. It increases intestinal immunity in children who have been already
immunized with OPV.
India to set up Maritime Research Coordination Centre in Sri Lanka
On January 19, 2020, Indian National Security Advisor (NSA) Ajit Doval met Sri Lankan President Gotbaya
Rajpaksa. He discussed setting up of maritime research coordination centre in the country. The
establishment of research centre is important to learn maritime activities in the Indian Ocean Region.
Highlights
The Sri Lankan President and the Indian NSA discussed improving bilateral relations and also to strengthen
cooperation between the armed forces of the countries. India recently extended military assistance of 50
million USD to Sri Lanka. The visit highlights that India is trying to create robust maritime security
cooperation involving Maldives and Sri Lanka.
Maritime Irritants between the Countries
In spite of two maritime agreements that were signed in 1974 and 1976, there are several irritants between
the countries. In 1974 agreement, India agreed to Sri Lanka’s sovereignty over Katchchativu island but with
some safeguards. The safeguards were included by India to protect its fishermen. However, Sri Lanka till
date complains that the agreement failed to give fishing rights to Sri Lankan fishermen.
Also. The Sethusamudram Shipping Canal Project that was undertaken by India to link Palk Bay and Gulf of
Mannar has been stalled due to environmental and livelihood concerns. The canal will cut short the
distances of Indian ships navigating between East and West coasts of India.
Group of Ministers report on legal framework to prevent sexual harassment
On January 19, 2020, the Group of Ministers headed by Home Minister Amit Shah finalized its report to
change the legal framework that will help prevent sexual harassment of women at work places.
Highlights
The Group of Ministers was constituted in October 2018 after the “Me Too” campaign. The group was
reconstituted under the Home Minster in July 2019. Apart from this, the GoI is also working on a project to
change the IPC (Indian Penal Code) that was introduced during British rule in 1860. The ministers have
recommended to strengthen the laws.
Referrals of the report
The report is predominantly based on the following
Vishaka guidelines issued by Supreme Court in 1997. It is be noted that the Sexual Harassment of
Women and Workplace, 2013 was constituted based on Vishaka guidelines
Justice J.S, Verma Committee recommendations. The committee was constituted in the awake of
Nirbhaya Gang Rape and murder in 2012
Verma Committee-The Committee recommended employment tribunal instead of ICC.
Significance
According to NCRB (National Crime Records Bureau), in 2017 and 2018, the number of sexual harassments

© 2020 GKToday | All Rights Reserved | https://www.gktoday.in 30


Current Affairs [PDF] - January 16-31, 2020

of women at work premises were 479 and 401 respectively. The cases were registered under IPC Section
509 and were highest in cities such as Delhi, Pune, Bengaluru and Mumbai.
Report on Central and State Water Departments based on Efficiency Targets: Gujarat leads
The Ministry of Jal Shakti recently launched rankings of Central and state governmental water departments
based on their efficiency targets. According to the report, Gujarat topped the ranking and Delhi was pushed
to the last.
Highlights
Among the seven central government departments, Survey of India topped the list followed by National
Institute of Hydrology and Central Water Commission. The Central Pollution Control Board received the
lowest rank.
In 2018, Telangana received the top most rank and Gujarat was at seventh position. On the Delhi, that had
received 35th rank in 2018 was pushed to the bottom most, which is 41.
Tamil Nadu was the best performer among all the states. The state was at 33rd position in 2018 and has
been pushed to 13th rank this time. Rajasthan that was ranked 16th in 2018 was awarded the best water
management status. This was mainly because, Rajasthan had set up SCADA (Supervisory Control and Data
Acquisition), a computer system that gathers and analyzes real time data.
Aim is the ranking
The ranking was done as a part of mid-term review of the central and state government departments. It also
aims to create Water Resource Information System (WRIS) in all states. This will help to create real time
NWIC (National Water Information Centre).
Performance parameters
The departments and states were ranked based on real-time data acquisition, analytical work, procurement,
data digitization, trainings, updating MIS and analytical work. Based on these parameters, the government
agency was awarded with a score between 0 and 100.
1 trillion USD required to achieve UN’s Decarbonization Target of Shipping Industry
The United Nations had set the target of reducing carbon emissions from shipping industry by 50% by 2050
as compared to the levels in 2008. A study of London based Energy Institute (University College London’s
Energy Institute) suggest that in order to achieve the target, the world has to spend 1 trillion USD.
Highlights
According to the United Nations, the emissions from shipping account to 2.2% of world carbon-dioxide
emissions. Reducing shipping emissions is ambitious and challenging as the shipping sector plays a major
role in the growth of the economy. 90% of world trade happen at sea.
The study says that in order to achieve the target set by United Nations, it is essential to invest at least 50
to 70 billion USD annually.
The Decarbonisation target was initiated by the International Maritime Organization.
When was the resolution adopted?
The International Maritime Organization regulates international shipping and its green house gas emissions.
The shipping industry was the only sector that was left out of the 1997 Kyoto protocol and its successor
2015 Paris agreement.
However, in April 2018, it was the IMO (operated under UNESCO) that adopted the resolution. According to
the resolution the strategy to achieve the target begins in 2020.
Delhi Government releases Guarantee card on free bus services, water & electricity
On January 19, 2020, the Delhi Government released the guarantee card that consisted of 10 guarantees for
the residents of Delhi. It included free bus services, electricity, water supply, pollution, etc.

© 2020 GKToday | All Rights Reserved | https://www.gktoday.in 31


Current Affairs [PDF] - January 16-31, 2020

Electricity
The card guaranteed uninterrupted power supply throughout the year. It also included free electricity for all
the residents up to 200 units of consumption. The Government has also assured to replace the overhead
electrical wires with underground cables.
Water
The card also assured supply of 24/7 clean drinking water to every household of the Territory by 2025. It
also added that 20,000 litres of free drinking water scheme that is being currently implemented will
continue along with other schemes
Infrastructure
The Guarantee card included free education, better health facility and mohalla clinics for those living in
unauthorized colonies. It also promised to build pucca houses under “Jahan Jhuggi Wahin Makan” scheme
for those living in slums.
Other Guarantees
The Guarantee card also aims at providing cheapest transport facilities. It has offered to run free bus
services to all students and women in the national capital. It also guarantees to make Delhi safer for women
by deploying “Mohalla Marshals”.
Above all, the guarantee card lists to reduce air pollution of the capital region.
National Agricultural Export Authority, APEDA launched 186 product testing laboratories
On January 20, 2020, the APEDA (Agricultural and Processed Food Products Development Authority) has
added 135 laboratories to the existing 51 laboratories. With this initiative, the total number of agricultural
product testing laboratories in the country has increased to 186.
Highlights
The new laboratories are being installed in states with high agricultural potential such as Gujarat,
Maharashtra, Andhra Pradesh, Tamil Nadu, Telangana and Karnataka. In order to strengthen the network, a
new policy has been adopted by APEDA. According to the policy, NABL (National Accreditation Board for
testing and calibration Laboratories) accredited laboratories are to be added to the APEDA network.
NABL
The NABL provides the procedure for integrated assessment and requirements for testing laboratories set
up in the country. It works under Department of Promotion of Industry and Internal Trade (DPIIT). Both
NABL and DPIIT operate under Ministry of Commerce and Industry.
Agricultural sector in India
Agricultural sector today employs more than 60% of the population. However, it accounts to only 17% of
India’s GDP, which is 2.6 trillion USD. India has achieved self sufficiency in food grain production. India is
one of the highest-ranking countries in terms of rice, dairy, cotton, fruits, meat, vegetables and seafood.
However, due to lack of storage infrastructure, 40% of the produce are lost. The losses alone account to 13
billion USD annually. The export capacity of India can be increased if these losses are minimized.
India prepares for Corona Virus-Steps of Health Ministry
India has been closely monitoring the happenings in China after more than 200 cases were confirmed to be
infected with Corona virus (a new stain). The increase in numbers is mainly because more than millions
travel for Lunar New Year in China.
On January 17, 2020 Thailand reported to have found the first case of the virus following Japan.
Highlights
India is preparing itself after it has learnt from WHO (World Health Organization) that the infections of the
virus are linked to sea food market. Managing the importation of the virus into the country is being
strengthened.

© 2020 GKToday | All Rights Reserved | https://www.gktoday.in 32


Current Affairs [PDF] - January 16-31, 2020

International travelers from China are being examined thoroughly using thermal scanners. These are
currently being done at Mumbai, Delhi and Kolkata international terminals.
The Health Ministry has issued directions to laboratory diagnosis, infection prevention and surveillance of
the virus.
The other measures of Health Ministry is as follows
Ministry of External Affairs has been requested to provide details of travelers from Wuhan city where
the virus is spreading rapidly
The states have been instructed to increase area of surveillance and laboratory support infection
prevention and control.
The Joint Monitoring Group operating under DGHS (Directorate General of Health Services) meets to
assess risks. WHO attends the meeting and provides regular updates.
GoI has issued travel advisory
National Institute of Virology, Pune has been appointed the nodal agency to test the virus samples.
Rapid Response Teams that are trained in MERS-CoV outbreak have been set up in states and union
territories.
Sea Food Scenario in India
The chances of India being the victim of the virus is minimal. This is because India’s imports of sea food
are highly limited. According to 2018 report of MPEDA (Marine Products Export Development Authority),
India exported sea food (fish predominantly) worth of 6,923 million USD and imported 138 million USD. USA
is the predominant import destination of India according to MPEDA. Import of sea food from other countries
is miniscule or negligent. As far as China is considered, Japan is its leading importer of sea food from
China, followed by USA, Hong Kong and Korea.
Though chances are minimal, India is preparing itself to encounter the situation.
NIC organizes Tech Conclave in New Delhi on “Technologies for NextGen Governance”
The National Informatics Centre (NIC) is to hold a 2-day technology Conclave in New Delhi. The event is to
be conducted between January 21, 2020 and January 22, 2020. The Conclave is to be inaugurated by Union
Minister Shri Ravi Shankar Prasad.
Highlights
The Technology Conclave will discuss application of Information and Communication Technology. The
conclave is to be held under the theme
Theme: Technologies for NextGen Governance
The Conclave will predominantly contribute towards building efficient Government officers in the country. It
will also help delivering high-quality services that primarily focus in citizens. The Conclave will focus on
hyperscale, cyber security, design thinking and architecture.
NIC plays a major role in adopting new technologies within the administration of government. Recently, NIC
launched a Centre for Block Chain Technology in Bengaluru.
India’s E-Governance
Recently India has been adopting electronic governance. With subsidy transfer to census 2021, GoI has
been adopting technologies for the governance to reach citizens at a faster rate. Some of the highly
successful e-governance initiatives include
National Land Record Modernization Programme-It aimed at Computerization of land records
Bhoomi Project-It is online delivery of land records
National E-governance plan has been formulated to improve delivery of government services to
citizens.
E-Office was launched to increase usage of work flow.

© 2020 GKToday | All Rights Reserved | https://www.gktoday.in 33


Current Affairs [PDF] - January 16-31, 2020

January 21, 2020


UNCTAD releases Global investment Trend report: India among top 10 recipients of FDI
On January 20, 2020, the United Nation Conference on Trade and Development released its report on Global
Investment Trend Monitor Report for the year 2019. The Global FDI of the year 2019 was 1.39 trillion USD.
This has declined by 1% as compared to 2018.
Highlights: World
According to the report, FDI in South Asia increased by 10% as compared to 2018. The inflows of Pakistan
and Bangladesh however declined by 20% and 6% respectively. China remained the topmost FDI recipient in
the world as it was in 2018 followed by the United States. The United States in 2019 attracted 251 billion
USD FDI and the inflows of China was 140 billion USD.
The report also said that the inflows into developing economies remained unchanged. The FDI in Latin
America and Caribbean rose by 16%. The FDI in African countries were 3% and was far less as compared to
other parts of the world.
Highlights: India
The growth of FDI in South Asia was mainly due to India. In 2019, the India’s FDI was 49 billion USD. The
report says that FDI in 2019 has increased by 16% as compared to 2018. The majority of the input in India
went into Information Technology.
Predictions of 2020
The report predicts that FDI flows in the country will increase in 2020.
“TIGERSHARKS”, First Squadron of Sukhoi-30 MKI aircraft inducted by IAF in Tamil Nadu
On January 20, 2020, the 222 Squadron of Sukhoi-30MKI fighter jets was inducted by the Indian Air Force in
Thanjavur, Tamil Nadu. The squadron has been named “Tiger Sharks”.
Highlights
The squadron has been inducted with modified Sukhoi-30MKI fighter jets that are modified to carry
BrahMos missiles.
Background
The 222 Squadron of Sukhoi-30MKI squadron called the Tiger Sharks was first raised in Ambala, Haryana in
1969. It also took part in 1971 Indo-Pakistan war. In 1985, Tiger Sharks were the first squadron to be
equipped with MiG-27.
In 2018, during the Gagan Shakti-2018 Indian Air Force Exercise, the capabilities of the aircraft were
demonstrated at large scale.
Tiger Sharks
So far there are 12 squadrons of Tiger Sharks inducted in India including the one recently inducted in Tamil
Nadu. The other 11 were deployed in Pune, Halwara, Sirsa, Jodhpur, Tezpur, Bareilly, Chabua.
Why Tamil Nadu?
The Squadron is being inducted in Tamil Nadu to check the fast-expanding strategic footprint of China in
the Indian Ocean Region. China had established its first military base in Djibouti in the region surrounding
Horn of Africa. In fact, the base at Djibouti was the first overseas army support base established by Chinese
People’s Liberation Army. The squadron will help to check China’s fast-growing footprint in Indian Ocean.
China’s Malacca Dilemma
China has been constantly finding ways to reduce its dependence on Straits of Malacca. This is to ensure
long term energy security policy as piracy in the route is increasing. This policy of China seeking alternate
routes is called “Malacca Dilemma”. It was introduced by President Hu Jintao in 2003.
Malacca Dilemma’s alternate routes
China on the lines of Malacca Dilemma has opened oil and gas pipelines in 2013 between Port Sitwe (in
© 2020 GKToday | All Rights Reserved | https://www.gktoday.in 34
Current Affairs [PDF] - January 16-31, 2020

Myanmar waters) and Chinese Kunming in Yunnan province. Also, China is developing Gwadar port of
Pakistan on the side lines of BRI initiative. A proposal to construct a canal across Isthmus of Kra is still
under consideration.
Carbon Disclosure Project: India ranks 5th
On January 21, 2020, the Carbon Disclosure Project, an annual report on carbon reduction activities of
different countries and their firms was released by a non-profit organization Global Reporting Initiative.
According to the report, the countries were ranked based on their corporate responsibilities of carbon
reduction. In order to assess the role played by them, their science-based targets were weighed.
Highlights
According to the report, 58 Indian companies disclosed their environment related activities to the
organization. Based on their disclosure, the report concludes that 98% of the top Indian companies have
now formed committees to address climate related issues. The change has been witnessed between 2018
and 2019.
United States topped the list with 135 companies being transparent in their disclosure. Followed by US,
Japan ranked second with 83 companies. On the third, fourth and fifth ranks were UK (78), France (51) and
India (38) respectively.
The report said that, over 6,900 companies have so far disclosed their data through Carbon Disclosure
Project. This contributes to 55% of world capitalization.
Global Reporting Initiative
The Global Reporting initiative helps businesses, organizations and governments to understand the impacts
of climate change, corruption and human rights.
GoI launches paperless licensing of petroleum service stations
On January 20, 2020, the Department of Industry and Internal Trade (DPIIT) announced that it has launched
paperless licensing process for petroleum service stations. The paperless licensing has been launched
under Petroleum Rules, 2002. The implementing agency of the order would be Petroleum and Explosives
Safety Organization (PESO).
Highlights
The issue of paperless license for road tankers was launched earlier this month. After its launch, 300
licenses have been issued so far. With the paperless licensing being extended for petroleum service
stations, it will benefit more than 70,000 pumps and oil marketing companies. Also, the move is important
because, of all the licenses the road tankers and petroleum service station licenses account to more than
85%. Therefore, this will benefit in decreasing the transit time of fuel as a greater number of vehicles can
easily be employed in the process.
PESO
PESO is a department formed under DPIIT and both the bodies work under Ministry of Commerce and
Industry. PESO administers the following acts
Explosive Act, 1884
Petroleum Act 1934
Explosive Substance Act
Inflammable Substance Act 1952
Environment Protection Act 1986
It controls the export, import, storage, transport and usage of explosive materials.
The authority has framed several rules such as Explosive rules, 2008, petroleum rules, 2002, Gas Cylinder
rules 2002, Calcium carbide rules, 1987, Ammonium nitrate rules, etc.

© 2020 GKToday | All Rights Reserved | https://www.gktoday.in 35


Current Affairs [PDF] - January 16-31, 2020

Global Social Mobility Index launched at World Economic Forum: India ranks 76th
On January 20, 2020, the World Economic Forum released the Social Mobility Index. The index measures
social mobility of the population world wide.
What is Social Mobility?
Social Mobility is the upward or downward movement of an individual in personal circumstances in relation
to their parents.
The countries’ social mobility was measured based on following 5 key dimensions as follows
Health
Education-Quality, Access and Equity
Technology
Work-Wages, conditions and opportunities
Protections and Institutions-Inclusive institutions and social protection
Relative Social Mobility is the assessment of effect of socio-economic background on a person’s life.
Findings of the Report
According to the report, Denmark ranks the first followed by Finland, Norway, Sweden and Iceland. India
ranked 76th out of 82 countries.
According to the report, India has improved in the areas of fair wage distribution and social protection. Also
India performed better in terms of life-long learning and working conditions. The countries that were
benefited out of social mobility includes US, China, India, Germany and Japan.
Impact of 4th Industrial Revolution
According to the report, the fourth industrial revolution has created several social inequalities. It is
disrupting labor market by inducing wage disparities.
January 22, 2020
Claims against India under Bilateral International Treaty dismissed
The Bilateral International Treaty organization, an international arbitration tribunal recently dismissed all the
claims against India.
Highlights
The claims against India was brought the organization by private firms in Cyprus and Russia. The claims
were under Agreement between Government of Russian Federation and Cyprus for promotion and protection
of investments. The claims have now been dismissed completely.
Bilateral Investment Treaty
BIT is an agreement that was established for FDI. It allows countries to set up rules for investments
th
between each other. The treaty is the successor of the 19 century friendship, commerce and navigation
treaty.
The first BIT was signed between Pakistan and Germany.
BIT ensures that foreign investors are treated equally as that of domestic investors. This is because, foreign
investments are restricted by governments to promote domestic entrepreneurs. For instance, US restricts
foreign investment in only 5 sectors and China restricts foreign investment in 100 sectors.
Significance
India entered BIT in the mid-90s. It is important for India to sign BIT to make conditions favourable for
foreign investors. While most of the treaties in the world focus on ‘most favored Nation’ treatment, BIT
treats foreign countries equally.
India and BIT
According to the data of Ministry of Finance, India has so far signed BIT with 75 states. Out of these 66 are
already in force and 9 are yet to be entered into force. India has signed 40 BITs with less developed and
© 2020 GKToday | All Rights Reserved | https://www.gktoday.in 36
Current Affairs [PDF] - January 16-31, 2020

developing nations.
As every other country, India has set its own BITs clauses. BIT clauses of India is not the same for all the
countries. The most important feature of India’s BITs is that it does not provide right to make investment in
India to the signatories. Every investment has its own rules fixed by India.
Women Power at Republic Day parade: Tania Shergil and CRPF bikers
The Republic Day parade held at Rajpath every year is one of the largest parades of the world. The parade
marches from Rashtrapati Bhawan along Rajpath until India Gate. This year the parade has certain events to
express the woman power in the defence sector.
Women Power
Tania Shergil
In 2020 republic day parade, Tania Shergil becomes the first woman officer to lead all-men contingent. The
Contingent was commissioned in March 2017 from Officer Training Academy, Chennai.
First Women CRPF Bikers
All women biker contingent of the Central Reserve Police Force will make their debut showcasing daredevil
stunts at the Republic Day parade. The team consists of 65 women CRPF personnel and will display their
acrobatic skills on Royal Enfield Bullet motorcycles.
The Contingent is to be commanded by inspector of Rapid Action Force. Rapid Action Force is an anti-riot
combat unit of Central Reserve Police Force. CRPF is the world’s largest paramilitary forces with more than
3.25 lakh personnel.
Other firsts of woman power
The CRPF raised the first all women battalion in Asia region in 1986. In 2018, the an all women bikers team
of the Border Security Force made a similar debut. In 2015, an all women marching contingents of navy,
army and air force debuted the national parade.
Start Up India-first time in Republic Parade; Indian Navy
The Department of Promotion of Industry and Internal Trade (DPIIT) operating under Ministry of Commerce
and Industry is to showcase tableau on Start Up India at the Republic Day Parade to be held at Rajpath, New
Delhi. The tableau is to be held under the theme
Theme: Reach for the sky
This is the first time Start Up India is participating at the Parade.
Highlights
The following are to be represented at the parade
The Tableau will showcase the stages of life cycle of a start-up
The front of the tableau depicts creative mind, middle represents kinds of support that are being
provided to start ups by the GoI and wheel will depict sectors of Indian Economy that have driven
growth employment generation.
The Map of India is to represent the spread of start-up India scheme
According to DPIIT, around 26,000 startups have been recognized. It has reached 551 districts in 28 states.
Today, the startup initiative attracts global investments and has created more than 2,91,000 jobs.
Indian Navy
The Indian Navy is to showcase its tableau at the Republic Day parade under the theme
Theme: Indian Navy-Silent, Strong and Swift
It is with the same theme the Indian Navy week was celebrated. It will showcase the fire power of Navy in all
three dimensions namely sub-surface, surface and air. The Kalvari class submarine, Harpoon missiles, MiG
29K fighter jets are to be part of the Tableau.

© 2020 GKToday | All Rights Reserved | https://www.gktoday.in 37


Current Affairs [PDF] - January 16-31, 2020

India Nepal inaugurate new check post, Jogbani-Biratnagar at the border


On January 21, 2020, Indian Prime Minister Narendra Modi and Nepalese PM Oli inaugurated an integrated
check post (ICP) at Jogbani-Biratnagar. This is the second check post that is being built at the border with
Indian Assistance.
Highlights
The aim of the new integrated check post is to facilitate trade and movement of people across the border.
The check post was built under assistance of India. India helped in the construction based on its
“Neighbourhood First Policy”. The check post was built over 260 acres with Indian assistance of Rs 140
crores.
The first integrated check post of the countries was built at Raxaul-Birgunj in 2018.
Significance
The check posts ensure better border management for the movement of passenger and cargo. They will
also help to boost 7 billion USD trade between the countries.
What is Integrated Check Post?
Integrated Check Posts are trade facilitation centres at borders of India. The centres help boost bilateral
trade and also help to improve movement of passengers in the border. This helps to generate employment,
developing the area surrounding the check post. Ultimately, it helps in overall growth of the economy.
There are 7 Integrated Check Posts in India. The GoI is planning to construct 14 more ICP and 13 Land
Customs Stations. The India-Pakistan ICP is located in the states of Punjab and Rajasthan. The following
are the other ICPs in the border located at the corresponding states
India-Nepal-Uttar Pradesh, Bihar, West Bengal and Uttarakhand.
India-Myanmar-àManipur and Mizoram
India-Bhutan-àWest Bengal
India-Bangladesh-àAssam, Mizoram, Meghalaya, Tripura and West Bengal.
32nd interaction of PRAGATI held: Rs 24,000 crores worth projects discussed
On 22 January, 2020, PM Modi chaired the 32nd PRAGATI meet. Under the Pro-Active Governance and Timely
Implementation, the Prime Minister reviewed projects that are being implemented by the union government
and their progress.
Highlights
The Prime Minister discussed 24 projects that were of worth Rs 24,000 crores at the meet. The projects
were spread over 9 states namely Telangana, Odisha, Maharashtra, Karnataka, Jharkhand, Andhra Pradesh,
Uttar Pradesh, Bihar and Kerala. It included three projects from Ministry of Railways, one from Ministry of
Petroleum and Natural Gas and five from Ministtry of Road Transport.
Also, the projects under Pradhan Mantri Jeevan Jyoti Bima Yojana, Crime and Criminal Tracking Network
Systems and Pradhan Mantri Suraksha Bima Yojana were discussed
Crime and Criminal Tracking Network Tracking System (CCTNS)
The CCTNS was established in 2009 to create comprehensive and integrated system for effective e-
governance. It will interconnect 15,000 police stations and digitize data related to FIR registrations. This
aids in developing developing a national database of criminals and crimes.
Previous PRGATI outcomes
At the 31st PRAGATI interactions, projects worth 12 lakh crores were reviewed. It also discussed grievances
of Indian citizens working abroad, Aspirational District Programme, National Agricultural Market and
infrastructural development programmes. Around 9 projects that were of 61,000 crores of rupees were
taken up for discussion. It included Union Territory of Jammu and Kashmir as well.

© 2020 GKToday | All Rights Reserved | https://www.gktoday.in 38


Current Affairs [PDF] - January 16-31, 2020

PRAGATI
PRAGATI was launched by PM Modi in March 2015. It is an interactive platform to address grievances of
common people. The issues faced by the implementing agencies are uploaded in the platform on third
Wednesday every month.
India joins Reskilling Revolution Initiative at World Economic Forum as a Founding Member
On January 22, 2020, India joined Reskilling Revolution initiative of World Economic Forum. It helps
economies to adopt o changes being introduced by Fourth Industrial Revolution.
Highlights
India joined the Reskilling Revolution Initiative launched at the World Economic Forum as founding member.
The other founding members include US, UAE, Russia, Pakistan, France and Brazil. The initiative aims at
providing better education, jobs and skills for more than 1 billion people in the world by 2030.
Significance
According to the World Economic Forum, around 75 million jobs are to displaced. This is predominantly
because of technological integration and automation. The transformation will also create large scale
unemployment affecting the labour population greatly. It will also create demand for 133 million new jobs.
Therefore, it is essential to shift the labour population towards the requirement of the technology upgrade.
In order to achieve this, the Reskilling Revolution Platform has been launched.
Report
In order to achieve the reskilling, the World Economic Forum also released a report namely “Jobs of
Tomorrow: Mapping Opportunity in the New Economy”. The report mapped 96 new emerging jobs in seven
fastest growing professions based on new technologies.
Fourth Industrial Revolution
The transformation is happening mainly due to fourth industrial revolution. Fourth Industrial Revolution is
the developing environment where robotics, Internet of Things and Artificial Intelligence are changing the
lives of people globally. It has increased machine operated automation and changed the labour force
involved.
World Economic Forum Survey: World loses hopes in Climate Science; Indians are most-trusting
The World Economic Forum published its opinion poll during its 50th annual meeting in Davos. The annual
meeting was called for urgent upskilling and better climate education.
Highlights
According to the survey, India and Bangladesh are the most trusting people in climate science. The citizens
of both the countries keep themselves updated about the happening of the world. Also, they trusted the
climate scientists and their discoveries. India was followed by Bangladesh, Pakistan, China and Turkey.
On the other hand, Ukraine and Russia lagged the most when it comes to updating themselves about
climate science.
The report says that the world has lost its trust in climate science. According the survey, the world believes
that global warming is mainly due to human activities.
About the Survey
The Survey was held in 30 countries covering 76% of world population. The survey has been presented on
the 2020 theme which is
Theme: Toward More Sustainable World and Toward a more Cohesive World.
Significance
The survey is important to learn about the status of mind and opinion of world population in different part.
This forms the base to organize campaigns. The survey will now act as the major decision maker when it
comes to design the type of awareness campaign to be adopted in a specific region.

© 2020 GKToday | All Rights Reserved | https://www.gktoday.in 39


Current Affairs [PDF] - January 16-31, 2020

World Economic Forum: 1 trillion trees and report on Password role in Cyber Crime
The World Economic Forum began on January 21, 2020 in Davos. The highlight of the forum this year is over
its initiative to plant 1 trillion trees in the next decade and also to equip 1 billion people with fourth
industrial revolution. The forum has released a report on the role of passwords in cyber crimes. According
to the report, fourth industrial revolution reduces cost spent on maintaining passwords and their secrecy.
Highlights: 1 Trillion Trees
The World Economic Forum has joined the “Tree Trillion Campaign” that was launched by UNEP (United
Nations Environment Programme) in 2006 as “Billion Tree Campaign”. In 2011, the project was handed over
to Plant for the Planet (a NGO).
The forum has launched 1t.org platform in order to support the campaign. Through the platform, the forum
aims to bring together the leading businesses, governments, civil societies and entrepreneurs all over the
world.
The initiative is being supported by the US, though it secluded itself from the initiative earlier.
Highlights: Role of Password
On January 22, 2020, the World Economic Forum released report on passwords and the cost involved in
maintaining their secrecy. According to the report, cybercrime is to cost 2.9 billion USD to the global
economy. Around 50% of IT Help Desk costs are spent on password resets. Therefore, it is essential to shift
the password-based system towards Artificial Intelligence and other fourth Industrial Revolution
technologies.
The report suggests top 5 technologies for password protection. It includes behavioural analytics,
biometrics, QR codes, zero-knowledge proofs and security keys.
January 23, 2020
Facts Box: “Three Capital Bill” of Andhra Pradesh
The Andhra Pradesh Government has passed the “Andhra Pradesh Decentralisation and Equal Development
of All Regions Bill”, 2020. The bill paves way for three capitals of the state.
Highlights
The three capitals are Amaravati-legislative capital, Vishakhapatnam-the executive capital and Kurnool-
judicial capital. The three-capital system is being adopted by the government of Andhra Pradesh in order to
ensure equal development throughout the state. This also aids in decentralization of government. The
above cities were selected as capitals for their strategic location to aid decentralization.
Committees
The cities were recommended by the committees namely B N Sri Krishna Committee, G N Rao Committee
and K Sivaramakrishnan Committee.
Indian States
There are other states in India with two capitals as well. It includes Maharashtra, Himachal Pradesh and
Jammu and Kashmir. The capitals of Maharashtra are Mumbai and Nagpur. The state holds winter session
of the state in Nagpur. Similarly, the capitals of Himachal Pradesh are Shimla and Dharamshala (Winter).
The capital of Jammu and Kashmir were Srinagar and Jammu (winter).
Other Countries
The other countries that have multiple capitals include Sri Lanka and Malaysia. In Sri Lanka,
Jayawardenepura Kotte is the seat of national legislature and is the official capital and Colombo is the seat
of judicial bodies and national executives.
National Startup Advisory Council setup to boost Startups and Innovations
On January 22, 2020, the Union Ministry of Commerce and Industry has setup National Startup Advisory
Council. The council will suggest ways to build strong start ups in the country. It is being set up to reduce
© 2020 GKToday | All Rights Reserved | https://www.gktoday.in 40
Current Affairs [PDF] - January 16-31, 2020

the time spent on tax compliance to less than one-hour a month and to boost the availability of domestic
and global capital.
India ranks 136 in World Bank’s “Starting a Business” ranking. The council is being set up to improve India’s
rank globally.
Composition of the Council
The council is to be chaired by the Minister of Commerce and Industry. The Council will have non-official
members nominated by the Union Government. The non-official members will hold office for two years.
The members will be selected from different categories such as founders of successful startups, persons
capable of representing interests of accelerators and incubators, etc.
The council will also include Ex-officio members. According to the ministry, they will be of the rank of Joint
Secretary. The DPIIT (Department of Promotion of Industries and Trade) will be the convener of the council.
Functions of the council
The Council will suggest measures to spread culture of innovation among all the citizens and students. This
will help to promote innovation among all sectors of the economy widely. It will also recommend measures
to facilitate public organisations. This will help to deliver better governance, promote creation and
commercialize intellectual property rights.
The Council will also suggest ways to make the process of starting, operating and exiting a business
simpler. It will also mobilize investments in startup sector and will provide access of the startups to the
global markets.
India-Brazil: BIT to be signed; Cabinet approves agreements on legal assistance, Oil
On January 23, 2020, India and Brazil will sign the Bilateral Investment Treaty during the visit of Brazilian
President Jair Bolsonaro. The President is visiting India as a Chief Guest of Republic Day celebrations.
Highlights
The President of Brazil is to visit India between January 24, 2020 and January 27, 2020. The countries will
sign 20 agreements during his visit. They will upgrade their Strategic Partnership Action Plan. The plan is to
serve as the base for the agreements to be signed. The countries are expected to sign agreements in the
sectors of technology, cooperation, logistics and sharing.
Cabinet Approval
On January 23, the union cabinet approved agreements between India and Brazil on Mutual Legal
Assistance in criminal matters and in the sector of oil and natural gas.
Mutual Legal Assistance
According to the agreement, the countries will aid each other in investigation of crimes. This will help to
track, confiscate and restrain instruments of crime and also the funds used to finance terrorism.
It is essential for India to sign such crime record sharing agreements with a greater number of countries to
maintain internal security. This helps to track the criminals and prevent extradition. It will also help to
hasten investigation processes and help end the cases with right justice.
Oil and Natural Gas
The Cabinet also cleared the agreement signed by India and Brazil on Oil and natural gas. According to the
agreement, the countries will work towards establishing cooperation in production and exploration of oil
and natural gas.
Brazil
Brazil is the 12th largest oil producer in the world. With raising tensions between US and Iran and increasing
oil prices, it is important for India to search for alternative to keep oil prices under control.
India participates at United Nations AIDS Programme at World Economic Forum
On January 21, 2020, India participated at the United Nations AIDS (UNAIDS) programme at the World

© 2020 GKToday | All Rights Reserved | https://www.gktoday.in 41


Current Affairs [PDF] - January 16-31, 2020

Economic Forum (WEF). The theme of the programme was


Theme: Access for all: Leveraging Innovations, Investments and Partnerships for Health
Highlights
The health needs, technologies and solutions to bridge economic inequalities in the health sector were
discussed. India was represented by Union Minister of state for Shipping and Chemical and Fertilizers
Mansukh Mandaviya.
UNAIDS Programme
The main objective of the programme is to coordinate global action on the AIDS disease and alleviate its
impact. It was established in 1994.
AIDS in India
The AIDS programme in India is implemented by NACO (National AIDS Control Organization). It formulates
policy and implements prevention and control programmes of AIDS in the country.
According to UNAIDS data, 2018, around 2.1 million people are living with AIDS in the country. In 2018,
69,000 AIDS related deaths occurred and 88,0000 new HIV infections were recorded. However, compared to
its neighbouring countries India has made good progress to bring the disease under control.
UNAIDS also says that India is the third largest country with HIV epidemic in the world.
NACO
NACO is the nodal organization to implement AIDS programmes of Indian Government. Apart from this, it
also conducts HIV estimation biennially along with ICMR (Indian Council of Medical Research). The first
round of such estimation was conducted in 1998 and the last was done in 2017.
Democracy Index released: India ranks 51; Slips 10 positions
On January 22, 2020, the Economic Intelligence Unit released Democracy Index for the year 2019. The index
was topped by Norway. Norway was followed by Iceland and Sweden at second and third positions
respectively. India was pushed to 51st rank slipping 10 positions as compared to its previous ranking
Highlights
According to the report, the slippage of India’s ranking was mainly due to the protests of Citizenship
Amendment Act (CAA), National Register of Citizens (NRC) and abrogation of special status to Jammu and
Kashmir.
Report on Kashmir
With respect to the Kashmir issue of Abrogation of Article 370, the report says that certain acts of
government over the issue led to the decline in the country’s ranking. It includes deployment of large troops
in Jammu and Kashmir region, restricting internet access and also placing local leaders under house arrest.
Report on NRC
The report said that the citizenship registration exercise excluded 1.9 million people from the final list. Also,
the majority of the population excluded were Muslims. This has enraged large Muslim population and has
stoked protests in various parts of the country.
History of India’s Index
The democratic indices are awarded over a score of 10. Since the launch of the Index in 2006, India has
obtained the lowest score of 6.9 this time in 2019. In 2018, India obtained 7.23. India’s highest score was
7.91 in 2014 and has declined since then. However, India’s scores have always remained above global
average. This year, the global average was 5.44.
January 23: Birth Anniversary of Subhash Chandra Bose celebrated
On January 23, 2020, India celebrated 123rd birth anniversary of Subhash Chandra Bose. He was born on
January 23, 1897. In order to commemorate Netaji’s birth anniversary, GoI is inaugurating a temple at
Varanasi in the name of the leader with a Dalit woman as a chief priest. The prayers are to be performed to

© 2020 GKToday | All Rights Reserved | https://www.gktoday.in 42


Current Affairs [PDF] - January 16-31, 2020

Bharat Mata at the temple.


About Netaji Temple
The temple is to be built in red, white and black colours. The red colour symbolizes revolution, black
represents strength and white for peace. The temple has been built to inculcate patriotism among the
citizens.
Azad Hind Fauj
Netaji formed a separate army called the “Azad Hind Fauj”. It was also called the Indian National Army or
the INA. There were around 85,000 soldiers in INA.
About Netaji
Netaji did his schooling in Cuttack. He also passed the Indian Civil Service exam. However, he joined the
non-cooperation movement and fought for India’s independence. He was elected as the president of Indian
National Congress in 1939. However, he was expelled due to his differences with the other Congress
leaders.
Controversies of Netaji’s Death
Different theories came with death of Subhash Chandra Bose. However, in 2017 GoI in its reply under Right
to Information act, confirmed that the leader had died on August 18, 1945 in a plane crash. He was 48 years
old when he died.
Ghana inspired by India’s Ujjwala; India to assist; MoU signed
On January 23, 2020, Ghana announced that the country will replicate the Ujjwala model of LPG connection
under the guidance of India (represented by Indian Oil Corporation).
Highlights
LPG coverage in Ghana is 23%. Citizens of Ghana spend hours together at petrol stations to refill their
cylinders. Ghana is to fix the issues around LPG with India’s support. Ghana inspired by India’s bottling
plants of LPG is to implement its replica aiming to cover 50% of population in the initial stage.
MoU
India has signed a Memorandum of Understanding with Ghana to provide assistance in Ghana’s National
LPG Promotion Policy. The MoU will strengthen cooperation between the countries and will increase the
support offered to National Petroleum Authority of Ghana by India.
Indian Oil Corporation will aid in developing infrastructure, technical support and value chain on the
sidelines of the agreement. This symbolizes India as a global leader in providing clean energy.
Achievement of PMUY
Under Pradhan Mantri Ujjwala Yojana, 8 crore cooking gas connection were provided to poor women in less
than 3 years. The Programme was launched to safeguard health of children and women by replacing
firewood with LPG.
Controversies
India’s CAG (Comptroller and Auditor General) in its report on PMUY expressed its concerns over delays in
supply of cylinder, low consumption, diversions of LPG, etc.
Facts Box: ILO’s Labour Report, 2020
The International Labour Organization released its “The World Employment and Social Outlook: Trends
2020” (WESO) report. The report stated that the global unemployment has been stable for the last nine
years. However, slowing economic growth means that for the increasing labour force, no new jobs are
created.
Highlights: World
The report has projected that the unemployment is to increase around 2.5 million in 2020. Currently there
are 188 million people unemployed in the world. This means one in five of the workers worldwide are living

© 2020 GKToday | All Rights Reserved | https://www.gktoday.in 43


Current Affairs [PDF] - January 16-31, 2020

in poverty.
The report also says that around 165 million people in the world do not have enough paid work and 120
million lack access to labour market. In all, more than 470 million are affected due to unemployment
problems.
Highlights: India
According to the report, the growth in countries of India and china in the past 13 years has caused the
global labour inequality to reduce. However, the labour income distribution within these countries has not
changed. The Gender gap in wages of the labours is more in South Asian countries such as India, Pakistan
and Bangladesh as compared to the rest of the world. While it stands at 15% in developed nations, in these
countries the gender gap of wages is 40%.
Defence Ministry approved procurement of 5,100 crores of hardware
On January 22, 2020, the Defence Acquisition Council approved the Procurement of defence equipment that
are of worth Rs 5,100 crores. The equipment are to be purchased from indigenous sources.
Highlights
The equipment to be purchased include electronic Warfare systems that are manufactured by DRDO. It also
includes other equipment that are manufactured by local Indian industries.
Also, the council approved the shortlist of Indian Strategic Partners. They will collaborate with the Original
Equipment Manufacturers (OEM) to construct six submarines. According to the approval, the private firms
are to be pulled in partnership with the OEMs.
The Council also approved the inclusion of iDEX (Innovations Defence Exellence) in Defence Procurement
Procedure.
iDEX
Innovations for Defence Exellence was launched in April 2018 to foster technology development in Defence
and Aerospace. It is being funded by Defence Innovation Organization that was formed by HAL And BEL.
Apart from funding the DIO will also provide policy guidance to iDEX.
Defence Acquisition Council
The Council was formed based on recommendations made by Group of Ministers on “Reforming the
National Security System”. The Group was formed after Kargil war, 2001. Defence Minister is the Chairman
of the council.
The Council approves acquisition proposals, transfer of technology decisions, field trial evaluation,
decisions of offset provisions that are above Rs 300 crores.
January 24, 2020
Phase Out of Ozone Depleting Chemicals achieved by India
Ministry of Environment, Forest and Climate Change (EFoCC) recently announced that India has achieved
the complete phase out of ozone depleting Hydro Chloro Fluoro Carbons (HCFC). The achievement has its
route to the Montreal Protocol
Highlights
In order to move towards environment friendly technologies, the Ministry of EFoCC prohibited import of
HCFC-141b. The order was issued according to Ozone Depleting substances (Regulation and Control)
amendment rules, 2019. The rules were issued under Environment (Protection) act, 1986.
The use of HCFC-141b by foam manufacturing industries have been come to an end with the stoppage of
import of HCFC-141b. This is because, HCFC-141b is not produced in India. The domestic requirements of
HCFC-141b so far had been met through imports.
Benefits
The phase out of HCFC-141b will help in healing the ozone layer in the stratospheric zone. Also, it will help
© 2020 GKToday | All Rights Reserved | https://www.gktoday.in 44
Current Affairs [PDF] - January 16-31, 2020

in climate change mitigation as the industries dependent on HCFC for foam production will be forced to
work on alternative technologies that contribute to lesser global warming.
Impacts
The step will have great impact on ozone layer. This is because around 50% of ozone depleting chemicals
being used in the country attribute to HCFC-141b.
On the other hand, polyurethane dependent sectors will face dents. HCFC is used as blowing agent in the
production of polyurethane foams. Polyurethane is used in applications such as water geysers,
refrigerators, thermo wares, furniture appliance, commercial refrigeration.
Govt measures to reduce impacts
The after effect of the phase out will greatly affect the MSME (Micro, Small and Medium Enterprises). In
order to prevent this, the Ozone Cell of Ministry of EFoCC had signed agreements with Central Institute of
Plastics Engineering and Technologies to find alternate technologies. Assistance is being provide to field
trails, training and product validation.
Montreal Protocol
The Montreal Protocol was signed by 190 countries in 1987 to phase out Ozone Depleting Substances. In
order to achieve this, India in 2017 launched a phase out plan. Though the target of the plan was 2023, India
achieved the goal far ahead of the deadline.
“Vyom Mitra”-ISRO’s half Humanoid-An experimental mission of Gaganyaan
ISRO (Indian Space Research Organization) is to send a half-humanoid robot called the “Vyom Mitra” as an
experimental mission to Gaganyaan. Vyom Mitra means a friend in Sky.
Highlights
ISRO introduced Vyom Mitra at the “Human Space Flight and Exploration-Present Challenges and Future
Trends” seminar that was held in Bengaluru. The robot is to fly in two unmanned mission that are planned in
December 2020 and in June 2021.
About Vyom Mitra
The robot can speak two languages namely Hindi and English. It can mimic human activities such as
recognizing other humans and responding to their queries. The robot can handle switch panel operations. It
is also capable of providing air pressure change warnings.
Gaganyaan mission
In order to make the first human space flight programme, Gaganyaan successful, ISRO has been taking
several initiatives. The food for the astronauts is being developed by the Defence Food Research
Laboratory. The astronauts to travel to space received their training in Russia.
Gaganyaan Mission was first recommended by ISRO’s Policy Planning Committee in 2004. In 2006, the
preliminary studies under the name “Orbital Vehicle” began. In 2008, the design to carry two astronauts was
finalized. In 2014, the testing of the launch vehicle GSLV-Mk III was successful. In 2018, the pad abort test
was conducted and succeeded by ISRO scientists. In 2019, astronauts completed their training in Russia.
January 24: National Girl Child Day
India celebrates National Girl Child Day on January 24 annually. It was initiated by Ministry of Women and
Child Development in 2008.
Highlights
The aim of the day is to spread awareness about inequalities faced by girls in the country. It also promotes
awareness about rights of girl child. Also, it increases importance of health, education and nutrition of girl
child. The National Girl Child Day also spreads various atrocities faced by girls in their lives. The
international day of girl child is celebrated on October 11 every year.

© 2020 GKToday | All Rights Reserved | https://www.gktoday.in 45


Current Affairs [PDF] - January 16-31, 2020

Need
Girls in India face humiliation, discrimination and oppression. According to CRY (Child Rights and You) India
is the home of more than one-third child brides in the world. There are more than 10 million child brides in
the world, They are forced into marriage before the legal age of 18
Sex Ratio
Sex Ratio is the ratio of male to female population. Sex ratio of the entire world is 101 males to 100 females
(2018 estimate). In India according to the Census 2011, the sex ration was 108.9. It has been increasing
since 1961. It was 102.4 in 1961, 104.2 in 1980 and 107.5 in 2001.
The natural sex ratio for a society should be between 103 and 107. However, in India it is higher in western
and north western states such as Haryana, Maharashtra, Jammu and Kashmir. The eastern and southern
states have normal sex ratio.
National Data and Analytics Platform released by NITI Aayog
On January 23, 2020, NITI Aayog announced that it will launch National Data and Analytics Platform
(NDAP). The Platform aims to democratize access of government data. In other words, the platform aims to
make the data across all sectors coherent.
Highlights
The NDAP will act as a platform for different stakeholders to access data in an easy way. It will host latest
data from all the government websites. Also, the platform will standardize formats in which data has to be
presented.
The progress of the platform is to be overseen by an inter-ministerial committee. The platform is to be
launched in 2021.
NITI Aayog
The NITI Aayog is National Institution for Transforming India. It is a policy think tank that was established
to achieve sustainable development goals through cooperative federalism. Cooperative Federalism
encourage healthy competition among states.
NITI aayog was established in 2015 replacing Planning Commission. Prime Minister is the chair person of
NITI Aayog. It also includes governing council that is composed of CM of all states and union territories.
NITI Aayog has planned to implement Block chain technology in E-governance. In order to achieve this, the
think tank has conceptualized “India chain”
January 25, 2020
25 January – National Voters Day
National Voters Day (राष्ट्रीय मतदाता दिवस) is celebrated on 25 January every year to mark the foundation
day of Election Commission of India (ECI). ECI was established on 25 January 1950. It’s a constitutional
body deriving its authority from Article 324 of the Constitution and Representation of the People Acts. The
day was started in 2011. Year 2020 marks 10th National Voters Day.
Objective
Objective of National Voters Day is to encourage, facilitate and maximize the enrolment of new voters and
active participation of the active voters.
Theme of National Voters Day – 2020
The theme of National Voters Day – 2020 is ‘Electoral Literacy for a Stronger Democracy.’
January 26, 2020
Ganga-Volga Dialogue held with Russia to enhance Connectivity
The Ganga-Volga Dialogue was held between India and Russia in New Delhi. The dialogue focused in
providing interface between the civilizations of the countries.

© 2020 GKToday | All Rights Reserved | https://www.gktoday.in 46


Current Affairs [PDF] - January 16-31, 2020

Highlights
The dialogue prioritized people-to-people contact. According to Russia, the dialogue acted as a good
platform for its policy of “Greater Eurasia”. The Dialogue was held on the following theme
Theme: Connectivity
The Dialogue also focused on Digital Connectivity, culture, tourism, healthcare, manufacturing technology,
economy, entrepreneurship, etc. The aim of the dialogue is very similar to the International North-South
Transport Corridor.
Proposals Discussed
The dialogue discussed the following key proposals
The impacts of rapidly changing international situation.
Ways of implementation of decisions taken in India-Russia Summit held in September 2019 at
Vladivostok. As an outcome of the summit India agreed to extend a line of credit of 1 billion USD in
order to develop far east Indian companies of Russia.
The development of economic activities in the riverine systems of Ganga and Volga.
Other Discussions
During the dialogue India placed its state of not being into any military alliance in the Quad grouping. This is
important for India to keep its trade and defence relations between with Russia and USA balanced. Also,
India’s policy of SAGAR Indian Ocean were also discussed. The relations between India and CSTO
(Collective Security Treaty Organization) were also discussed.
Department of Biotechnology held International Summit on Women in STEM
On January 25, 2020, the Department of Biotechnology held an International Summit on Women in STEM
(Science, Technology, Engineering and Mathematics) on the following theme
Theme: Visualizing the Future: New Skylines
The summit aimed to increase the participation of women in STEM field.
Highlights
The summit show cased the success of several women in the field of STEM. The summit also hosted
different sessions on empowering women through networking, leadership building, interactive exercises
with scientists.
The summit served as a platform for the students to interact with leaders in the field. This will help them
widely in developing ideas and frame a future plan in their career.
Constitution
According to Article 51A, it is the duty of every citizen to develop scientific temper, spirit of inquiry and
humanism.
Significance of the summit
The National Science Foundation predicts that around 80% of the jobs created in the next 10 years will be
STEM oriented. However, India lags students in the field of problem-solving, innovation and creativity.
Also, women are under represented in the field. According to UNESCO, only 29% of STEM R&D are
contributed by women.
The hinderance in India’s growth in STEM is that the contribution of education system towards STEM is
lagging.
India & World Bank sign Agreement of 210 million USD in agriculture and rural transformation
The Government of Maharashtra and World Bank signed a 210 million USD agreement to facilitate
agriculture value chains, increase market access and build resilient crops.
Highlights
The Project is to be implemented in 36 districts of the state of Maharashtra. Around 43% of farmers and

© 2020 GKToday | All Rights Reserved | https://www.gktoday.in 47


Current Affairs [PDF] - January 16-31, 2020

farm workers are expected to participate in the project. The main aim of the agreement is to facilitate small
farmers compete agri-business investments and build crops resilient to flood and droughts.
It also aims in building Sanitary and Phytosanitary measures
Scenario in Maharashtra
Today, Maharashtra is the largest producer of fruits and vegetables in the country. It is also the largest
exporter of pulses, fruits, cotton, vegetables and soybean. However, the area of cultivation of rice and wheat
in the state has been declining constantly in the state.
Also, 43% of farm-workers and farmers participating in the project are women.
Other Programmes
India is also conducting other programmes under National Action Plan on Climate Change to promote
climate smart technologies and practices and also use of less water.
Sanitary and Phytosanitary Measures
These are measures that are applied to protect animal and human life from the risks of spread of pests,
diseases, toxins and contaminants in food.
India’s first Super Fab Lab inaugurated in Kerala
On January 25, 2020, India’s first Super Fab Lab was inaugurated in Kerala. The laboratory is to function in
collaboration with Massachusetts Institute of Technology (MIT), USA. It is the only laboratory that is being
installed outside US.
Highlights
The Super Fab Lab is being installed in collaboration with KSUM (Kerala Startup Mission). The laboratory
will break down barriers between physical world and digital world.
What is a Fab Lab?
A Fab Lab is a laboratory that offers digital fabrication. It is colloquially defined as, “How to make almost
anything”. It is an array of computers that covers almost all materials. It is used in businesses to develop
prototypes and also in education. It includes CNC cutting, plasma metal cutting, laser cutting, electronics
and microprocessors and 3D printing and scanning.
Vigyan Ashram
Vigyan Ashram was the first Fab Lab to be opened in India. It was established in 2002. Also, it was the first
fab lab to be opened outside USA (MIT).
Fab Fi
Fab Fi is the wireless network that is currently being operated in Afghanistan, US and Kenya. It provides high
speed internet from a central provider.
KSUM
The Kerala Start up Mission is an entrepreneurship development programme that creates environment and
infrastructure to support high-technologies.
Indian Railway’s first Waste to Energy Plant uses “PolyCrack” Technology
The East Coast Railways commissioned the first waste-to-energy plant of Railways sector of India. It is the
fourth such plant in the country. The plant has been set up at a cost of Rs 1.79 crores with Polycrack
technology.
Highlights
The plant installed converts waste collected from the garbage disposal units into carbon powder, diesel and
gas using Polycrack Technology. The diesel oil is to be sold by the Railways to other stake holders for
commercial purposes. The gas and carbon powder are to be used in brick manufacturing. The best thing of
the plant is that it does not leave any waste at the end.
The plant has the capacity to process 500 kg of waste per day and will earn Rs 17.5 lakh per annum.

© 2020 GKToday | All Rights Reserved | https://www.gktoday.in 48


Current Affairs [PDF] - January 16-31, 2020

The first of such plant was established by Infosys in 2011 at Bengaluru. The second plant was established
in Delhi in 2014 and the third was installed by Hindalco in 2019.
Polycrack Technology
In Polycrack Technology, waste is processed and reformed in 24 hours. The only drawback is that it
requires larger area. The technology involves de-polmerization, cracking, reforming, scrubbing, particle
filtration and rapid quenching.
The major benefit of the method is that it does not require pre-segregation.
January 27, 2020
Bharat Parv, a festival of culture being held at Red Fort
On January 26, 2020, Bharat Parv 2020 begun at Red Fort in New Delhi. The programe is to continue till 1st
February, 2020.
Highlights
The main objective of the programme is to increase patriotism among the citizens and also to encourage
them visit tourist places in the country. The programme is being organized by Ministry of Tourism. The
theme of the programme is
Theme: Celebrating 150 years of Mahatma Gandhi & Ek Bharat Shreshtha Bharat
It also aims at developing the spirit of “Dekho Apna Desh”.
About Bharat Parv
Bharat Parv is recognized as National Festival. It includes diversified dances of the country, food courts,
handicrafts from all over the country. The festival also showcases Republic Day Tableaux that participated
in the parade.
Bharat Parv is being celebrated since 2016. It showcases rich cultural diversity of India. Also, it aims to
popularize the idea of “Ek Bharat Shreshtha Bharat”
India-Brazil ink 15 agreements during Brazil President’s visit on Republic Day
The President of Brazil, Mr Jair Bolsonaro visited India to take part in Republic Day celebrations. During his
visit, the countries signed several agreements. Alongside, the fifth session of India-Brazil Trade monitoring
mechanism was also held. President Bolsonaro was accompanied by a huge business delegation. They will
hold several meetings with their Indian counterparts during the India-Brazil Business Forum that is to be
held in 27, January, 2020.
Highlights
The countries signed agreements on trade, cyber security, information technology. The countries have set
target to increase bilateral trade to 15 billion USD by 2022.
Outcomes
Animal Husbandry is the new area over which the countries have identified potential cooperation. This is
mainly due to the common genetic heritage between the countries.
Concerns
The countries agreed to address the issue of sugarcane bilaterally. The issue was raised by Brazil at WTO
complaining India is extending support to local sugarcane growers. According to WTO, Most Favoured
Nation status should be awarded by the members to every other member. This will exclude special tariff and
also exclude restriction on imports and exports helping to increase trade globally.
Padma Awards announced on Republic Day
On January 26, 2020, Government of India announced 141 Padma Awards on the occasion of 71st Republic
Day celebrations.
Highlights
Former Union Ministers George Fernandes, Arun Jaitley and Sushma Swaraj are to be awarded with Padma
© 2020 GKToday | All Rights Reserved | https://www.gktoday.in 49
Current Affairs [PDF] - January 16-31, 2020

Vibhushan posthumously. Former PM of Mauritius Anerood Jugnauth, Boxer MC Mary Kom, Hindustani
classical vocalist Chhannulal Mishra are also to be awarded with the prestigious award.
The former CM of Goa Manohar Parrikar is to be conferred with the award posthumously.
Two Bangladeshis are to awarded honoured with Padma awards as well. It includes High Commissioner of
Bangladesh to India Mr Syed Muazzem Ali who passed away in 2014. He rebelled against the Pakistani
Government and declared his allegiance to Bangladesh in 1971. He was serving in the Pakistan embassy
during his allegiance.
Padma Awards
The awards are one of the highest civilian honours of India. The awards are presented in three categories
namely Padma Vibhushan, Padma Shri and Padma Bhushan. The awards are conferred based on the
recommendations made by the Padma Awards Committee. The committee is constituted by the Prime
Minister every year. However, the nomination process is open to the public as well. Even self-nominations
are allowed.
Padma Vibhushan is awarded for exceptional and distinguished services. Padma Shri is awarded for
distinguished service and Padma Bhushan is awarded for distinguished service of high order.
DRDO displays A-SAT weapon system at Republic Day Parade
On January 26, 2020, the Defence Research Development Organization (DRDO) displayed the A-SAT weapon
system. A-SAT is the Anti-Satellite Weapon System. India is one of the few countries in the world to own the
technology. The other countries include US, China and Russia.
Highlights
DRDO launched “Mission Shakti”, the first A-SAT mission. The mission demonstrated anti-satellite
technology. The mission destroyed a live orbiting satellite at the Low Earth Orbit. The mission used a
missile at a speed of 11 km per second to destroy the satellite.
The technology used in the A-SAT mission was named by India as “hit to kill” technology. The covert
technology was developed by India. It enables to destroy enemy satellite with 10-centi metre accuracy.
International Treaty
India is a signatory of the international treaty on space, the 1967 Outer Space Treaty. According to the
treaty, mass destruction of weapons in outer space are alone prohibited.
Shiv Bhojan Tali Scheme launched in Maharashtra
st
At the 71 celebrations of Republic Day, Maharashtra Govenrment launched the Shiv Bhojan Scheme. The
scheme will offer meals at Rs 10 to the poor.
Highlights
On the eve of Republic Day, at least one Shiv Bhojan canteen was started at the district head quarters all
over the state. The scheme aims at providing meals to the poor at designated centres during stipulated
time.
The scheme aims at providing affordable, quality food to all. The State Government has estimated that the
scheme would cost Rs 6.4 crores and is expected to run for three months.
About the Centre
Also, the centres of the scheme are to opened at places where the below poverty line citizens live. Also, it is
to be opened in areas like district hospitals, markets, railway stations. Initially the Government has planned
to set up 50 such centres. The numbers are to be increased based on the feedback.
Kobe Bryant, a retired NBA star dies in Helicopter Crash
The NBA (National Basketball Association) star Kobe Bryant died in a helicopter crash. He retired from the
game in 2016. He has won two gold medals in Olympics, one in 2008 and the other in 2012. The star had
won 5 championships so far.

© 2020 GKToday | All Rights Reserved | https://www.gktoday.in 50


Current Affairs [PDF] - January 16-31, 2020

Kobe Bryant
Bryant was the first guard in the American history to play 20 seasons. He was also a five-time champion. He
won the championship in 2000, 2001, 2002, 2009 and 2010.
National BasketBall Association
The NBA is a basketball league located in North Amercia. It is composed of 30 teams. It was founded in
New York City in 1946. NBA is one of the four major sports league USA. The other 3 major sports league
include Major League Baseball, National Football League and National Hockey League.
All the four leagues are commonly referred as “Big Four”.
NBA in India
NBA launched the first basket ball school in 2017. The school was launched at Mumbai. The school trains
both male and female basket ball players between the age of 6 and 18.
Global Potato Conclave to be held in Gujarat
The Global Potato Conclave is to be held between January 28 and January 31, 2020. PM Modi is to attend
the conclave. PM Modi is to address the conclave.
Highlights
The conclave will hold live demonstrations of potato varieties, farm equipment, management practices. The
conclave will include researchers from more than 30 countries and is to be conducted under 10 themes. It
will cover next generation breeding practices, efficient management practices of potato cultivation,
management strategies of potato diseases and pests. The themes of the conclave are as follows
Potato Biotechnology and Omics
Germplasm Management and Next Generation Breeding
AI and ICT in Potato R&D.
Potato Pest Management
Potato Disease Management
Technology transfer, social issues and policies
Potato Value Chain management
Advances in Seed Technology
Climate change and crop management
Post-harvest management and value addition.
Significance
After rice and wheat, potato is the third most important food crop in the world. The growth of potato has
increased by 40% between 1961 and 2016. However, the area under potato production has decreased from
22.14 million hectares to 19.24 million hectares.
Andhra Pradesh Cabinet nods to abolish Legislative Council
On January 27, 2020, the Andhra Pradesh cabinet cleared to abolish Legislative Council. It is to be noted
that the Legislative Council stalled two bills that were passed by the assembly. It includes AP Capital
Region Development Authority Repeal Bill and Decentralization bill.
State Legislative Council
Andhra Pradesh is one of the few states to have bicameral legislature. The states with legislative council
includes Bihar, Maharashtra, Karnataka, Uttar Pradesh and Telangana.
Constitutional Provisions
Article 169 of the constitution defines Legislative Council. As of November 2019, there are 6 states with
State Legislative Council. The members of the State Legislative Council are selected in the following way
One-third are elected from the local bodies such as Gram Panchayat, municipalities and district
councils.

© 2020 GKToday | All Rights Reserved | https://www.gktoday.in 51


Current Affairs [PDF] - January 16-31, 2020

One-third are elected by members of State Legislative Assembly.


One-Sixth of the members are nominated by the governor
One-twelfth are graduates who have resided for three years in the state
One-twelfth are teaching professions.
GoI signs “Historic Accord”, Bodo Agreement
On January 27, 2020, the Government of India signed the historic agreement with the nine factions of the
National Democratic Front of Bodoland (NDFB). According to the agreement, the Bodoland Territorial Area
District will hold special rights for the locals. However, the outsiders will have to obtain “permit” to work in
the territory.
Highlights
The agreement will provide political rights to the Bodo Tribals living in Assam. The agreement was signed in
the presence of Home Minister Amit Shah and Assam Chief Minister Sarbanand Sonowal and the leaders of
the NDFB.
The Agreement will also provide economic benefits to the Bodos.
Bodo Accord- Bodoland Territorial Area District
The accord signed is the third agreement signed in 27 years. The first was signed with All Bodo Students
Union in 1993 and the second was signed with the Bodo Liberation Tigers in 2003. The third agreement led
to the formation of Bodoland Territorial Council under the sixth Schedule of the Constitution. The BTC now
administrates 4 districts in Assam namely Chirang, Kokrajhar, Baska and Udalguri. These areas are called
the Bodoland Territorial Area Districts.
January 27: International Holocaust Remembrance Day
The United Nations Department of Global Communications marked the International Day of Commemoration
in Memory of Victims of Holocaust on January 27. This year, 2020 marks 75th anniversary of the day.
The Day was officially announced in November 2005 at the United Nations General Assembly
Highlights
The International Day marks the ending of world war II. It also marks the establishment of United Nations
formed in response to the crimes of Holocaust and Second World War. This year, the theme of the day is as
follows
Theme: 75 Years after Auschwitz-Holocaust Education and Remembrance for Global Justice
The day also marks the anniversary of Liberation of Auschwitz. Auschwitz was a complex that operated
more than 40 exterminations camps and concentration camps during World War II.
The Holocaust
The holocaust was the genocide of the European Jews between 1941 and 1945. Two-thirds of European
population were Jews. Around two-thirds of the European population were Jewish and Nazis under the
leadership of Hitler murdered around 6 million.
GoI new Target: Indian Railways to run 100% on electricity by 2024
On January 27, 2020, the Union Minister Piyush Goyal announced that Indian Railways will run 100% on
electricity by 2024. In such case, Indian Railways will be the first Railways to run 100% on electricity. Also,
the Indian Railways has planned to make the network zero-emission network by 2030.
Electrification Project
Previously in 2019, the target to run 100% electrified Railways was set to be achieved by 2023. So far, till
2017-18, 4,087 km route had already been electrified. Upon success, the fuel bill will reduce by Rs 13,510
crores.
Current Scenario
Today Indian trains run on diesel or electricity. Around two-third of the freight movement in India is through

© 2020 GKToday | All Rights Reserved | https://www.gktoday.in 52


Current Affairs [PDF] - January 16-31, 2020

Indian Railways. At present, Electric traction accounts to 57.91% of energy expenses of Indian Railways (as
per November 2019)
Electrification of Railways is also Government’s plan to create 500 GW of renewable energy by 2028.
Need
According to NITI Aayog, the carbon emission from the Indian Railways is around 6.84 million tonnes
(2014).
January 28, 2020
Corona virus: WHO increases global risk from moderate to high
On 27 January, 2020, the World Health Organization arrived in China to discuss the ways the Corona Virus is
spreading. The death toll due to the virus has reached 106 and around 4,000 people have been infected so
far.
Highlights
The WHO had earlier reported that the spread of the virus is moderate. However, it has now changed its
status of risk of the virus as “high”.
Spread of the Virus
The virus has spread to France, China, Canada, US, Japan, Thailand, Sri Lanka. In India, an Indian medical
student (Rajasthan) who recently travelled from China has been quarantined.
Effects of the virus
Several countries like US, Canada, India, France have warned against travel to China. The Global stocks have
fell down and Chinese Yuan has dipped to its weakest level in 2020. The oil prices have hit a three-month
low as China is making moves to keep its citizens at home. China has extended its Lunar New Year holiday
to restrict people mobility in the country.
Chinese New Year
The Chinese follow a lunisolar calendar. The first day of the Chinese New Year always begins as a New
Moon Day. The Lunisolar calendar divides a year into 24 solar terms.
India’s first underwater Metro: To begin in March 2022 in Kolkata
The first underwater Metro is being constructed in Hooghly river. The project is awaiting clearance of 2.8
million USD from the Indian Railways.
Highlights
The Hooghly Metro is one of the oldest Metro in the country. It was started in 1984. The new under water
tunnel will carry 20% of city’s population and will cross the river in less than a minute. Currently it takes 20
minutes in the ferry to cross the river and an hour to cross the Howrah bridge.
Infrastructure and Environment
While the infrastructural developments are increasing exponentially, it is important to check their effects on
environment. According to World Wide Funds, by 2050 the Earth will hold 75% more infrastructure than that
it does today. Therefore, it is essential to study their Environmental impacts and take necessary alternate
actions to protect the environment.
Environmental Impact Assessment (EIA)
According to United Nations Environmental Programme (UNEP), EIA is the evaluation of environmental
impacts and also cultural, human-health and socio-economic impacts of a proposed project.
EIA in India
EIA in India began in 1976. However, it was an administrative decision till 1994. In 1994 Environmental
(Protection) act, 1986 was amended to provide legislative support to EIA process
Bengal becomes the fourth state to pass resolution against CAA
The Citizenship (Amendment) Act was legislated by the Parliament during this winter session. However,
© 2020 GKToday | All Rights Reserved | https://www.gktoday.in 53
Current Affairs [PDF] - January 16-31, 2020

several states are agitating over the non-inclusion of Muslims in the law. Kerala was the first state to pass a
resolution against the act. Recently, the state of West Bengal has passed similar resolution.
Highlights
After Kerala, Punjab and Rajasthan, West Bengal passed a resolution against CAA. According to West
Bengal, the act does not stick to secularism. Also, the state claims that NPR, CAA and NRC are co-related.
NPR
NPR is National Population Register that holds the names of the residents of India. The register is prepared
and updated based on the Citizenship Act 1955 and Citizenship (Registration of Citizens and Issue of
National Identity Card) rules, 2003. The NPR was updated in 2010 and the details were collected along with
the Census of India, 2011.
The register holds biometric and demographic details of the residents.
Legislation
Section 14A of the Citizenship Act says that the Central Government has to register every citizen
compulsorily and issue an identity card to him. Also, the Government shall maintain a register of Indian
Citizens. In order to achieve this, it shall establish a National Registration Authority.
US: 25,000 Child porn cases in 5 months in India; Rajya Sabha panel report on child porn
On January 28, 2020, a non-profit organization in the US called the National Centre for Missing and
Exploited Children (NCMEC) said that over 25,000 cases of child pornography have been uploaded in 5
months in India. The data has been shared by the organization with the National Crime Records Bureau.
Highlights
The Home Ministry has signed an agreement with the NCMEC in 2019 to obtain data on Child pornography
being uploaded in India. According to Ministry of Home Affairs, Delhi tops the list when it comes to Child
sexual Abuse material followed by Maharashtra, Gujarat, Uttar Pradesh and West Bengal.
POCSO Act
In June 2019, the Indian Government widened the definition of Child pornography under POCSO (Prevention
of Children from Sexual Offences) act. It now covers visual depiction, video, photograph, digital or
computer-generated image that appears to depict a child.
Rajya Sabha Panel
An Ad-hoc committee was constituted by the Chairman of Rajya Sabha Shri M Venkaiah Naidu. The
committee has submitted its report on January 26, 2020. The committee has made 40 recommendations to
prevent sexual abuse of children.
The report calls to remove such content also enable parents for early detection of access to such
content by children.
The committee has also recommended to amend POCSO act and IT act, 2000.
January 28: Lala Lajpat Rai Birth Anniversary Celebrated
On January 28, 2020, the India celebrated the legend Lala Lajpat Rai’s 155th birth anniversary. He is one of
the most celebrated freedom fighters in the country. His patriotism earned him titles such as “Punjab
Kesari” and “Lion of Punjab”.
Lala Lajpat Rai
He was born on 28 January, 1865 in Punjab. His father was a Government School teacher.
Lal Bal Pal
He was one of the triumvirates of the Lal Bal Pal. It includes Lala Lajpat Rai of Punjab, Bal Gangadhar Tilak
of Maharashtra and Bipin Chandra Pal of Bengal. The three leaders changed the political discoursed of the
Indian Movement. They mobilized Indians across the countries and strengthened Swadeshi Movement. They
also were active participants in involving Indians against Bengal Partition.

© 2020 GKToday | All Rights Reserved | https://www.gktoday.in 54


Current Affairs [PDF] - January 16-31, 2020

Movements
He was an active participant of Hindu Reform Movements, Indian Independence Movements, Indian
Nationalist Movements. Young men Bhagat Singh and Chandrasekhar Azad were greatly influenced by Rai.
He was also a major participant of the protests against Simon commission.
Institutions
He founded several institutions and organizations such as Hisar Bar Council, Hisar Arya Samaj, Hisar
Congress, National DAV Managing Committee.
Resolution against CAA in European Parliament; India’s Measures
On January 29, 2020, the European Parliament is set to discuss the Citizenship (Amendment) Act (CAA).
Highlights
Though the resolution made by the parliament do not affect the decision of the European Council or
commission, it might impinge the bilateral relations between India and European Union. PM Modi is to visit
Brussels in March 2020 to attend India-EU summit and the External Affairs Minister is scheduled to visit in
mid-February.
Six Resolutions
Around six resolutions are to be taken up for discussion in the EU Parliament. It includes
Implementation of NRC in Assam and the security lock down in Jammu and Kashmir
Resolution moved by the S&D Group is one of largest resolution with 154 members supporting it.
According to the resolution, the CAA has the potential to create largest statelessness crisis in the
world. S&D group is the Socialist and Democratic Group
The resolution by EPP Group with 182 members supporting it says that the law could have negative
international image for India. The EPP Group is the European People’s Party Group.
India’s Measures
The Lok Sabha Speaker has written to the European Parliament requesting the EU parliament to respect
sovereign processes of fellow legislatures.
Also, India believes that CAA is an internal matter and is not yet settled within India. There are more than 20
pending cases on CAA in the Supreme Court.
However, there has not been any official response towards EU’s move from the Ministry of External Affairs.
Supreme Court on CAA
The Supreme Court on January 22, 2020 pronounced its decision on CAA without hearing the case. The
apex court has given 4 weeks for the centre to respond to the petitions that challenged CAA.
India-EU relations
The European Union is India’s largest trading partner. The European Union account to 92 billion Euros
(2018-19) of trade. It accounts to 18% of Indian Exports
Britain releases new 50 pence coin marking BREXIT
The British Government has unveiled a new 50 pence coin commemorating Brexit. The coin has the
inscription “Peace, Prosperity and Friendship with all nations”. Around 3 million coins are to be distributed
from post offices, banks and shops from January 31, 2020.
India and Brexit
More than 800 Indian companies have invested in UK. Some of them have made their investments aiming at
the wider European market. UK is the third largest FDI contributor in India. Also, India is the largest G20
investor of the United Kingdom. The key sectors attracting investment in the UK are agritech, health care,
drinks and food.
On the other hand, India has to adjust to the world order that is changing due to Brexit. For a short term, it is
estimated that Brexit will have a negative impact on 108 billion USD business of India.

© 2020 GKToday | All Rights Reserved | https://www.gktoday.in 55


Current Affairs [PDF] - January 16-31, 2020

Britain and Brexit


According to the International Monetary Fund, Brexit will reduce the size of the Britain Economy by 5%. The
impact will reduce its economic activities by 3.8% to 7.5% by 2030.
Passport Renewal Reminder Services begun by Passport Seva Kendras
On January 28, 2020, the Ministry of External Affairs launched the Passport renewal reminder services. The
Passport Seva Kendras operating under the Ministry of External Affairs will send SMS about the renewal
Highlights
The ministry is to send two Reminder messages, one before 9 months and another before 7 months. Along
with the reminder message, the passport holders will also be informed about the authenticate website of
the Ministry which is www.passportindia.gov.in.
Indian Passport
The Indian Passport is issued by the Ministry of External Affairs. The passport serves as a proof of Indian
Citizenship under Passports Act, 1967. Around 65 million Indians hold passports as of the end of 2015. In
2015, India issued around 12 million new passports. This number has so far been exceeded only by US and
China.
Types of Passports
There are three types of passports as follows
Ordinary passport-It is dark blue in colour. It is issued to citizens travelling for vacation, business
trips and study.
Official Passport-It is white in colour. It is issued to individuals representing Government of India
Diplomatic Passport-It is maroon in colour. IT is issued only to top government officials and
diplomatic couriers
Recently, India has ranked 84th in Henley’s Passport Index.
Report Card on Smart Cities to be released by June 2020
The Government of India will release a report card on 100 selected smart cities by June 2020. The cities are
to be assessed based on public performance index, ease of living and climate in June. The initiative is being
undertaken as the Smart City Project completes 5 years in June.
Highlights
The cities are also to be assessed based on education, economic ability, safety, housing, development,
mobility and health. Of these factors, 35% of weightage is to be given to quality of life, 20% to sustainability,
20% to economic viability and 30% to citizen participation.
Smart City Mission
Smart City Mission is being implemented by the Ministry of Urban Development. The mission aims at
developing 100 smart cities in the country. Till January 2018, 99 cities were selected to be upgraded under
the mission. Except West Bengal, all the states have selected at least one city under the mission. The
mission was launched by PM Modi in 2015.
MGNREGA scheme faces funds shortage
The MGNREGA (Mahatma Gandhi Rural Employment Guarantee Employment Act) scheme released its
financial statement recently. According to the statement, almost 96% of the money has already been spent.
Only 2,500 crores of rupees is left to sustain the scheme for the next two months.
Highlights
According to the financial statement of the scheme, 15 states are in red meaning, there is fund depletion or
fund unavailability. Among all the states, Rajasthan has the highest negative net balance of Rs 620 crores.
Following Rajasthan, Uttar Pradesh had a negative net balance of Rs 323 crores. The budget allocated to
the scheme was Rs 60,000 crores and is less as compared to the previous years.

© 2020 GKToday | All Rights Reserved | https://www.gktoday.in 56


Current Affairs [PDF] - January 16-31, 2020

Current Scenario
The pending dues in the state of Rajasthan is alone Rs 1,950 crores. Under the scheme, 75% of the wages
should be paid within 15 days of time period. However, there is delay in the payment currently due to lack of
funds. This will create lack of confidence among the citizens over the scheme.
January 29, 2020
Supreme Court: African Cheetah permitted to be introduced in India
On January 28, 2020, Supreme Court ruled that African Cheetah shall be permitted to be introduced in India.
The cat is being introduced on experimental basis.
Highlights
The Apex Court had put the move on hold 10 years back. It was proposed by the then Environment Minister
Jairam Ramesh. The proposal was to introduce foreign Cheetahs into the Palpur Kuno sanctuary in Madhya
Pradesh. However, the Apex Court stalled the proposal fearing the Cheetahs might become into conflict
with the project of reintroduction of lions in the sanctuary.
Supreme Court formed a three-member committee based on whose recommendation the judgement was
pronounced.
Need
Cheetah is the only mammal being hunted to extinction in India. The last Cheetah was sighted in 1952.
IUCN Red List
Cheetahs are classified as “Critically Endangered” under the IUCN Red List.
Why are the Cheetahs reintroduced in India?
With the Cheetahs extinct in India, the environmentalists suggest that it is essential to reintroduce them in
order to maintain balance in the ecosystem.
10 more Wet lands declared Ramsar Sites: Maharashtra gets its first Ramsar site
On January 28, 2020, the Union Ministry declared that Ramsar Convention had added 10 more sites from
India. Today, there are 37 Ramsar sites in the country.
Highlights
Among the 10 sites, Maharashtra (Nadur Madhameshwar) gets its first Ramsar site. The other wetlands
sites include one in Uttar Pradesh and 3 in Punjab (Beas Conservation Reserve, Keshoupur-Miani and
Nangal).
Wetland Restoration
In the past 6 months, the ministry of Environment prepared four pronged strategies for restoration of
wetlands. It includes wetland health cards, preparing a baseline data, preparing targeted Integrated
Management and enlisting wetland mitras.
Significance
It is important to conserve wetlands for water, food, groundwater recharge, fibre, water purification, erosion
control, flood moderation and climate regulation. Wetlands are the major source of fresh water. They help
to soak rain water and recharge ground water.
Conserving Wet Lands will also help in achieving Nal Se Jal scheme. The scheme aims to provide water
connection to every house hold by 2024.
Bhuvan Panchayat 3.0 launched by ISRO
On January 29, 2020, ISRO (Indian Space Research Organization) launched Bhuvan Panchayat version 3.0.
Highlights
Under the project, the ISRO will collaborate with Gram panchayat members to understand their data
requirements. The portal will function with the help of satellite technology of ISRO. The project will run for
two years. The portal aims at helping village development planning process under the Ministry of
© 2020 GKToday | All Rights Reserved | https://www.gktoday.in 57
Current Affairs [PDF] - January 16-31, 2020

Panchayati Raj.
What is Bhuvan Project?
Bhuvan is a satellite application that is powered by ISRO. It allows users to explore 2D and 3D
representation of the earth.
Other Initiatives of ISRO
Along with Department of Land Resources ISRO has developed Srishti to monitor Integrated Watershed
Management Programme. ISRO is also setting up Telangana Water Resources Information System (TWRIS)
on the Bhuvan Web Portal.
Other Bhuvan Initiatives
Bhuvan-GAIL monitors pipelines using space technology to address pipeline safety concerns. The
MGNREGA of every Gram Panchayat are to be geo-tagged with the help of BHUVAN portal.
Composite Regional Centre launched in Port Blair
The Ministry of Social Justice and Empowerment launched the Composite Regional Centre (CRC) for skill
development, empowerment of persons and rehabilitation in Port Blair. The Composite Regional Centres in
India are located in Kozhikode, Bhopal, Lucknow, Patna, Guwahati, Srinagar, Ahmedabad, Mandi and Suraghi
(Chhattisgarh).
Highlights
The CRCs are centres set under Ministry of Social Justice and Empowerment in order to create
infrastructure and resources for persons with disabilities. These centres focus at locals rather than urban
areas.
Beneficiaries
The GoI targets only a part of the population through these centres. It includes persons with low-vision,
mental retardation, blindness, locomotor disability, hearing impairment, mental illness, cerebral palsy,
leprosy, multiply persons and Autism Spectrum Disorder.
Aim
The Central Government aims to reach the unreached disabled population through these centres. The CRCs
help in fastening the process of establishing rehabilitation services. The centres aim at training of
rehabilitation professionals and capacity building at centre, district and state levels.
“Operation Vanilla” launched by Indian Navy for Disaster Relief at Madagascar
On January 29, 2020, the Indian Navy launched Operation Vanilla in the Southern Indian Ocean. The
operation was launched based on the request received from Madagascar.
Highlights
The Operation has been launched to assist the population of Madagascar that were affected by Cyclone
Diane. The Indian Navy ship Airavat has been deployed in the relief mission. The operation will provide
succor to the flood affected population. The ship will also set up a medical camp and offer water, food and
other essential relief materials.
Madagascar
Madagascar is an island country in the Indian Ocean. It is the fourth largest island in the world. India has
been trying to include Madagascar and Comoros in its Indian Ocean Vision for their strategic location. On
the other hand, Seychelles and Mauritius are part of Indian Ocean Region division.
All the four islands Seychelles, Mauritius, Madagascar and Comoros are members of African Union and
Indian Ocean Commission.
National Commission for Homeopathy bill, 2019 approved by Cabinet
On January 29, 2020, the Union Cabinet approved National Commission for homeopathy bill, 2019.

© 2020 GKToday | All Rights Reserved | https://www.gktoday.in 58


Current Affairs [PDF] - January 16-31, 2020

Highlights
The bill is being introduced for the following amendments
To introduce reforms in the field of Homeopathy education
To enable accountability and transparency in protecting the interests of general public.
Current Legislation
The Homeopathy Central Council act, 1973 is currently governing the Homeopathy education and practices
in the country. The act has been extracted based on the Indian Medical Council Act, 1956.
Homeopathy Central Council
The functions, regulation and powers of the council is very similar to that of the Indian Medical Council of
India. The amendments are being introduced as there has been serious detrimental effects in medical
education and delivery of Homeopathy health care services in the country.
The Council operates under Ministry of AYUSH. Also, it is one of the professional councils that have been
constituted under University Grants Commission (UGC). The professional councils include AICTE, Bar
Council of India, Medical Council of India, Veterinary council, nursing council, etc.
Cabinet: National Commission for Indian System of Medicine Bill, 2019 approved
The Union Cabinet on January 29, 2020 approved the official amendments to National Commission for
Indian System of Medicine bill, 2019. The bill is currently pending in Rajya Sabha
Highlights
The amendments will introduce necessary regulatory reforms in the Indian System of Medicine Education. It
will also enhance transparency and accountability towards the public. It will promote affordable health care
services in all parts of the country.
National Commission for Indian system of Medicine (NCIM)
The main objective of establishing NCIM is to ensure adequate supply of skilled medical professionals and
also increase ethical standards of medical standards of Indian System of Medicine.
NCIM bill, 2017
The bill facilitates maintenance of register of Indian systems of medicine. It also encourages interface
between different medicinal systems. The bill provides periodic assessments of medicinal institutions. An
advisory council for the system shall be constituted.
The bill also enables joint sitting of NCIM, council of Homeopathy and modern medicine at least once a
year.
The Medical Termination of Pregnancy (Amendment) Bill, 2020 approved by Cabinet
On January 29, 2020, the Union Cabinet approved Medical Termination of Pregnancy (Amendment) Bill,
2020. The bill aims to amend the Medical Termination of Pregnancy act, 1971. The bill is to be introduced in
the coming session of Parliament
Highlights
According to the proposal, termination of pregnancy is to be extended to 24 weeks. It was 20 weeks earlier.
The extension is only for special category of women. It includes rape victims, survivors of rape and other
vulnerable women like differently-abled women.
Significance
The bill is a step towards increasing access of women to safe abortion services. The months have been
increased considering the advancements in medical technology. The amendments were proposed by the
Ministry of Health and Family Welfare.
First Legalization of Abortion
In India the Shah Committee was appointed to review the socio-cultural and medical aspects of abortion. It
was this committee that recommended legalization of abortion for the first time.

© 2020 GKToday | All Rights Reserved | https://www.gktoday.in 59


Current Affairs [PDF] - January 16-31, 2020

Cure for Corona Virus: AYUSH recommends Unani Medicines


The World Health Organization recently increased the spreading risk of Corona Virus from medium to high.
With the virus spreading to different parts of the world like Japan, US, UK, Sri Lanka, Germany, India has
found its cure!
Highlights
A Siddha Doctor from Tamil Nadu has claimed to have formulated a herbal medicine to cure Corona Virus.
The Ministry of AYUSH has recommended Ayurvedic and Homeopathic preventions for the infections
caused by the new stain of Corona Virus. The Ministry has also provided a set of preventive management
steps. The steps are based on Unani, Ayurvedic and Homeopathic practices.
Genome Sequencing
The Chinese researchers have found out from the genome sequences of the new Virus that the virus
belongs to Corona Virus family. The sequencing also says that the virus has similarities with Severe Acute
Respiratory Syndrome (SARS) corona virus.
SARS
The SARS outbreak happened in 2002 and 2003. The virus took the lives of around 774 people all over the
world. SARS also began spreading from China. However, since 2004 no SARS cases have been found.
“Madhavpur Mela” of North East reviewed; Cabinet approves 30% NEC allocation to new projects
On January 29, 2020, the Union Minister of Development of NER (North Eastern Region), reviewed the
preparations of Madhavpur Mela in Gujarat. It is to be organized in Porbandar (the birth place of Gandhiji).
Also, the Cabinet has approved that 30% of North Eastern Council funds are to be allocated to the new
projects of the North East Region.
Highlights
All the eight north eastern states are to participate in the Mela. It includes Assam, Sikkim, Manipur,
Meghalaya, Mizoram, Tripura, Arunachal Pradesh and Nagaland. The Mela is to commence in April 2020.
Madhavpur lies on the sea shore and the site of marked by the Madhavrai Temple. The temple belongs to
15th century. The Hindu festival associated with the Mela is “Ram Navami”.
Significance
The Mela will integrate the cultures of North-East regions and Gujarat under Ek Bharat Shreshtha Bharat.
Developmental Aspects
Apart from cultural importance, the mela will also focus on the developmental aspects of the north eastern
states. The Mela will focus on setting up of Bamboo Industrial Park in Assam, establishment of North East
Convention Centre at Dwarka and introduction of sea plane.
Cabinet Approval
The cabinet has approved that 30% of the North East Council funds are to be allocated to new projects.
These projects have been given least importance so far. With increased fund allocation they will be able to
reach in pace with the older projects and help in faster development
January 30, 2020
NITI Aayog’s Aspirational District Ranking released: Chandauli of UP tops
On January 29, 2020, the NITI Aayog released the ranking of Aspirational Districts of the country for the
month of December 2019. The ranking is released every month.
Highlights
The Chandauli district of UP topped the list followed by Bolangir of Odisha, YSR (AP) in second and third
positions respectively. Sahibanj of Jhatkhand and Hailakandi of Assam were placed in fourth and fifth
places. The rankings were done for 112 aspirational districts. The districts were selected from six
developmental areas in December 2019.
© 2020 GKToday | All Rights Reserved | https://www.gktoday.in 60
Current Affairs [PDF] - January 16-31, 2020

Parameters
The areas evaluated during the ranking includes Health and Nutrition, agriculture, education, skill
development, financial inclusion, basic infrastructure and water resources.
Aspirational Districts Programme
The Programme was launched in January 2018. The main objective of the programme is to transform the
districts that have been pockets of under-development. These cities show relatively lesser progress. Also,
the ranking helps in competitive Federalism and hence hastens overall development.
The Programme aims at Convergence, Collaboration and Competition.
World Sustainable Development Summit organized by TERI first time in New Delhi
The World Sustainable Summit is an annual event organized by The Energy and Resources Institute (TERI).
It is being organized since 2001. The event has become a premier international event focusing on
developing actions.
Highlights
The event is to be held between January 29, 2020 and January 31, 2020. This year, the event is being held at
New Delhi. The theme of the event is
Theme: ‘Towards 2030: Making the Decade Count’
The Summit brings together Nobel Laureates, decision-makers and political leaders from multilateral
institutions. After signing the Paris Agreement in 2015, the summit aims to provide long term solutions to
issues related to future of humanity.
About the 2020 Summit
The Summit includes Sustainable Action Dialogues, Regional dialogues, corporate conclaves, IFAT Delhi
2020 (Exhibition of Clean Technologies), Youth Volunteer Programmes
Sustainable Development
The concept of Sustainable Development has its roots in sustainable forest management during 17th and
18th century. The Global Movement for Sustainable Development began in 2005. It launched the UN decade
of Education for Sustainable Development. This was the turning point for the world to begins its journey
towards sustainable development.
War Memorial built by Assam Rifles in Nagaland
The Assam Rifles has constructed war memorial for 357 personnel who sacrificed their lives fighting
against insurgency in the North Eastern State. The Memorial is first of its kind in the state. It has been
constructed at Mokokchung in Assam.
Highlights
The War Memorial has been designed in concentric circular form. It has three converging posts signifying
all the three forces Army, Assam Rifles and Air Force.
Assam Rifles
It is the oldest paramilitary force in india. It was called the “Cachar Levy”. The four forces namely Assam
Frontier Police, Eastern Bengal Military Force, Assam Military Police and the Assam Military Force were
joined to form the Assam Rifles in 1917. The predecessor forces of Assam Rifles have served in World War
I, Middle East and Europe. Assam Rifles participated in World War II.
Assam Rifles operate under Home Ministry. Since 2000, they are guarding the Indo-Myanmar Barrier under
“one border one force” policy of Government of India.
Veer Samriti
Veer Samriti is the second world war cemetery located in Kohima.
Surajkund Mela-International Crafts Fair organized by Haryana with HP as theme state
On February 1, 2020, President Ram Nath Kovind will inaugurate the 34th Surajkund International Crafts Fair.

© 2020 GKToday | All Rights Reserved | https://www.gktoday.in 61


Current Affairs [PDF] - January 16-31, 2020

The fair is being organized by the Haryana Government in Faridabad. Around 20 countries and the states of
India are participating in the fair.
Highlights
The partner country of the fair is Uzbekistan. The theme state of the fair is Himachal Pradesh. This year the
state of Himachal Pradesh will showcase its handicrafts, heritage and tourism potential. The Tourism
Department has constructed replica of Ram Bag Gate and Bhimakali temple. The state will also perform its
traditional dances and music.
The event will also host fashion shows and install 70 shops for Handloom, Bamboo and metal crafts.
Significance
The fair will help to promote tourism. It will also aid the state of Himachal Pradesh to display its huge
adventure potential focusing mainly on water sports. The fair was upgraded to international level in 2013
and since 2015, around 20 countries are being invited.
UN Development Programme: Telangana tops sustainable growth goals performer list
According to the United National Development Programme (UNDP), Telangana has emerged the best
performing states in terms of achieving Sustainable Development Goals (SDG).
It is to be noted that in December 2019, India released SDG India Index. India was the first country to
release SDG index. The index was launched by NITI Aayog.
Highlights
Telangana topped the UNDP ranking of Indian states based on their progress in achieving SDGs. The state
of Telangana scored 82. It was followed by states of Andhra Pradesh and Karnataka together with a score
of 72. However, Kerala followed by Himachal Pradesh lead the list.
The states have been scored based on Sustainable Development Index.
Telangana
The state has improved in 8 out of 17 SDGs. It has moved from score of 75 to 82. It out performed in clean
water, energy, sanitation. The state was also ranked number one in terms of Reduced Inequality. It was
ranked third in Affordable and Clean Energy and fourth in climate action.
Performance of Telangana
In terms of economic growth, the state has jumped from 75% to 82%. In terms of clean water and sanitation,
the state has improved from 55% to 84%. In terms of sustainable cities and communities the state has
improved from 44% to 62%.
January 30: World Neglected Tropical Diseases Day
Every year January 30 is to be marked as World Neglected Tropical Diseases day. The day creates
awareness in addressing the tropical diseases. This is the first time World NTD Day that is being marked.
Neglected Tropical Diseases
The Neglected Tropical Diseases are caused by pathogens such as bacteria, helminths, viruses and
protozoa. They are common in low-income countries of Asia, Africa and America.
The diseases affect more than 149 countries.
According to the WHO, 20 health conditions are categorised as Neglected tropical diseases. They are as
follows
Bacterial and Fungal NTDs include Leprosy, Bruli Ulcer, Trachoma, Yaws, Mycetoma,
Chromoblastomycoses
Arbovirus infections such as Rabies, Dengue, Chikungunya, scabies
Chagas disease, Leishmaniasis and African Trypanosomiasis caused by Protozoans
Dracunculiasis, Taeniasis, Schistosomiasis, Onchocerciasis, Echinococcosis caused by Helminth

© 2020 GKToday | All Rights Reserved | https://www.gktoday.in 62


Current Affairs [PDF] - January 16-31, 2020

Significance
WHO estimates that more than a billion population spread across 149 countries are affected from
Neglected Tropical Diseases. The common NTD of India is Lymphatic filariasis.
Transparency International: Assam tops in budget formulation
According to the survey conducted by Transparency International, Assam tops in terms of budget
formulation followed by Odisha and Andhra Pradesh. The states that were ranked lower in terms of budget
formulation are Goa, Maharashtra and Punjab.
Highlights
The survey was conducted by the organization based on four parameters such as budgetary process, public
disclosure, post budget fiscal management and efforts to make budget citizen friendly.
Transparency International
The international Non-Governmental Organization measures and prevents criminal activities arising due to
corruption. It releases Global Corruption Index annually. The organization is also a member of UNESCO,
United National Global Compact, UNESCO Consultative Status, United Nations Sustainable Development
Group.
The Headquarters of Transparency International is located in Germany.
Other Key products of Transparency International
The Organization also provides Global Corruption Barometer that asks citizens about their personal
experiences of corruption in their daily lives. Apart form this the organization also publishes Government
Anti-Corruption Index.
SAMPRITI-IX: India-Bangladesh Joint Military Exercise to be held in Meghalaya
A Joint Military Exercise between India and Bangladesh is to be held at Umroi, Meghalaya between 3rd
February, 2020 and 16th February, 2020. The exercise is called “SAMPRITI”
Highlights
During the joint exercise, Command Post Exercise and Field Training Exercise is to be conducted according
to United Nations Charter. The troops of both the armies will also practice Counter Terrorist operation.
India-Bangladesh Defence Cooperation
In 2017, India and Bangladesh signed defence agreements in order to conduct military exercises. India
since then has been assisting Bangladesh in training their officers.
In October 2019, the countries signed MoU on coastal surveillance system. The MoU also included a
Standard Operating Procedure for operating the ports of Chattogram and Mongola. Apart from this, it
allowed India to use 1.82 cusec of water from Feni river of Bangladesh.
Other Exercises
The first ever India-Bangladesh Naval exercise was conducted in Vishakhaptnam in October 2019. The
Bangladesh Naval Ships also participated in CORPAT before arriving to Vizag for the joint Naval Exercise.
CORPAT is Coordinated Patrol. This will help both the countries to replenish security in the Bay of Bengal
Region and also in the Indian Ocean Region.
Indian Railways to roll out Automatic Identification and Data Collection
The Indian Railways is to implement plans for RFID (Radio Frequency IDentification). The railways is to tag
around 3,50,000 rolling stocks by 2021.
Highlights
Under the programme, around 3,500 coaches have been fitted with RFID so far. This will help the Indian
Railways in early identification of rolling stock. A rolling stock includes carriages, locomotives, wagons and
other vehicles used in Railways.
In order to ID the rolling stocks and collect their tracking data, the Indian Railways has joined hands with

© 2020 GKToday | All Rights Reserved | https://www.gktoday.in 63


Current Affairs [PDF] - January 16-31, 2020

GS1 Indian. The tie up has been made under “Make in India” initiative.
RFID
RFID is a technology that exchanges data trough Radio Frequency. The tag is used to track individual
objects. The RFID tags are designed by Centre for Railway Information Systems.
CRIS
The CRIS develops and maintains information systems of Indian Railways. CRIS is located in Chanakyapuri,
New Delhi. It was established in 1986.
Need
Indian Railways is the largest railway system in the world. It carries around 3 million tonnes of freight and
23 million passengers in over 65,000 kilometres route. Therefore, it is essential to trace rolling stocks and
track them accurately.
Indian National Commission for Cooperation with UNESCO held in New Delhi
On January 30, 2020, the Indian Commission for Cooperation with UNESCO (INCCU) was chaired by the
Union Human Resource Development Minister Shri Ramesh Pokhriyal “Nishank”. UNESCO is United Nations
Educational, Scientific, and Cultural Organization. Along with the UNESCO commission, the event was also
attended by the sub commissions namely Social Sciences, Culture, Education, Natural Sciences and Culture.
Highlights
After the reconstitution of the commission, it has met for the first time. The sub commissions will aid in
achieving the objectives of UNESCO in India. This will help in strengthening India’s image globally. The
meeting also stressed on achieving 2030 Sustainable Development Goals.
One of the SDG, Quality Education is to be achieved through New Education Policy.
India and UNESCO
India is a founding member of UNESCO. The UNESCO office in India is a cluster office for 11 countries. They
are Bhutan, Nepal, Bangladesh, Pakistan, Sri Lanka, Maldives, Afghanistan, Iran, Myanmar, India and
Mongolia.
January 31, 2020
WHO: International Health Emergency declared over Corona Virus
On January 30, 2020, the World Health Organization declared International Health Emergency over Corona
Virus.
Highlights
The organization has declared emergency as the disease has spread to 18 other countries apart from China
where it originated. Also, the disease is spreading through human-to-human contact.
According to WHO, there are more than 7,711 confirmed cases and 12,167 suspected cases all over the
world. The spread of the disease now fulfils the criteria of PHEIC and hence emergency has been declared..
What is PHEIC?
PHEIC is Public Health Emergencies of International Concern. According to IHR (International Health
Regulations, 2005) of WHO, PHEIC is defined as an extraordinary event that fulfils the following
The disease constituted public health risk spreading to other states internationally
It has the potential to require international action
Upon declaration of emergency, WHO forms an Emergency Committee to provide advise on the necessary
actions.
India’s scenario
India so far has reported its first case of novel corona virus in Kerala. India has also requested permission
from China to bring back Indian nationals through 2 flights.

© 2020 GKToday | All Rights Reserved | https://www.gktoday.in 64


Current Affairs [PDF] - January 16-31, 2020

World Games Athlete of the Year, 2019-Title won by Rani Rampal


The Indian Women Hockey Team player Rani Rampal has won the “World Game Athlete of the Year, 2019”
title. She is the first hockey player in the world to receive the title.
Athlete of the Year
The title is awarded to the winner selected through a voting process. The award is presented to an awardee
who is outstanding in terms of social concern, performance and good behavior.
About Rani Rampal
Rani Rampal is called the “Queen of Indian Hockey”. She joined the Indian Hockey National Team at the age
of 15. She is the current captain of the team. In 2020, Rani Rampal was awarded with Padmashri, one of
India’s most prestigious awards.
Rani had participated in 2009 Asia cup and won silver medal. Also she was a part of 2010 Common Wealth
games of 2010.
Other nominated players
Around 20 players all over the world were nominated for the voting process. Among them, Stanislav Horuna,
a Ukranian Karate player secured second rank followed by the Canadian Athlete Rhea Stinn who finished
third power lifting.
CII celebrates 125 years; Tamil Nadu CM launches new scheme at the event
The Confederation of Indian Industry (CII) is celebrating its 125th birth anniversary in the year 2020. In order
to commemorate its anniversary, the organization has been hosting several events in different parts of India
under the theme
Theme: CII@125: Business and Beyond
History
The CII is India’s premier business association that works to sustain an environment that is conducive
towards the economic growth of the country.
The Journey of CII began as Engineering and Iron Trades Association (EITA) in 1895. Later the Engineering
Association of India that was established in 1942 merged with EITA to form the Association of Indian
engineering Industry (AIEI) in 1974. In 1986, it became the Confederation of Engineering Industry (CEI).
Finally, in 1991, the CEI became the CII.
Tamil Nadu Scheme
The Tamil Nadu Chief Minister on 30th January 2020, announced “Direct CTO (Consent To Operate) Scheme
for Green Industries” to mark the 125th birth anniversary of CII. The scheme also helps to improve Ease of
Doing Business.
The scheme aims at relaxing the clearances of Green category industries.
Pillars of CII
The CII works on the four pillars as follows
Building Competitiveness
Policy Advocacy
Creating an equitable society
Expanding industry’s global footprint
Google: Tangi Application; AI Model for Killer Whales; 1 million USD Grant
On January 31, 2020, Google announced I million USD grant to promote news literacy among Indians. Also,
the organization has launched a new application called “Tangi”.
Highlights: Grant
The Google News Initiative has announced to contribute 1 million USD grant to improve news literacy of
Indians. The grant is to be used to build a strong network against fake news producers in the country.

© 2020 GKToday | All Rights Reserved | https://www.gktoday.in 65


Current Affairs [PDF] - January 16-31, 2020

Around 300 workshops, boot camps and sessions are to be organized to train journalists, academicians and
fast-checkers on the authentication of the published content on the internet. The training is to be provided
in 7 Indian languages
Highlights: Tangi Application
The Tangi Application of Google focuses on Do It Yourself videos. The name of the application has been
inspired from TeAch aNd GIve.
Highlights: AI Model
Google has used an AI model that listens to Killer Whales, identifies the origin of sound and displays its
location. The model has been installed in Canada. According to Centre for Whale Research, only 73
Southern Resident Orcas (a sub species of Killer Whales) are left in the world
TERI: India’s Steel Demand to increase carbon emissions by 600 MT
On January 31, 2020, at the World Sustainable Development Summit released its report on India’s Steel
Demand. According to the report, if the steel demand of the country continues to grow, the carbon-dioxide
emissions from the steel sector will reach 600 million tonnes by 2050.
Highlights
The TERI released its report titled, “Towards a Low Carbon Steel Sector: Overview of the Changing Market,
Technology and Policy Context for Indian Steel”. According to the report, the current carbon-dioxide
emissions from steel sector is 242 Million Tonnes. This is 35% of total carbon dioxide emissions of the
country. It is to quadruple in three to four decades.
Suggestions made by the report
In order to manage the increase in steel demand emissions, India should start increasing the use of
domestic scrap and deploy energy efficiency measures. The report also recommends establishing low
carbon technologies. It also expects that the decarbonization technologies that are in the demonstration
stage will reach commercial markers in next 20 years.
TERI also suggests penalty for emissions starting from 2030. This will force industries to adopt deeper
decarbonization technologies.
Economic Survey 2020 tabled in the Parliament by the Finance Minister
The Economic Survey of the Fiscal Year 2020 has been tabled in both the houses of Parliament. The survey
highlights economic trends in the country, development over the past 12 months and facilitates better
mobilization of resources.
Highlights
The Current Fiscal Growth is 5%
The projected GDP growth for the year 2020-21 is 6% to 6.5%
The industrial growth for the Fiscal Year 2020 (2019-20) has been 2.5%
The Forex Reserves of the Fiscal Year 2019 (2018-19) was 461.2 billion USD
The GST collection for the FY20 has grown by 4.1%
Theme and its significance
The theme for the Economic Survey 2020 is
Theme: Wealth Creation, Promotion of pro-business policies, strengthening of trust in the economy
Significance
The investments grew between 2008 and 2013. After 2013, the credit (money inflow towards the economy
or investments) has been slowing down. The year 2013 saw the highest investments (between 2008-19)
mainly due to corporate sector. The credit has been declining mainly due to global scenario of economic
slowdown. This decline of investments is the major reason for the economic slow down after 2017.
Therefore, it is essential to CREATE WEALTH. In other words, to increase investments. This will aid in long

© 2020 GKToday | All Rights Reserved | https://www.gktoday.in 66


Current Affairs [PDF] - January 16-31, 2020

term economic growth according to the survey.


Infrastructure
The Survey says that in order to achieve 5 trillion USD by 2024-25, India has to spend 1.4 trillion USD. This
has to be done to avoid the constraint of lack of infrastructural growth in the path of India’s target.
Livestock
According to the survey, Livestock has become a major secondary source of income for rural families. This
has gained the sector a major role in achieving India’s goal of doubling farmer’s income by 2022. The
growth of Livestock has been 7.9% in the last five years.
Green India
The Survey also says that the forest and tree cover has reached 80.73 million hectares. This is 24.56% of
total geographical area of the country. The states that have gained forest cover are Karnataka, Andhra
Pradesh, Jammu and Kashmir. On the other hand, the states showing decline in forest cover are Manipur,
Meghalaya, Arunachal Pradesh and Mizoram.
Ayushman Bharat
The scheme proposed to set up one and a half Health and Wellness Centres (HWC) by 2022. So far 28,005
HWCs have been set up as on 14th January, 2020.
Out of Pocket Expenditure
The Out of Pocket Expenditure on Helath has declined from 64.2% in 2013-14 to 58.7% in 2016-17.
Job Creation
The job creation of salaried employees or regular wage employees increased by 5% in 2017-18 as compared
to 2011-12. The jobs increase was 18% in 2011-12 and increased to 23% in 2017-18
Around 2.62 crore new jobs were created. This included 1.21 crores in rural and 1.39 crores in urban.
Green Bond Market
India has become the second largest emerging Green Bond Market after China. India in 2019 joined the
International Platform of Sustainable Finance to increase the environmentally sustainable investments
Service Sector
The Service sector accounts to 55% of the Indian Economy according to the Survey. It contributes two-third
of the FDI inflows and 38% of Indian exports.
Other Inferences
The Inflation declined from 3.2% in April 2019 to 2.6% in December 2019. This reflects weakening of
demand in the economy
Women employment between 2011-12 and 2017-18 has increased by 8%
Amazon partnering with Eastern Railways sets up pick-up kiosk in Kolkata after Mumbai
After Mumbai, the Amazon India private e-commerce firm has set up its second pick up kiosk in Kolkata’s
Sealdah Railway Station.
Highlights
The pick-up kiosk has been set up in partnership with the Eastern Railways. It is to be noted that in 2019, the
E-Commerce firm had set up pickup kiosk in four railway stations of Mumbai. The pick up kiosk allows the
customers to pick up packages at railway stations.
Tie-up with Indian Railways
In October 2019, Amazon India tied up with the Indian Railways to transport customer packages through
Railways on the routes of Mumbai to New Delhi, New Delhi to Kolkata.
What are the benefits?
This tie up with the Amazon India, will help Indian Railways meet its freight and passenger earning targets.
This will help in its dwindling finances. The Freight earnings of Indian Railways till November 2019 has been

© 2020 GKToday | All Rights Reserved | https://www.gktoday.in 67


Current Affairs [PDF] - January 16-31, 2020

4.32% less than the same period of previous year and 19.72% lower than the target fixed. Similarly, the
passenger earning was 5.33% lesser than the same period of previous year.
ISRO preparing 500 low cost satellite launch vehicles in 5 years
The Indian Space Research Organization is preparing itself to launch vehicles of cost Rs 30-35 crores.
These vehicles are capable of carrying satellites weighing 500 kg
Highlights
According to ISRO, it has equipped itself to manufacture these satellite vehicles in around three weeks.
ISRO has allocated 1,600 million USD to manufacture these satellite vehicles. Of this allocated amount, 870
million USD is to be used to produce PSLV (Polar Satellite Launch Vehicle). The remaining is to be used for
Geosynchronous Satellite Launch Vehicle.
The plan of ISRO is to launch around 500 PSLVs in next five years.
The GOCO (Government Owned Contractor Operated) model has been performing well since its launch in
both defence and space sectors. The model will help in achieving the target of ISRO.
Current Scenario
India is looking for opportunities to collaborate with UK and France Space agencies. The UK Space Trade
Mission visited India in the mid of January 2020 to explore areas of cooperation.
India is also looking for opportunities to join hands with France to set up a model for the Moon Programme
that is currently under operation in India.
Vidya Bal-Social Activist, Feminist passed away
The Veteran Feminist, Social Activist and writer passed away in private nursing home on January 30, 2020.
She was known for her heroic oppressive practices involving women. She was a forefront fighter for women
equality with men
Highlights
In 2016, she approached Mumbai High Court demanding enforcement of law to allow women in all Hindu
Religious places. Her Public Interest Litigation was against restriction of women at Shani Shingnapur
temple under Maharashtra Hindu Place of Worship Act, 1956. She won the case paving way for women’s
right inside the temple.
Apart from Shani Shingnapur, she has also fought several legal battles for women to enter religious places
of worship.
About Vidya Bal
She worked as an editorial staff of monthly magazine Stree in 1964. Later she founded Miloon Saryajani in
1989.
She has written famous biographies namely Kamlaki and Valvantatil.
Her literary works include Sanwad, Katha Gaurichi, Tumachya Majhyasathi, Shodh Swatahacha,
Aparajitanche Nihshwas.
Indian Railways: First Banana Container Train to boost exports; First High Capacity Parcel Van
With its firsts increasing, Indian Railways has recently launched a high capacity parcel van at Delhi
Safdarjung Railway Station.
Also, the Indian Railways has launched an exclusive Banana Container train to boost the exports of the fruit.
Highlights: Parcel Van
The new parcel van was designed and manufactured in Kapurthala Rail Coach Factory. It is the first of its
kind in Linke Hofmann Busch (LHB) coaches. The LHB coaches are the coaches of the Indian Railways that
have been developed by the LHB company of Germany. They are mostly produced in Kapurthala Coach
Factory.
The parcel van is to run at high speed of 130 km per hour. The train includes collapsible partitions and

© 2020 GKToday | All Rights Reserved | https://www.gktoday.in 68


Current Affairs [PDF] - January 16-31, 2020

sliding doors. Also, it is Head On Generation compliant.


Highlights: Fruit Train
On January 30, 2020, Indian Railways launched the first “Fruit Train”. The train will transit from Anantpur in
Andhra Pradesh to Mumbai’s Jawaharlal Nehru Port. The train dispatched the first shipment of 890 tonnes
of banana in 43 refrigerated containers.
The APEDA (Agricultural and Processed Food Products Export Development Authority) along with export
firms trained 500 farmers to cultivate bananas in more than 1800 hectares of land.
First indigenous bio-jet fuel powered IAF aircraft lands successfully in Leh
The AN-32 aircraft of Indian Air Force powered with 10% blend of bio-jet fuel took off and landed
successfully in Leh airport.
Highlights
This is the first time, bot the engines of an aircraft are powered with bio-jet fuel. Leh is located at an altitude
of 10,682 feet above sea level and is one of the highest operational air field. It challenging to fly an aircraft
in the region due to wind turbulence and proximity of mountainous terrain.
Fuel Technology
The technology of the fuel was produced by CSIR-IIP in 2013. However, it couldn’t be used for commercial
purposes due to lack of aviation test facilities. Indian Air force in 2018, sponsored the project and
channelized material resources and human resources to complete fuel testing.
The bio-jet fuel is manufactured from “Tree-Borne oils”. The raw materials for the oil is grown and procured
from Chhattisgarh tribal areas. This will help in reducing India’s dependence on crude oil imports and also
will assist in reducing carbon emissions.
Current Affairs Quiz: January 16, 2020
1. When is the Army Day celebrated across India, every year?
[A] January 13
[B] January 14
[C] January 15
[D] January 16
Answer: January 15
Army Day is celebrated every year on January 15, across the country to honour the contributions made
by the soldiers to the country. The 72nd Army day was recently observed on January 15, 2020.
On January 15, 1949, General KM Carriappa became the first Commander-in-Chief of Indian Army after
Independence. This day is being commemorated every year as Army Day. The Indian Army holds Army
Parade every year in the Parade ground, New Delhi. Captain Tania Shergill becomes the first female to
lead an all-men contingent at the Army Day Parade.
2. Which bank has recently signed an MoU with South-Central Railway zone of the Indian Railways, for
doorstep collection of earnings from the railway stations?
[A] Punjab National Bank
[B] Bank of Baroda
[C] State Bank of India
[D] Bank of India
Answer: State Bank of India
The State Bank of India and the South-Central Railway recently signed an agreement for doorstep
collection of earnings from all the 585 railway stations of the zone.
At present, the earnings of the smaller stations are sent through cash chest available with the Train
Guards, the process being very tedious. The agreement will ensure collection of traffic earnings by the
© 2020 GKToday | All Rights Reserved | https://www.gktoday.in 69
Current Affairs [PDF] - January 16-31, 2020

bank and it will be remitted to the Government account without any delay.
3. India recently provided nearly 1 lakh academic text books to which country, to assist its development in
higher education and research?
[A] Madagascar
[B] Malawi
[C] Mozambique
[D] Zimbabwe
Answer: Madagascar
The Indian ambassador for Madagascar, Abhay Kumar recently handed over nearly 1 lakh academic text
books to the East African country, Madagascar for assisting its development in higher education and
research. The books were received by the Minister for Higher Education and Scientific Research of
Madagascar, in an event held in its capital city, Antananarivo.
The books were published by the National Council for Educational Research and Training (NCERT) and
the total cost of the books is nearly 76 lakh rupees.
4. Which cricketer won the ‘Sir Garfield Sobers Trophy’ for ‘Player of the Year’, presented by the International
Cricket Council (ICC)?
[A] Virat Kohli
[B] Ben Stokes
[C] Kane Williamson
[D] Rohit Sharma
Answer: Ben Stokes
The International Cricket Council (ICC) recently presented the ICC Annual awards. The biggest award
named ‘Sir Garfield Sobers Trophy’ for Player of the Year was bagged by the England All-rounder Ben
Stokes.
Indian opener, Rohit Sharma was named the ‘ODI Player of the year’, while the Australian fast bowler Pat
Cummins was presented the ‘Test Player of the year award’. The ‘Spirit of Cricket Award’ was presented
to Virat Kohli. Another Indian cricketer, Deepak Chahar was given the ‘T20I Performance of the Year’
award.
5. Which company has recently announced to invest $ 1 billion to digitise Small and Medium Businesses in
India?
[A] Google
[B] Amazon
[C] Microsoft
[D] IBM
Answer: Amazon
Recently, the head of Amazon, Jeff Bezos announced that the company will invest $ 1 billion to digitise
Small and Medium Businesses in India. Jeff also announced that Amazon India will export several ‘Make
in India’ products, worth $10 billion, by the year 2025.
He made this announcement during his recent official visit to India, where he met many Small and
Medium businessmen, top Indian business leaders and government officials. Recently, The
Confederation of All India Traders (CAIT) alleged that the company follows unfair business practices.
6. Which organisation releases the Annual Status of Education Report (ASER) in India?
[A] Ministry of Human Resources and Development
[B] NITI Aayog

© 2020 GKToday | All Rights Reserved | https://www.gktoday.in 70


Current Affairs [PDF] - January 16-31, 2020

[C] NGO ‘Pratham’


[D] UNICEF
Answer: NGO ‘Pratham’
The Non-Governmental organisation, called ‘Pratham’ releases the Annual Status of Education Report
(ASER) in India. The ASER for the year 2019 was released recently. The report states that five-year-old
children in private institutions performed better in learning activities than the same-aged children in
Government-run schools and anganwadis.
The enrolment levels among girls in the age group of 4 to 8 years, were higher than those among boys.
But more girls are being enrolled in state-run schools than boys, who are enrolled more in private
schools, showing a clear gender gap.
7. Which global tele-communication company’s arm has recently launched a new privacy-focused search
engine called ‘OneSearch’?
[A] AT&T
[B] Verizon
[C] Vodafone
[D] China Mobile
Answer: Verizon
Verizon Media, an arm of the international tele- communication company, Verizon communications, has
recently launched a new privacy-focused search engine called ‘OneSearch’.
The search engine claims that the search history of the users will not be stored and the self-destructible
search data of the users will not be shared with advertisers. The new search engine is also expected to
give unbiased and unfiltered results, which will be personalised only based on locations and will
separate IP addresses from users and their search results.
8. Eminent filmmaker and director Manmohan Mohapatra, who recently passed away, was from which
state?
[A] West Bengal
[B] Odisha
[C] Karnataka
[D] Andhra Pradesh
Answer: Odisha
Veteran filmmaker and director Manmohan Mohapatra from Odisha recently passed away. He had won
National Film awards for best Odia feature film, for eight consecutive years.
Seeta Raati was his first full-fledged Odia film, which he directed in 1976. It was the first Odia film to be
screened at an international film festival, held abroad. He also directed several Hindi films. The Chief
Minister of Odisha, Naveen Patnaik has condoled the demise of the eminent director.
9. The District administration of which state has won the Swachhata Darpan Award for adopting innovative
methods in plastic waste management?
[A] Telangana
[B] Kerala
[C] Odisha
[D] Gujarat
Answer: Odisha
The district administration of Puri, Odisha has recently won the Swachhata Darpan Award for adopting
innovative methods in plastic waste management. The award was presented by the Department of

© 2020 GKToday | All Rights Reserved | https://www.gktoday.in 71


Current Affairs [PDF] - January 16-31, 2020

Drinking Water and Sanitation (DDWS) under Ministry of Jal Shakti.


The award was presented during the workshop organised by ODF Sustainability in New Delhi, by the
Bollywood actor Aamir Khan. The Odisha Government has launched a scheme last year, to make Puri
completely free from plastic.
10. The ‘Integrated Road Accident Database (IRAD)’ recently launched at the road safety stakeholders’ meet,
was developed by which institution?
[A] Indian Institute of Technology -Madras
[B] Indian Institute of Science
[C] Indian Institute of Technology- Kanpur
[D] Indian Institute of Technology- Kharagpur
Answer: Indian Institute of Technology -Madras
Union Ministers Rajnath Singh and Nitin Gadkari launched the ‘Integrated Road Accident Database
(IRAD)’ during the road safety stakeholders’ meet. IRAD is a central accident database management
system, to analyse the causes of road accidents and create solutions to reduce them.
The database was developed by the Indian Institute of Technology-Madras (IIT-M) with the project cost
of Rs. 258 Crores. This initiative is supported by the World Bank and it is implemented through the
National Informatics Centre.
Current Affairs Quiz: January 17, 2020
1. According to the recent study conducted by the Swiss bank Julius Baer, which is the most expensive city
of the world for luxury goods?
[A] Mumbai
[B] Hong Kong
[C] New York
[D] Singapore
Answer: Hong Kong
The Swiss Bank Julius Baer has recently conducted a study on the 28 cities around the world to rank
them as expensive in terms of luxury goods and services. According to the study, the bank tracked a
basket of luxury goods and the most expensive city for the selected luxury goods is Hong Kong.
Asia is considered to be the most expensive continent as the 2nd, 3rd and 5th ranks were bagged by
Shangai, Tokyo and Singapore respectively.
2. The ‘National Highway Excellence Awards’, which was recently presented by the Union Road, Transport
and Highway Minister, was instituted in which year?
[A] 2016
[B] 2017
[C] 2018
[D] 2019
Answer: 2018
The ‘National Highway Excellence Awards’ was instituted by the Union Ministry of Road, Transport and
Highway, in the year 2018. It was then changed to an annual version and the second edition of the
awards was presented by the Union Minister Nitin Gadkari in New Delhi recently.
The awards are presented to motivate the companies that perform well in the construction, operation,
maintenance, tolling phase and road safety. There are seven categories in the award and 12 winners
were selected from several shortlisted applicants.
3. The country’s first-of-its-kind exhibition on ‘Indian Heritage in Digital Space’ was inaugurated in which
© 2020 GKToday | All Rights Reserved | https://www.gktoday.in 72
Current Affairs [PDF] - January 16-31, 2020

city?
[A] Varanasi
[B] Puri
[C] New Delhi
[D] Jaipur
Answer: New Delhi
Union Culture Minister Prahlad Singh Patel inaugurated the country’s first of its kind special exhibition
on ‘Indian Heritage in Digital Space’ and a two-day-long first international heritage symposium in New
Delhi.
The exhibition was organised in association with Indian Institute of Technology, Delhi and it will be made
open to the general public for one month. Digital installations of Indian monuments were showcased,
which were created under the Indian Digital Heritage (IDH) initiative of the Department of Science and
Technology (DST).
4. The 7th Conference of Commonwealth Parliamentary Association (India Region) was organised in which
city?
[A] New Delhi
[B] Mumbai
[C] Lucknow
[D] Ahmedabad
Answer: Lucknow
The 7th Conference of Commonwealth Parliamentary Association (CPA)- India Region, was recently
inaugurated in Lucknow, Uttar Pradesh.
The Speaker of Lok Sabha, Om Birla inaugurated the conference at the legislative assembly building in
Lucknow. Several parliamentarians and delegates from some foreign countries will participate in the
conference, whose theme is ‘Role of Legislators’. The CPA was formed to support the Parliamentarians
and their staff to implement good governance measures and values of the Common wealth.
5. How many Indian astronauts have been selected for India’s first human space mission?
[A] Two
[B] Three
[C] Four
[D] Five
Answer: Four
Four astronauts, who are from Indian Air Force were selected for the first human space mission of India.
Recently it was announced that the four astronauts will undergo a training in Russia for 11 months,
which is to be started this month.
After the training phase in Russia, the astronauts will be trained in operation and simulation of the crew
and service module in India, which has been designed by the Indian Space Research Organisation
(ISRO). Before the first manned mission, ISRO will send a humanoid to space by the end of 2020.
6. Which cricketer was named as the Captain of ICC’s ODI and Test teams of the year?
[A] Mahendra Singh Dhoni
[B] Virat Kohli
[C] Kane Williamson
[D] Eoin Morgan
Answer: Virat Kohli

© 2020 GKToday | All Rights Reserved | https://www.gktoday.in 73


Current Affairs [PDF] - January 16-31, 2020

The International Cricket Council recently announced its team of the year and also presented the annual
awards. Indian Cricket team skipper Virat Kohli was named as the Captain of both ICC’s ODI and Test
Teams of the year. Apart from Virat, only Mayank Agarwal found place in the ICC Teat team, while the
other Indians in the ODI Team were Rohit Sharma, Mohammed Shami and Kuldeep Yadav.
Virat has also won the Spirit of Cricket award, presented by the ICC. Along with Virat, who captains both
the teams, Ben Stokes of England and Mitchell Starc of Australia were placed in both the ODI and Test
teams of ICC.
7. The United States recently signed the first phase of historic trade deal with which country?
[A] India
[B] China
[C] Russia
[D] Switzerland
Answer: China
After a year of several negotiations, the US signed the first phase of the trade deal with China. The
agreement, which was signed by the US President, Donald Trump and the Vice Premier of China, Liu He.
The first phase of the deal, which Trump quoted as ‘historic’, includes Protection of Intellectual Property,
Removal of American Financial Services and Ending Currency Manipulation. This deal is expected to re-
balance the US- China bilateral relations and resolve the long-pending disputes between the countries.
8. Alok Kansal has been recently appointed as the General Manager of the Western Railway. Where is the
headquarters of Western Railways located?
[A] Gandhi Nagar
[B] Ahmedabad
[C] Mumbai
[D] Surat
Answer: Mumbai
Western Railway zone of the Indian Railways connects Mumbai, the financial capital of India with Gujarat
and some parts of Madhya Pradesh and Rajasthan, with its headquarters being the Churchgate station
of Mumbai.
Alok Kansal, a 1983-batch officer of the Indian Railway Service of Engineers (IRSE) has been recently
appointed as the General Manager of the Western Railway zone. The previous General Manager was Anil
Kumar Gupta, who retired on November 2019
9. Which technology company has recently launched the new version of its ‘Edge Chromium’ browser for
both Mac and Windows platforms?
[A] Google
[B] Microsoft
[C] Mozilla
[D] Apple
Answer: Microsoft
Tech Major Microsoft has recently announced the launch of its new Edge Chromium browser for both
the Mac and Windows platforms.
Though the enterprise users are initially targeted by Microsoft, other Windows and macOS users could
also download the browser manually from the official website of Microsoft and install it. It is expected
that the Windows 10 users will be provided the browser through a windows update. The Edge has similar
features like those in Chrome and also supports Chrome extensions.

© 2020 GKToday | All Rights Reserved | https://www.gktoday.in 74


Current Affairs [PDF] - January 16-31, 2020

10. The entire government of which country resigned recently after the constitutional amendments
announced by its President?
[A] China
[B] France
[C] Russia
[D] Germany
Answer: Russia
After the President of Russia, Vladimir Putin announced certain constitutional amendments that would
extend his powers beyond Presidency, the entire government of Russia resigned. This also includes the
Prime Minister of Russia, Dmitry Medvedev.
Putin made this announcement during his annual state of the nation address. He also proposed Mikhail
Mishustin, the head of Russia’s tax service as the next Prime Minister. The leaving Prime Minister
Medvedev may take on the position of deputy head of Russia’s Security Council.
Current Affairs Quiz: January 18, 2020
1. Which major tech-company recently announced that it has planned to cut more than half of carbon
emissions across its supply chain, by 2030?
[A] Google
[B] Microsoft
[C] Infosys
[D] IBM
Answer: Microsoft
The Chief Executive of Microsoft Corporation, Satya Nadella recently announced that the company has
set a new ambition to address climate change and promised to remove the carbon emissions the
company has made.
Microsoft has planned to cut more than half of carbon emissions across its supply chain by 2030. The
plan also includes the creation of a “Climate Innovation Fund,” which will invest $1 billion for developing
technologies to remove carbon. The company aims to remove the entire direct and indirect carbon
emissions it had made since 1975 (founding year of Microsoft), by 2050.
2. Recently, the toll-free number ‘8884333331’ for Missed Call Alert facility of users, was launched for which
application?
[A] FASTags
[B] BHIM
[C] UMANG
[D] NPS
Answer: FASTags
Indian Highways Management Company, incorporated by National Highway Authority of India (NHAI),
has recently introduced the ‘Missed Call Alert Facility’ to check the FASTag balance of the users.
The toll-free number ‘8884333331’ will be available throughout the day. The users of FASTags, who have
registered their mobile numbers with NHAI Prepaid wallet, can give a missed call to the number, to
check their FASTag balance. The prepaid payment can be done through the My FASTag App, which can
be linked to the NHAI wallet or to the Savings account of 13 banks.
3. The second Tejas Express, which was recently flagged off, connects Mumbai with which city?
[A] Pune
[B] Hyderabad

© 2020 GKToday | All Rights Reserved | https://www.gktoday.in 75


Current Affairs [PDF] - January 16-31, 2020

[C] Ahmedabad
[D] Nagpur
Answer: Ahmedabad
The second Tejas Express in the Mumbai- Ahmedabad route was recently flagged off from Ahmedabad
by the Chief Minister of Gujarat, Vijay Rupani. The commercial run is expected to start on January 18 and
it will run in the route for all days of the week except Thursday.
Tejas Express is a semi-high speed and fully air-conditioned train, which is run by the wholly owned
subsidiary of Indian Railways, Indian Railway Catering and Tourism Corporation (IRCTC). The first Tejas
Express between New Delhi and Lucknow was inaugurated in October 2019. For Tejas Express, IRCTC
follows a dynamic pricing scheme and it pays compensation to the customers on train delays.
4. The ‘World Economic Situation and Prospects report’ is released by which organisation?
[A] World Bank
[B] United Nations (UN)
[C] International Monetary Fund (IMF)
[D] Asian Development Bank (ADB)
Answer: United Nations (UN)
The United Nation’s World Economic Situation and Prospects (WESP) report was recently released. As
per the report, the UN projects the GDP growth of India in the current fiscal year to be 5.7 %. The UN also
predicted that the GDP growth rate of the next fiscal year will rise to 6.6%.
In the WESP report of 2019, the UN had predicted India’s growth rate to be 7.6% for the current fiscal
year, which has now been lowered to 5.7%. As per the recent report, East Asia continues to be the
world’s fastest growing region and the largest contributor to global growth. The report also predicted
that the global growth would dip to 1.8 per cent this year.
5. The first edition of “Krishi Manthan” summit is being held in which state?
[A] Assam
[B] Tamil Nadu
[C] Andhra Pradesh
[D] Gujarat
Answer: Gujarat
The 1st edition of Krishi Manthan is being held at Ahmedabad, Gujarat. Krishi Manthan is the Asia’s
largest “Food-Agri-business-Rural Development” Summit. It is organized by the Food and Agri Business
Committee of the Indian Institute of Management (IIM), Ahmedabad. It is a platform for Business,
industry, academia, policymakers and other stakeholders to share their knowledge and expertise.
It is a 2-day event where more than 1,500 delegates including students, working professionals from
Food, Agriculture and allied sectors would be participating.
6. Who has been recently conferred with the 29th Saraswati Samman Award?
[A] Vikram Seth
[B] Mahabaleshwar Sail
[C] Vasdev Mohi
[D] Padma Sachdev
Answer: Vasdev Mohi
The famous Sindhi Writer and poet Vasdev Mohi has been chosen for the prestigious 29th Saraswati
Samman Award. The Saraswati Samman Award is a famous award in the field of Literature, presented by
the KK Birla Foundation.

© 2020 GKToday | All Rights Reserved | https://www.gktoday.in 76


Current Affairs [PDF] - January 16-31, 2020

Vasdev Mohi is being conferred with this award for his short story collection named ‘CHEQUE BOOK’.
The collection of short stories describes the sufferings and agonies of the marginalised section of the
society. This was published in 2012 in Sindhi language.
Vasdev Mohi has recieved several other important national awards like Sahitya Akademi Award, Sindhi
Sahitya Academy Award, Gangadhar Mehar National Award and Lifetime Achievement Award of Sindhi
Academy.
7. The Government of India and the Government of Assam has signed a Loan Agreement with which
International Financial Organisation, to modernise Assam’s Passenger Ferry Sector?
[A] Asian Development Bank
[B] Asian Infrastructure Investment Bank
[C] World Bank
[D] New Development Bank
Answer: World Bank
To help modernise the passenger ferry sector of Assam, the Government of India, Government of Assam
and the World Bank has signed a loan agreement at New Delhi for $ 88 million.
The Assam’s passenger ferry sector operates passenger freight service in major rivers including the
Brahmaputra. There are more than 360 ferry routes in Assam, and most of them cut through the
Brahmaputra or serve its islands. Hence, this is a very important means of transportation in the
Brahmaputra Valley. The Assam Inland Water Transport Project (AIWTP) will help the state to improve
the passenger ferry infrastructure.
8. Which Indian satellite has been recently launched from French Guiana?
[A] RISAT 2B
[B] GSAT 30
[C] GSAT 7A
[D] IRNSS 1H
Answer: GSAT 30
The Indian Space Research Organisation’s high-power communication satellite, GSAT 30 has been
launched successfully from French Guiana on 17th January, 2020. The aim of this satellite is to provide
high quality telecommunication, Cellular and Broadcasting service.
The satellite was put in place in the Geo Synchronous Orbit by the Ariane-5 rocket. GSAT 30 weighs 3357
kg and it is to serve as the replacement satellite to INSAT-4A with enhanced coverage. The operational
life of the GSAT 30 is expected to be more than 15 years.
9. Which country is to offer e-visa facility to Indian Tourists?
[A] Canada
[B] South Africa
[C] Kenya
[D] Ireland
Answer: South Africa
Indian Tourists will be offered with the facility of e-Visa by South Africa. This information has been
furnished by the South African Tourism Minister, Mmamoloko Kubayi-Ngubane. The online Visa Portal
would be launched by next week on a pilot basis in India.
The most number of Visa requests from Mumbai is processed by Nigeria followed by South Africa. As
per the official data, over 81316 Indians have travelled to South Africa as of October 2019 registering a
30% increase on year-on-year basis and this is expected to grow up to 50% in near future.

© 2020 GKToday | All Rights Reserved | https://www.gktoday.in 77


Current Affairs [PDF] - January 16-31, 2020

10. What is the name of the Biggest Airborne Exercise recently conducted by the Indian Army?
[A] Him Vijay
[B] Dhruv Sakthi
[C] Winged Raider
[D] Yudh Abhiyas
Answer: Winged Raider
The biggest airborne exercise called the “Winged Raider” was conducted by the Indian Army in North
Eastern Theatre recently. This exercise witnessed the participation of more than 500 special force
troops, who parachuted from C-130 Hercules and C-17 Globemaster transporter Air-crafts of Indian Air
Force.
During the exercise, the newly inducted aerial equipments were tested with acute precision and
seamless integration between the Indian Army and Indian Air Force in some of the most difficult
terrains. This exercise demonstrated the operational readiness of Paratroopers and Air Warriors to
undertake airborne missions.
Current Affairs Quiz: January 19-20, 2020
1. Which Indian entrepreneur was recently honoured with the Australia’s highest civilian honour,‘Order of
Australia’?
[A] Anand Mahindra
[B] Kiran Mazumdar-Shaw
[C] Azim Premji
[D] Uday Kotak
Answer: Kiran Mazumdar-Shaw
The Founder and Chairperson of the famous Bio-technology company Biocon, Kiran Mazumdar Shaw
was recently honoured with the Australia’s highest civilian honour, ‘Order of Australia’.
The award was presented to her for significant service to promote bilateral relationship between
Australia and India, especially in commercial and educational aspects. Kiran Mazumdar Shaw is the
fourth Indian citizen to receive this honour after Mother Teresa, former Attorney General Soli Sorabjee
and ace cricketer Sachin Tendulkar.
2. How is the ‘Revised National TB Control Programme (RNTCP)’, the scheme of Ministry of Health & Family
Welfare, recently renamed?
[A] National Tuberculosis Elimination Programme (NTEP)
[B] National Tuberculosis Control Programme (NTCP)
[C] National Tuberculosis Removal Programme (NTRP)
[D] National Tuberculosis Abolition Programme (NTAP)
Answer: National Tuberculosis Elimination Programme (NTEP)
The Revised National TB Control Programme (RNTCP), the scheme of Ministry of Health & Family
Welfare was recently renamed as the National Tuberculosis Elimination Programme (NTEP).
The Union Ministry of Health & Family Welfare recently intimated the Chief Secretaries of all States and
Union Territories about the change of name. In 2018, the Prime Minister of India Narendra Modi
announced 2025 as the target year for eliminating Tuberculosis form India, 5 years ahead of the
Sustainable Development Goal. On account of the target, the programme was renamed to NTEP.
3. Bapu Nadkarni, who recently passed away, belonged to which field?
[A] Politics
[B] Literature

© 2020 GKToday | All Rights Reserved | https://www.gktoday.in 78


Current Affairs [PDF] - January 16-31, 2020

[C] Sports
[D] Arts
Answer: Sports
Former Indian cricketer Bapu Nadkarni passed away recently in Mumbai. He was a left-arm spinner and
an all-rounder, who played for India in international cricket matches. He had created the record of
bowling 21 successive maiden overs, when he achieved it in the test match against England.
Known for his bowling skills, Nadkarni had a bowling economy of just 1.64 after playing in 191 first-
class matches. Nadkarni represented India in 41 Test matches and subsequently served as a member of
the National Selection Committee and the joint secretary of Mumbai Cricket Association.
4. Which famous international company is to release its original film named ‘The Banker’?
[A] Amazon
[B] Apple
[C] Netflix
[D] HBO
Answer: Apple
Apple is scheduled to release one of its first-ever original films ‘The Banker’, which is one of the earliest
films for Apple TV plus.
The movie, which was set to be released in December last year was postponed due to some allegations.
So, it will be released in theatres and subsequently to its subscribers in the month of March, this year.
The movie is based on the real-life story of two African-American businessmen of 1960s, Bernard
Garrett Sr. and Joe Morris who helped people to overcome the racial barriers in the field of banking.
5. Which Indian wrestler won the Gold medal at the Rome Ranking Series 2020 event, held recently?
[A] Geeta Phogat
[B] Babita Phogat
[C] Vinesh Phogat
[D] Sakshi Malik
Answer: Vinesh Phogat
Top Indian wrestler Vinesh Phogat won the Gold medal in the 53 kg category, at the Rome Ranking
Series 2020 event, held recently. In the final bout for gold, she defeated the Ecuadorian wrester Luisa
Elizabeth Valverde 4-0. In the journey towards the final match, Vinesh overpowered two top-rated and
tough Chinese wrestlers. This is her first Gold medal in the year 2020.
Earlier, in the tournament, another Indian wrestler Anshu Malik won the silver medal in the 57 kg
category.
6. The birth rate of which country fell to the lowest, since the founding of the country seven decades ago?
[A] India
[B] China
[C] United States
[D] Brazil
Answer: China
An official data from China revealed that the country’s birth rate fell to the lowest, since 1949, the year in
which the People’s Republic of China was founded seven decades ago. According to the data, the birth
rate was 10.94 per thousand, which was lesser than the previous birth rate of 12.43 per thousand, in
2017.
As China has a rapidly growing aging population, it allowed urban couples to give birth to two children,

© 2020 GKToday | All Rights Reserved | https://www.gktoday.in 79


Current Affairs [PDF] - January 16-31, 2020

against the one-child policy which was in effect. As per another recent study, India will overtake China in
population in 2027.
7. The 6.5-kilometre long Z-Morh tunnel, which was seen in news recently, is to be constructed in which
state/UT?
[A] Himachal Pradesh
[B] Sikkim
[C] Jammu and Kashmir
[D] Assam
Answer: Jammu and Kashmir
The construction project of the Z-Morh Tunnel, to be built in in Jammu and Kashmir has been awarded to
APCO Amarnathji Tunnelway. An MoU was recently inked between National Highways and Infrastructure
Development Corporation (NHIDCL) and the company in this regard, in the presence of Road Transport
and Highways Minister Nitin Gadkari.
The project entails a cost of Rs. 2,379 crores and it would provide road connectivity to the tourist spot
Sonamarg, in Jammu & Kashmir. The tourist destination mostly remains closed due to heavy snowfall.
8. The passengers returning from which country are to be screened for novel Corona virus (nCov), in
selected Indian airports?
[A] Indonesia
[B] China
[C] Thailand
[D] Japan
Answer: China
For nearly three weeks, China is hit by a new viral disease which was reported to spread by novel Corona
virus (nCov). It killed one person and infected more than 40 in the country. Two other people in Japan
and Thailand who had travelled to China were also infected by the virus. Recently, the Union Health
Ministry instructed the airport authorities of Delhi, Mumbai and Kolkata to screen passengers returning
from China and to check through thermal scanners if they carry the virus.
Coronaviruses are a large family of viruses, that cause diseases such as the Middle East respiratory
syndrome coronavirus (MERS-CoV) and Severe acute respiratory syndrome (SARS).
9. The ‘APNA UREA – Sona Ugle’ brand of Hindustan Urvarak and Rasayan Limited (HURL) was recently
launched by which Union Minister?
[A] Ramvilas Paswan
[B] D.V. Sadananda Gowda
[C] Harsimrat Kaur Badal
[D] Harsh Vardhan
Answer: D.V. Sadananda Gowda
The Union Chemicals and Fertilizers Minister D.V. Sadananda Gowda recently launched the ‘APNA UREA
– Sona Ugle’ brand and logo of Hindustan Urvarak and Rasayan Limited (HURL).
HURL is a Joint Venture Company, which is promoted by the three PSUs and Maha Ratna Companies –
Coal India Limited, NTPC Limited and Indian Oil Corporation Limited. HURL has undertaken the revival
process of three sick Urea plants, situated at Gorakhpur (Uttar Pradesh), Barauni (Bihar) and Sindri
(Jharkhand). The plants are to be commissioned in February 2021.
10. China’s Great Wall Motors (GWM) is set to acquire the manufacturing plant of which automobile
company located in Maharashtra?

© 2020 GKToday | All Rights Reserved | https://www.gktoday.in 80


Current Affairs [PDF] - January 16-31, 2020

[A] General Motors


[B] Ford
Ford
[C] Volkswagen
[D] Toyota
Answer: General Motors
China’s leading automobile manufacturer, Great Wall Motors (GWM) has signed an agreement with the
American manufacturer General Motors (GM) to acquire GM’s manufacturing plant located in Talegaon,
near Pune in Maharashtra.
General Motors announced that Chevrolet Sales India, which is the sales arm of GM in India will continue
as an independent entity. It will continue to provide after-sales service and spare part support to existing
Indian customers. GWM has planned to enter into the Indian market by 2021 and this acquisition would
speed-up its launch.
Current Affairs Quiz: January 21, 2020
1. Which famous International Tennis tournament is held in the month of January, every year?
[A] US Open
[B] Australian Open
[C] French Open
[D] Wimbledon
Answer: Australian Open
The Australian Open tournament is the first of the four Grand Slam tennis tournaments held every year,
before the French Open, Wimbledon and the US Open events. It is organized by Tennis Australia and
generally held during the last fortnight of every January. The event comprises of many categories
including Singles and Doubles matches for Men and Women, Mixed Doubles, Junior Championship and
wheel-chair matches.
The 2020 Australian Open is the 108th edition, scheduled from January 20 to February 2 at Melbourne
Park, Victoria, Australia. The defending Champions of the event in Men’s singles and Women’s singles
are Novak Djokovic and Naomi Osaka respectively.
2. Which state recently organized the world’s longest awareness human-chain for more than 18000 km?
[A] Uttar Pradesh
[B] Bihar
[C] Odisha
[D] West Bengal
Answer: Bihar
The state government of Bihar under its ‘Jal-Jeevan-Hariyali’ campaign, recently organized a record
18034 km long human chain, which witnessed the participation of more than 5 Crore people.
The Bihar Chief Minister Nitish Kumar participated in the event along with the Deputy Chief Minister and
his Cabinet members. The human chain was organized to create awareness among the public on climate
change and social ills such as dowry and child-marriage. The human-chain, which is claimed to be the
longest in the world is the third of its kind event after the chains formed in Bihar in 2017 and 2018.
3. Which Indian city has been adjudged as the ‘World’s most dynamic city’, as per the JLL City Momentum
Index, 2020?
[A] Mumbai
[B] Bengaluru

© 2020 GKToday | All Rights Reserved | https://www.gktoday.in 81


Current Affairs [PDF] - January 16-31, 2020

[C] Kolkata
[D] Hyderabad
Answer: Hyderabad
The capital of Telangana and a major hub of technology industries, Hyderabad was recently adjudged as
the ‘World’s most dynamic city’, as per the JLL City Momentum Index, 2020.
The US-based by global real estate services firm Jones Lang Lasalle (JLL), recently released the index
by assessing 129 cities around the world. In this seventh edition of the ranking, Hyderabad was placed
at the top spot followed by another Indian tech-hub Bengaluru. Southern city Chennai is placed at the
fifth place while Delhi is at the sixth place. India leads the 2020 Index with its seven cities being placed
in the top 20 ranks.
4. Which country recently test-fired a nuclear-capable ballistic missile, named ‘K-4’, which can be launched
from submarine?
[A] China
[B] Russia
[C] India
[D] Israel
Answer: India
Recently, India has successfully test-fired fired a 3,500 km range nuclear capable missile from the coast
of Andhra Pradesh. The ballistic missile named ‘K-4’ that can be launched from a submarine, is
developed by Defence Research and Development Organisation (DRDO).
The missile, which is to be fitted into the Arihant-class submarines of the Indian Navy, has ideal
configuration to strike its targets. Arihant is India’s first nuclear-powered submarine of 6000 tonnes,
which was successfully tested in November 2018.
5. Recently, 146 Irrawady dolphins were recently spotted in Chilika lake. Where is the Chilka lake situated?
[A] Himachal Pradesh
[B] Odisha
[C] Jammu & Kashmir
[D] Uttarakhand
Answer: Odisha
Chilika Lake is the largest brackish water lake of India, situated in the state of Odisha. It is also home to
the highest single lagoon population of the endangered species called Irrawady dolphins.
The Chilika Development Authority (CDA) recently undertook the Dolphin Census and 146 Irrawady
dolphins were directly sighted. The estimated range of population of the dolphins is 133-172. The
population of the aquatic mammal species last year was 151. After removal of certain encroachments in
the lake, the dolphins are found colonising in the central and southern regions of the lake for the first
time.
6. Which state is to set up a committee to suggest measures to improve forest tourism?
[A] Madhya Pradesh
[B] Maharashtra
[C] Karnataka
[D] Kerala
Answer: Maharashtra
The Maharashtra government is to set up a committee to suggest measures to increase the number of
visitors to the state’s forest areas. The decision was taken at a meeting chaired by Aaditya Thackeray,

© 2020 GKToday | All Rights Reserved | https://www.gktoday.in 82


Current Affairs [PDF] - January 16-31, 2020

the state’s Tourism Minister.


The proposed committee will comprise of members from the state tourism and forest development
corporation. The main objective of the panel would be promoting forest conservation as well as
boosting tourism in the state. The infrastructure of the existing tourism centres would also be improved
7. The Union Home Minister Amit Shah is to launch POL NET 2.0 platform. What is POLNET 2.0 associated
with?
[A] Political campaign
[B] Political Network Service
[C] Police Network Service
[D] Police complaint registration
Answer: Police Network Service
The Union Home Minister Amit Shah is to launch the new platform POL NET 2.0. during the national
conference of ‘Heads of public protection and Disaster relief organisations’ on January 20 in New Delhi.
The Directorate of Coordination Police Wireless manages and operates POLNET or the police network
services in the country. The Directorate has created a new robust communication platform with
improved multi-media facilities, for Police and Security forces called POL NET 2.0. It will be beneficial
during disasters or law and order problems.
8. What is the name of the exhibition of the Indian electrical industry, recently inaugurated by the Union
Minister of Heavy Industries & Public Enterprises?
[A] ELECASE
[B] ELECRAMA
[C] ELECTRON
[D] ELECTRIC 2.0
Answer: ELECRAMA
Union Minister of Heavy Industries & Public Enterprises, MoEFCC, MoI&B, Prakash Javadekar
inaugurated ELECRAMA 2020 in Greater Noida, Uttar Pradesh. Minister of Power and New and
Renewable Energy (IC) R K Singh was also present during the inaugural ceremony.
ELECRAMA is organised by the Indian Electrical & Electronics Manufacturers’ Association (IEEMA), once
in every two years. It is an exhibition cum conference in the field of Indian electrical industry. The event
displays various products and technologies in the field and creates a trade platform between buyers and
sellers. It also hosts various seminars for knowledge exchange.
9. Name the female actor who won the ‘Outstanding Performance by a Female Actor in a motion picture’
award in the Screen Actors Guild Awards?
[A] Renée Zellweger
[B] Jennifer Aniston
[C] Olivia Colman
[D] Elisabeth Moss
Answer: Renée Zellweger
The 26th edition of the annual Screen Actors Guild (SAG) Awards were presented recently. The
‘Outstanding Performance by a Female Actor in a motion picture’ was won by Renée Zellweger for
featuring in the movie titled ‘Judy’.
Renée Zellweger recently received the Golden Globe award for her performance in the biopic of ‘Judy’.
The ‘Outstanding Performance by a Male Actor in a Leading Role’ in a motion picture was presented to
Joaquin Phoenix for the movie ‘Joker’. The South Korean film-Parasite became the first foreign-language

© 2020 GKToday | All Rights Reserved | https://www.gktoday.in 83


Current Affairs [PDF] - January 16-31, 2020

film to win the Award for best performance by a cast in a motion picture.
10. Which state cabinet has recently approved the proposal for creating three capitals for the state?
[A] Maharashtra
[B] Andhra Pradesh
[C] Madhya Pradesh
[D] Uttar Pradesh
Answer: Andhra Pradesh
The state cabinet of Andhra Pradesh has approved a proposal to create three state capitals namely
Amaravati, Visakhapatnam, and Kurnool. The ruling Jagan Mohan Reddy government introduced the bill
in the state assembly.
The ‘AP Decentralisation and Inclusive Development of All Regions Bill, 2020’ was approved by the
cabinet for a decentralised and balanced growth and development of the state. It also provides for
creating several zones within the state and establishing zonal planning and development boards. The
government is to set up its legislative capital in Amaravati, secretariat and executive capital in
Visakhapatnam and judicial capital in Kurnool.
Current Affairs Quiz: January 22, 2020
1. What is India’s GDP growth rate for the current fiscal, as estimated by the International Monetary Fund
(IMF)?
[A] 5.2 %
[B] 5.0 %
[C] 4.8 %
[D] 4.6 %
Answer: 4.8 %
The International Monetary Fund (IMF) decreased India’s GDP growth forecast to 4.8 per cent for this
fiscal year, which is 1.3 percentage point lesser than its previous estimates.
The IMF also projected that India’s economy will grow by 5.8 per cent next year and 6.5 per cent in
2021-2022. The Global economic growth rate was also lowered by the IMF to be 3.3 % in the current year
and 3.4 % in 2022. India’s GDP forecast dip was mainly due to the NBFC crisis and weak demand in rural
India. The slowdown in India is said to be the reason for cut in global growth rate.
2. Which Indian state has topped the production of vegetables in 2018-19, as per the recent data from the
Ministry of Agriculture?
[A] Uttar Pradesh
[B] West Bengal
[C] Madhya Pradesh
[D] Gujarat
Answer: West Bengal
The Ministry of Agriculture recently released the State-wise horticulture production data for the year
2018-19. As per the data, West Bengal was the top State in vegetable production with 29.55 million
tonnes (mt) of vegetables last year. The state accounted for 15.9 % of the country’s total vegetable
production in 2018-19.
The number one state during 2017-18 was Uttar Pradesh, which is pushed to the second place this year,
with 27.71 mt vegetables. In fruit production, Andhra Pradesh continued to hold the top spot with 17.61
mt, followed by Maharashtra with 10.82 mt.
3. Who was recently appointed as the new Managing Director of the State Bank of India (SBI)?
© 2020 GKToday | All Rights Reserved | https://www.gktoday.in 84
Current Affairs [PDF] - January 16-31, 2020

[A] Challa Sreenivasulu Setty


[B] Rajnish Kumar
[C] Atanu Kumar Das
[D] Sanjiv Chadha
Answer: Challa Sreenivasulu Setty
The Appointments Committee of the Cabinet has approved the proposal of the Department of the
Financial Services (DFS) for appointing Challa Sreenivasulu Setty as the Managing Director of the State
Bank of India (SBI).
Challa Sreenivasulu Setty, who is currently serving as the Deputy managing director of the country’s
largest lender, has been appointed as the MD of the SBI for a period of three years. Last year, an MD of
SBI, Anshula Kant was appointed as the MD and Chief Financial Officer of World Bank.
4. What is the rank of India in the ‘Social Mobility Index’, recently released by the World Economic Forum?
[A] 56
[B] 66
[C] 76
[D] 86
Answer: 76
In the recently released World Economic Forum’s Social Mobility Index, India was ranked at 76 out of 82
countries. Denmark was placed at the first rank in the inaugural edition of the index.
The index assessed the countries across five key dimensions, which are health, education, technology,
work and protections& institutions and ten pillars distributed among the five dimensions. According to
the index, the most socially mobile societies in the world are all European, the first five spots being held
by Denmark, Norway, Finland, Sweden and Iceland.
5. Which organisation recently released the ‘Time to care’ report?
[A] World Bank
[B] Asian Development Bank
[C] NGO Oxfam
[D] NGO Pratham
Answer: NGO Oxfam
Oxfam, a confederation of charitable organizations and rights group recently released the ‘Time to care’
report, which focussed on global inequalities. The report said that inequalities pertaining at the global
level is deep-rooted and vast while the number of billionaires has doubled in the last decade.
It also revealed a comparison that the total wealth of 63 Indian billionaires is higher than the total Union
Budget of India for the fiscal year 2018-19 (Rs 24,42,200 crore). It also suggested that gender
inequalities must be resolved for an inclusive development.
6. Who has been recently appointed as the President of Institute of Company Secretaries of India (ICSI)?
[A] Ashish Garg
[B] Aditya Garg
[C] Nagendra Rao
[D] Narendra Rao
Answer: Ashish Garg
The Institute of Company Secretaries of India (ICSI) recently appointed Ashish Garg as its new President
for the year 2020. Another member of ICSI, Nagendra D Rao has been elected Vice-President of the
Institute. Ashish Garg was elected to the central council of ICSI for the 2015-18 term and has been re-

© 2020 GKToday | All Rights Reserved | https://www.gktoday.in 85


Current Affairs [PDF] - January 16-31, 2020

elected for the 2019-22 term.


The Institute of Company Secretaries of India (ICSI) is a New Delhi based statutory body to promote and
regulate the profession of company secretaries in India.
7. Which country is the largest recipient of Foreign Direct Investment (FDI) in 2019, according to the latest
UNCTAD report?
[A] India
[B] China
[C] United States
[D] Singapore
Answer: United States
The United Nations Conference on Trade and Development (UNCTAD) recently released its ‘Global
Investment Trend Monitor report’ and listed the countries on the basis of receipts of Foreign Direct
Investment (FDI).
As per the report, the United States was the largest recipient of FDI in 2019 with inflow of investments
amounting to $251 billion. The second largest recipient was China at $140 billion in 2019. India was also
among the top 10 FDI recipients in 2019, which attracted investments worth $49 billion. This is 16%
higher than the previous year FDI inflow of the country. The report also stated that majority of India’s FDI
inflow went to its service sector.
8. The UN World Tourism Organization recently released its ‘UNWTO World Tourism Barometer report’.
Where is the head-quarters of UNWTO located?
[A] Paris
[B] Madrid
[C] Tokyo
[D] New York
Answer: Madrid
The UN World Tourism Organization (UNWTO) is the United Nations Agency for promoting sustainable
tourism, which is head-quartered at Madrid, Spain.
UNWTO recently released its comprehensive report on global tourism titled ‘UNWTO World Tourism
Barometer report’. According to the report, 1.5 billion international tourist arrivals were recorded in the
year 2019, around the globe. This is an increase of 4% from the previous year and this is also the tenth
consecutive year recording growth. It also predicted a growth of 3% to 4% in tourism sector, for the year
2020.
9. Who was recently appointed as the Managing Director and Chief Executive of Bank of Baroda?
[A] Challa Sreenivasulu Setty
[B] Rajnish Kumar
[C] Sanjiv Chadha
[D] Atanu Kumar Das
Answer: Sanjiv Chadha
The Appointments Committee of the Cabinet appointed Sanjiv Chadha as the Managing Director and
Chief Executive of the public sector bank, Bank of Baroda. Chadha was serving as the Deputy Managing
Director at SBI Capital Markets.
Another two key appointments were also made, which includes Lingam Venkat Prabhakar as MD & CEO
of Canara Bank and Atanu Kumar Das as MD & CEO of Bank of India. While Lingam Venkat Prabhakar
was the Executive Director of Punjab National Bank earlier , Atanu Kumar Das was Executive Director at

© 2020 GKToday | All Rights Reserved | https://www.gktoday.in 86


Current Affairs [PDF] - January 16-31, 2020

Bank of India who will be promoted to the post of MD& CEO.


10. Man Mohan Sood, who passed away recently, belonged to which field?
[A] Music
[B] Sports
[C] Literature
[D] Politics
Answer: Sports
The 80-year-old Former Indian cricketer and National Selector, Man Mohan Sood recently passed away.
Sood was born on 06-07-1939 in Lahore, Punjab (now in Pakistan), India. He had played one Test match
representing India and 39 first-class matches with a century. Man Mohan Sood also had served as an
administrator at the Delhi and District Cricket Association (DDCA) and a member of the National
Selecting Committee
Current Affairs Quiz: January 23, 2020
1. Which is the most polluted city in India, according to Greenpeace India’s recent report?
[A] Gurugram
[B] Jharia
[C] Lucknow
[D] Noida
Answer: Jharia
According to the recent report released by the environmental NGO Greenpeace India, Jharia, a city in
Jharkhand is the most polluted city in India, which is followed by Dhanbad of the same state.
The report also revealed that six of the country’s ten most polluted cities are in the state of Uttar
Pradesh. Lunglei city of Mizoram is the least polluted city and it is the only city with Particulate matter
(PM) levels under the World Health Organisation’s (WHO) prescribed level. Greenpeace India has used
data from the Central Pollution Control Board and analysis of particulate matter (PM) data of 287 cities,
to rank the most polluted cities in India
2. Which state is the first in the country to implement agricultural land leasing policy?
[A] Gujarat
[B] Tamil Nadu
[C] Uttarakhand
[D] West Bengal
Answer: Uttarakhand
The State government of Uttarakhand recently released a notification that it has made a policy to lease
agricultural land, becoming the first Indian state to implement such policy.
According to the policy, any institution, company or NGO can take farm land on lease for 30 years,
provided the leased land is below 30 acres. The farmers who own the land will receive the corresponding
rent during the leased period. Difficulties in leasing land for agriculture and plantation in hilly areas and
consolidation of fragmented land are being solved by the policy.
3. What is the name of the indigenous artillery guns, which are to be inducted into the Indian Army?
[A] Sharang
[B] Shrirang
[C] Sevak
[D] Senak
Answer: Sharang
© 2020 GKToday | All Rights Reserved | https://www.gktoday.in 87
Current Affairs [PDF] - January 16-31, 2020

As per the recent announcement from the Indian Army, the first batch of 18 indigenously upgraded
‘Sharang’ artillery guns are to be inducted into the Army by March 31, 2020.
The barrel of the indigenously designed gun has been upgraded from 130 mm to 155 mm. As a result,
the hitting range of the gun has been increased by 12 km kilometres, to 39 kilometres. The Gun Carriage
Factory (GCF) in Jabalpur had been awarded the global contract for upgrading the Sharang artillery gun,
and the project was done with the help of ordnance factories, Indian Army and DRDO.
4. Which defence Force of India has recently signed an MoU with Geological Survey of India?
[A] Indian Army
[B] Indian Navy
[C] Indian Air Force
[D] Indian Coast Guard
Answer: Indian Navy
A Memorandum of Understanding (MoU) was recently signed between Indian Navy and Geological
Survey of India (GSI). The agreement was on ‘Sharing of seabed sediments data, products and expertise
for Naval Application in Meteorology and Oceanography’.
The Marine and Coastal survey Division of GSI has mapped most of the country’s Exclusive Economic
Zone. This repository of data collected using the oceanic research vessels of GSI, will now be used by
the Indian Navy. The data would be used for oceanographic modelling during their maritime operations
in the Indian Ocean region (IOR).
5. The National Bravery Awards for children are presented by which ministry/ organisation annually?
[A] Ministry of Women and Child Development
[B] Ministry of Defence
[C] UNICEF
[D] Indian Council for Child Welfare
Answer: Indian Council for Child Welfare
The National Bravery awards for the year 2019 were recently presented by the Indian Council for Child
Welfare (ICCW) in New Delhi.22 children including 10 girls were selected for the National Bravery
awards. Among them, one award was given posthumously. The highest award called Bharat award was
conferred on a 15-year-old Kerala boy Adithya, for saving more than 40 lives after a bus accident.
Indian Council for Child Welfare was formed in the year 1952, which initiated this award to recognize
children for their brave and spontaneous selfless service or their daring act against a social crime.
6. Retired IAS Officer Yudvir Singh Malik has been recently appointed as the Chairman and Managing
director (CMD) of which crisis-hit firm?
[A] IL&FS
[B] DHFL
[C] Unitech
[D] Altico Capital
Answer: Unitech
Retired IAS Officer Yudvir Singh Malik has been recently appointed as the Chairman and Managing
director (CMD) of crisis- hit realty firm Unitech. He assumed office on January 21, 2020.
A few days back, the Supreme Court directed the Centre to take total management control of Unitech
and appoint a new board of nominee directors. As per the directions, the Centre has replaced the board
of the realty firm and asked the new board to come out with a resolution framework within two months.
The firm was reported of allegedly using flat-buyer’s money and default the delivery of flats.

© 2020 GKToday | All Rights Reserved | https://www.gktoday.in 88


Current Affairs [PDF] - January 16-31, 2020

7. Which Private sector bank launched ‘card-less cash withdrawal’ facility from its ATMs, recently?
[A] HDFC Bank
[B] Axis Bank
[C] ICICI Bank
[D] Kotak Mahindra Bank
Answer: ICICI Bank
India’s leading Private sector bank recently launched card-less cash withdrawal’ facility from its ATMs.
The facility will be available throughout the day, across all ICICI Bank ATMs, which count to nearly
15000.
For availing the card-less service, the customer needs to log into the ICICI mobile app – ‘iMobile’ and
generate a temporary PIN. The customer then can withdraw cash by entering the OTP received and the
PIN, without the need of carrying any card. The daily transaction limit and limit per transaction is
₹20,000.
8. As per the recent survey of Chief Executive Officers (CEOs) by consultancy firm PwC, which country has
the highest growth prospects?
[A] China
[B] India
[C] United States
[D] Germany
Answer: United States
The leading international consultancy firm PwC recently released the results of its annual survey of
Chief Executive Officers (CEOs) across the world. The firm surveyed more than 1,580 CEOs in 83
countries about global economy prospects. As per the survey, the United States has the highest growth
prospects, according to the CEOs.
China remained in the second place while India retained the position of the fourth rank. The report also
stated that 53 per cent of global CEOs believe international economic growth would decline in the next
12 months.
9. The recently set-up ‘National Start-up Advisory Council’ is to be chaired by which Union Minister?
[A] Minister of Micro, Small and Medium Enterprises
[B] Minister of Corporate Affairs
[C] Minister of Commerce and Industry
[D] Minister of Skill Development and Entrepreneurship
Answer: Minister of Commerce and Industry
The Union government recently announced that a National Start-up Advisory Council has been set up.
The Council will advise the Centre on steps to be taken to create a strong environment for start-up
companies in India.
The Council will be chaired by the Union Commerce minister while it will have non-official members with
2-year-term, to be nominated by the Centre. Other members include founders of successful start-ups and
veterans who have started and grown companies in India. The nominees of the ministries and
organisations concerned, not below the rank of joint secretary will be ex-officio members.
10. India’s first ever convention centre in Africa was recently inaugurated in which African country?
[A] Libya
[B] Ghana
[C] Niger

© 2020 GKToday | All Rights Reserved | https://www.gktoday.in 89


Current Affairs [PDF] - January 16-31, 2020

[D] Algeria
Answer: Niger
Indian External Affairs Minister Jai Shankar recently inaugurated the first ever convention centre of India
in Africa. The Mahatma Gandhi International Convention Centre (MGICC) was inaugurated in the
Western-African country Niger with the assistance from India.
The International Convention Centre was jointly inaugurated by President of Niger Mahamadou Issoufou.
This is also the first-ever visit by an Indian Foreign Minister to the West African country. India has
recently extended $15 million grant to Niger in support of organizing the African Union (AU) summit held
in Niger last year.
Current Affairs Quiz: January 24, 2020
1. Hassan Diab, who was seen in news recently, is the Prime Minister of which country?
[A] Israel
[B] Jordan
[C] Syria
[D] Lebanon
Answer: Lebanon
Lebanon formed a new government recently under Prime Minister Hassan Diab amidst various economic
and political crises. The country remained without an effective government, after the former Prime
Minister Saad Al Hariri quit from his role in October 2019 under pressure from protests against state
corruption and mismanagement.
The President of Lebanon, Michel Aoun nominated Hassan Diab as the new Prime Minister. Hassan Diab
is a 60-year-old professor at the American University of Beirut, who heads a new cabinet of 20 members.
The members were mostly backed by political parties while the protestors demanded a new government
with independent technocrats.
2. Which Indian tele-com service provider recently got approval from the telecom department to raise the
FDI limit to 100%?
[A] Jio
[B] Bharti Airtel
[C] Vodafone Idea
[D] BSNL
Answer: Bharti Airtel
Bharti Airtel recently got approval from the Department of Telecommunications (DoT) to increase the
FDI limit up to 100% of the paid-up capital from the earlier limit of 49%. The company already received
permission from the Reserve Bank of India (RBI) for overseas portfolio investors to invest up to 74% in
Bharti Airtel.
This DoT approval will help the tele-com company to source more funds from overseas investors.
Singapore Telecommunications Ltd or Singtel, which owns a 35% stake in Bharti Airtel is expected to
increase its stake in Bharti Airtel.
3. According to the recent notification issued by the Ministry of Law and Justice,India, which country was
declared as the ‘reciprocating territory’?
[A] United Arab Emirates
[B] Oman
[C] Saudi Arabia
[D] Yemen

© 2020 GKToday | All Rights Reserved | https://www.gktoday.in 90


Current Affairs [PDF] - January 16-31, 2020

Answer: United Arab Emirates


The Ministry of Law and Justice recently issued a Gazette Notification, which declared the United Arab
Emirates to be a “reciprocating territory” under Section 44A of the Civil Procedure Code, 1908. The
notification also declared a list of courts in the UAE to be ‘Superior Courts’ under the same section of
CPC.
The orders passed by certain designated courts from a ‘reciprocating territory’ can be implemented in
India, as if they were passed in India. Besides UAE, the other countries which were declared as
reciprocating territories include United Kingdom, Singapore, Bangladesh, Malaysia and New Zealand
among others. The decision is believed to help bring down the time required for executing verdicts
between the two countries.
4. What is India’s rank in the Carbon Disclosure Project (CDP) India annual report?
[A] Fifth
[B] Sixth
[C] Seventh
[D] Eighth
Answer: Fifth
Carbon Disclosure Project-India (CDP), an environmental non-profit organisation released its annual
report of 2018, titled “Corporates #StepUp Climate Action”. As per the report, 25 companies committed
to science-based targets in alignment with the Paris Agreement by December 2018. This has earned the
5th rank to India, which is only after the US, Japan, Britain and France in corporate climate action.
The report examines carbon reduction activities of companies and revealed that 58 Indian companies of
the 59 firms surveyed between 2018 and 2019, have board-level oversight of climate-related issues.
5. Which Indian actor was recently honoured with the Crystal Award of the World Economic Forum?
[A] Priyanka Chopra
[B] Deepika Padukone
[C] Kareena Kapoor
[D] Anushka Sharma
Answer: Deepika Padukone
Famous Bollywood actor Deepika Padukone received the Crystal Award during the 50th Annual meeting
of the World Economic Forum in Davos, Switzerland. The award was presented to her in recognition of
her efforts to create awareness about the importance of mental health.
Hilde Schwab, Chairwoman and Co-Founder of the World Economic Forum’s World Arts Forum presented
the award to the actor. Deepika Padukone runs a foundation named ‘Live Love Laugh’, which works
towards conducting awareness programs in Indian schools. It also funds for free psychiatric treatment
and medical education programs.
6. Which state government has recently launched the ‘Mukhyamantri Krishak Durghatna Kalyan Yojana’?
[A] Gujarat
[B] Uttar Pradesh
[C] Maharashtra
[D] Andhra Pradesh
Answer: Uttar Pradesh
The state government of Uttar Pradesh Government has recently launched Mukhyamantri Krishak
Durghatna Kalyan Yojana. Under the scheme financial assistance of Rs. 5 Lakh will be provided to the
family of farmers on their death or occurrence of physical disability while working. The farmers in the

© 2020 GKToday | All Rights Reserved | https://www.gktoday.in 91


Current Affairs [PDF] - January 16-31, 2020

age group of 18 to 70 are covered under the scheme.


The assistance program will also cover the people who work in the fields of other owners and receive
the crops after harvesting. The state has also decided to implement Mukhyamantri Paryatan
Samvardhan Yojana to boost tourism in state.
7. What is the name of the humanoid robot, which was recently unveiled by the Indian Space Research
Organisation (ISRO)?
[A] Gagan Mitra
[B] Vyom Mitra
[C] Vayu Mitra
[D] Human Mitra
Answer: Vyom Mitra
The Indian Space Research Organisation (ISRO) recently unveiled the humanoid robot, called Vyom Mitra
at the inaugural session of the conference “Human Spaceflight and Exploration — Present Challenges
and Future Trends”, held in Bengaluru.
This humanoid with feminine features is to be launched into the space ahead of the first manned space
mission of India. The humanoid will simulate the human functions required for space during its test
flight. It can perform all crew functions like switch-panel operations, environmental control and life-
support system. ISRO has scheduled two trail flights with humanoids in December 2020 and July 2021.
8. What is the capital of the newly formed Union Territory of Dadra and Nagar Haveli and Daman and Diu?
[A] Daman
[B] Nagar Haveli
[C] Dadra
[D] Silvassa
Answer: Daman
Union Cabinet, chaired by Prime Minister Narendra Modi recently announced that the city of Daman will
be the head-quarters of the newly constituted Union Territory of Dadra and Nagar Haveli and Daman and
Diu. Daman is a city in the erstwhile UT Daman & Diu, located on the west coast of India.
Two separate Union Territories of India, Daman & Diu and Dadra & Nagar Haveli were merged to a single
Union Territory of Dadra and Nagar Haveli and Daman and Diu on December 3, 2019. Silvassa was the
previous capital of the erstwhile UT Dadra and Nagar Haveli.
9. Indian Oil Corporation Ltd (IOCL) of India recently signed an MoU to assist the Petroleum Authority of
which country?
[A] Ghana
[B] Argentina
[C] Brazil
[D] Peru
Answer: Ghana
A memorandum of understanding (MoU) was recently signed between the Indian Oil Corporation Ltd
(IOCL) and National Petroleum Authority (NPA) of Ghana to expand the country’s Liquified petroleum
gas (LPG) network.
Indian Oil will provide assistance and share its technical expertise with the Petroleum authorities of the
African country to implement the National LPG Promotion Policy of Ghana. The announcement was
made by the Ministry of Petroleum and natural gas. Indian Oil is also expected to assist Ghana in areas
of infrastructure development for the new LPG Value chain.

© 2020 GKToday | All Rights Reserved | https://www.gktoday.in 92


Current Affairs [PDF] - January 16-31, 2020

10. Union Minister for Petroleum & Natural Gas and Steel has recently launched a volunteer scheme
‘SERVICE’ for the employees of which Indian PSU?
[A] Coal India Limited (CIL)
[B] Steel Authority of India Limited (SAIL)
[C] National Thermal Power Corporation (NTPC)
[D] Bharat Heavy Electricals Limited (BHEL)
Answer: Steel Authority of India Limited (SAIL)
Union Minister for Petroleum & Natural Gas and Steel has recently launched a volunteer scheme
‘SERVICE’ for the employees of the Indian PSU Steel Authority of India Limited (SAIL).
The scheme called, SAIL Employees Rendering Volunteerism & Initiatives for Community Engagement
(SERVICE), will be made operational from the foundation day of the company, which is on January 24.
Under the scheme, the employees are encouraged to voluntarily indulge themselves in social activities,
with main focus on education, health and women empowerment.
Current Affairs Quiz: January 25, 2020
1. When is the International Day of Education observed every year ?
[A] January 21
[B] January 24
[C] February 21
[D] February 24
Answer: January 24
January 24 is observed as the International Day of Education every year by UNESCO. It celebrates the
role of education for Development and Peace. The theme for this year’s International Day of Education is
“LEARNING FOR PEOPLE, PLANET, PROSPERITY AND PEACE”. On December 2018, the United Nations
(UN) General Assembly adopted a resolution, which declared January 24 as ‘International Day of
Education’.
This year’s celebration is the second-year of observing the day and it has considered Education and
Learning as the greatest Renewable Resource and it reaffirms that the Education is a Fundamental
Right.
2. Ministry of Rural Development (MoRD) has recently signed an MoU with which of the following
organizations?
[A] World Bank
[B] Bill and Melinda Gates Foundation
[C] Asian Development Bank
[D] BRICS Bank
Answer: Bill and Melinda Gates Foundation
An MoU has been signed by the Ministry of Rural Development (MoRD) with Bill and Melinda Gates
Foundation (BMGF). This MoU is under the Deendayal Antyodaya Yojana – National Rural Livelihoods
Mission (DAY-NRLM). The MoU aims to strengthen the rural poor institutions at grass root level. DAY
NRLM and BMGF has a shared focus on improving the lives of poor people and the marginalized, by
creating employment opportunities through rural institutions.
DAY NRLM was launched by the MoRD IN June 2011. The scheme has an agenda of covering 7 crore
rural population in the country through self-managed Self-Help Groups (SHGs) and other such
institutions in a period of 8 to 10 years.
3. Who has been recently awarded with the Subash Chandra Bose Aapda Prabandhan Puraskar, 2020?

© 2020 GKToday | All Rights Reserved | https://www.gktoday.in 93


Current Affairs [PDF] - January 16-31, 2020

[A] Vijay Kumar


[B] Sylendra Babu
[C] Kumar Munnan Singh
[D] Naveen Kumar
Answer: Kumar Munnan Singh
Kumar Munnan Singh has been selected for the Subhash Chandra Bose Aapda Prabandhan Puraskar
2020 under the Individual category. Under the Institution category, Disaster Mitigation & Management
Centre, Uttarakhand has been selected for the award. Kumar Munnan Singh was the founding member of
the National Disaster Management Authority in 2005. He is also lauded for establishing the National
Disaster Response Force (NDRF), from scratch.
The award is given by the Government of India to recognize the commendable work done by individuals
and organisations in the field of disaster management. The award is announced every year on 23rd
January which marks the birth anniversary of Netaji Subhash Chandra Bose.
4. Which state has recently made reading of preamble to constitution as a mandatory practice in schools?
[A] Uttar Pradesh
[B] Madhya Pradesh
[C] Gujarat
[D] Maharashtra
Answer: Madhya Pradesh
As per an order issued by the State Education Department of Madhya Pradesh, the state government has
made the reading of the Preamble to the Constitution in schools mandatory. As per the order, the
Preamble would be read out every Saturday, by the head master or the teacher in all the primary, middle,
high and higher secondary schools after prayer, along with the students. It is said that such practice in
school would inculcate the interest in our constitution among the students.
Maharashtra: Maharashtra has also has made reading of Preamble of the Constitution in all primary and
secondary schools, starting from January 26, 2020. The resolution in this direction was adopted in 2013
but was not implemented so far.
5. Which state’s Tourism department has received the special mention honour as “Emerging Global
Destination” in UN World Tourism Organisation Global Destination Award 2019?
[A] Uttarakhand
[B] Manipur
[C] Assam
[D] Kerala
Answer: Kerala
The Barrier Free Tourism Project implemented by Kerala Tourism in Trissur District has received the
special mention honour as “Emerging Global Destination” in UN World Tourism Organisation Global
Destination Award 2019.
In March 2019, the Kerala Tourism has launched the barrier free project to make 120 destinations in
Kerala disabled friendly. 80 such projects have been implemented and 8 of them are in the district of
Trissur. Kerala is the first state to work as per the guidelines issued by UN WTO, on making the
destinations disabled friendly.
6. What is the name of the award recently presented by the Indian President to children, in the fields of
innovation, social service, sports, art and culture and bravery?
[A] Janani Puraskar

© 2020 GKToday | All Rights Reserved | https://www.gktoday.in 94


Current Affairs [PDF] - January 16-31, 2020

[B] Shakti Puraskar


[C] Bal Shakti Puraskar
[D] Bal Puraskar
Answer: Bal Shakti Puraskar
The Bal Shakti Puraskar award is given to children in the fields of innovation, social service, scholastic
activities, sports, art and culture and bravery. The Indian President Ramnath Kovind recently presented
the awards to 49 children in the age group of 5-18 years.
Pradhan Mantri Rashtriya Bal Puraskar award was instituted by Ministry of Women and Child
Development and is given under two categories. Bal Shakti Puraskar is awarded to meritorious children
while Bal Kalyan Puraskar is awarded to individuals and institutions, who have contributed for child
welfare.
7. What is the current investment limit for Foreign portfolio investors in government and corporate bonds,
after the recent directions of RBI?
[A] 20%
[B] 25%
[C] 30%
[D] 35%
Answer: 30%
The Reserve Bank of India (RBI) recently increased the investment limit for FPIs in government and
corporate bonds from 20 per cent to 30 per cent.
As of now, the Foreign portfolio investors (FPI) could not invest more than 20 per cent of their total
investment in either central government securities or state development loans or corporate bonds. This
relaxation is expected to attract more investments and increase the inflow of funds into the Indian
market. FPIs are allowed to invest in exchange-traded funds that invest only in debt instruments.
8. Which organisation is set to develop the National Data and Analytics Platform (NDAP)?
[A] Ministry of Statistics and Program Implementation
[B] NITI Aayog
[C] Finance Commission
[D] National Development Council
Answer: NITI Aayog
The Think tank of India, NITI Aayog is to develop the National Data and Analytics Platform (NDAP).
Recently the Vice Chairman of NITI Aayog released the Vision document for the National Data and
Analytics Platform.
The stake holders can access all government data through the platform, in a user-friendly manner. The
platform will comprise of the latest data from various government websites. It will also host tools for
analysis of the data and different types of visualisation. The first version of NDAP will be released in
2021. A Technical Advisory Group (TAG) will also be set up with experts in the field, who guide in the
development and management of the platform.
9. India recently handed over the consignment of Measles and Rubella vaccines to which country, after a
suspected outbreak?
[A] Sri Lanka
[B] Maldives
[C] Bangladesh
[D] Nepal

© 2020 GKToday | All Rights Reserved | https://www.gktoday.in 95


Current Affairs [PDF] - January 16-31, 2020

Answer: Maldives
More than thirty thousand doses of Measles and Rubella vaccines to the Maldives to control the
outbreak of Measles in the island country.
Though Maldives has eliminated the spread of Measles, the government in Male suspected an outbreak
of the disease after it detected the case. The country made an emergency request to India, for the
vaccines, which was handed over within three days of the request. Maldives had earlier approached
Denmark and UNICEF for the vaccines. It is also said that four people of Maldives tested positive for the
disease in the past one week.
10. World Holocaust Forum was recently convened in which city, to remember Holocaust and counter anti-
Semitism?
[A] Moscow
[B] Jerusalem
[C] Paris
[D] Berlin
Answer: Jerusalem
World Holocaust Forum was recently convened in the capital city of Israel, Jerusalem. The forum
marked the 75th anniversary of the liberation of the Auschwitz death camp.
More than 40 global leaders attended the forum, which includes the Russian President Vladimir Putin, US
Vice-President Mike Pence, Presidents of France and Germany and the Prince of Wales. The forum was
organised by the World Holocaust Forum Foundation, which is an international organisation found to
preserve the memory of the Holocaust and to promote tolerance between all religions and nationalities.
Current Affairs Quiz: January 26-27, 2020
1. The first edition of Ganga-Volga dialogue was recently held in New Delhi, to enhance the bilateral co-
operation between India and which country?
[A] China
[B] Russia
[C] Nepal
[D] Bangladesh
Answer: Russia
The inaugural edition of the Ganga Volga Dialogue was recently held in New Delhi, the idea of which was
originated during the India-Russia Summit between President Putin and Prime Minister Modi, held in
October 2018.
The Dialogue was organised by the Union Ministry of External Affairs, India in association with Dr.
Syama Prasad Mookerjee Research Foundation. Several scholars of India and Russia participated in the
event and had discussions in the context of India-Russia bilateral cooperation, with the main focus on
culture and civilization.
2. The International Summit on Women in STEM– “Visualizing the Future: New Skylines”, was recently held
in which city?
[A] Mumbai
[B] New Delhi
[C] Hyderabad
[D] Kolkata
Answer: New Delhi
The Department of Biotechnology, under the Ministry of Science and Technology organized an

© 2020 GKToday | All Rights Reserved | https://www.gktoday.in 96


Current Affairs [PDF] - January 16-31, 2020

International Summit on Women in STEM (Science, Technology, Engineering and Mathematics) –


“Visualizing the Future: New Skylines” at India Habitat centre, New Delhi.
The Summit was organized to encourage the participation of women in STEM field and for developing
their career in the field. Renowned women scientists from different countries, who pursue their career in
the STEM field addressed the participants and motivated them to pursue their scientific career.
3. What is the theme of the National Voters’ Day celebrated across the country?
[A] Electoral Literacy for Stronger Democracy
[B] Voting Rights For All
[C] Ethical Voting
[D] Sab Se Pehle Voting
Answer: Electoral Literacy for Stronger Democracy
National Voters’ Day is being celebrated across the country on January 25. It was first celebrated on
January 25, 2011, to commemorate the foundation day of the Election Commission of India (ECI). Which
was established on the same day in 1950.
The National Voters’ day of this year marks the tenth year of celebration and the 70th year of functioning
of the Election Commission of India. The theme for the National Voters’ Day, 2020 is ‘Electoral Literacy
for Stronger Democracy’. Indian President Ram Nath Kovind will be the Chief Guest of the celebration
organised at New Delhi, by the ECI.
4. India signed and MoU with the World Bank for the Agri-business and Rural Transformation Project of
which state?
[A] Gujarat
[B] Maharashtra
[C] Madhya Pradesh
[D] Haryana
Answer: Maharashtra
The Government of India, the state government of Maharashtra and the World Bank signed a loan
agreement of $210 million for the Agri-business and Rural Transformation Project of the state.
The fund will help small and marginal farmers to increase their market access and assist them to
participate in the value chains. The farmers are encouraged to adopt climate resilient production
techniques in agriculture. The project will be implemented in all thirty-six districts of the state of
Maharashtra and nearly one million farming households are expected to benefit from the project.
5. Which Indian Private sector bank has recently launched its wealth management platform called ‘Pioneer
Banking’?
[A] ICICI Bank
[B] HDFC Bank
[C] Axis Bank
[D] IndusInd Bank
Answer: IndusInd Bank
Leading Private Sector bank IndusInd bank has launched its wealth management platform called
‘Pioneer Banking’, which serves the banking needs of its high net-worth customers.
The new service of wealth management recently introduced is expected to be merged with the other set
of personal and commercial banking products offered by the bank to its customers. The new service
mainly focuses the high net-worth individual (HNI) customers and provides various benefits including
Free doorstep banking up to Rs 4 lakh per day and Services of dedicated wealth managers and

© 2020 GKToday | All Rights Reserved | https://www.gktoday.in 97


Current Affairs [PDF] - January 16-31, 2020

investment specialists.
6. Which steel plant of India was recently awarded at the WEF Annual meet, for being included in the WEF’s
Lighthouse Network?
[A] Salem Steel Plant
[B] Tata Steel Kalinganagar
[C] Vijayanagar Steel Plant
[D] Vishakhapatnam Steel Plant
Answer: Tata Steel Kalinganagar
Tata Steel Kalinganagar was recently awarded at the WEF Annual meet, for being included in the WEF’s
Lighthouse Network.
In July 2019, Tata Steel Kalinganagar became the first and only Indian manufacturing plant to be
included in the coveted ‘Lighthouse Network’ of the World Economic Forum. Other ten manufacturing
plants from around the world were also included in the network. Tata Steel’s IJmuiden plant located in
the Netherlands has already been included in the network.
7. The Global Consortium for Digital Currency Governance was recently announced by which organisation?
[A] World Bank
[B] World Economic Forum
[C] International Monetary Fund
[D] World Trade Organisation
Answer: World Economic Forum
The first Global Consortium which focussed on creating a framework for Governance of Digital
Currencies was recently announced by the World Economic Forum. The announcement was made during
the last day of the 50th annual meeting of WEF, held in Davos, Switzerland.
This initiative is the first-of-its-kind to create a global platform for financial institutions, government
representatives and technical experts in the field. The consortium is announced with the objective of
increasing access to the financial system through inclusive policies. The Governance of Digital
currencies, including cryptocurrencies and stable-coins would also be focussed.
8. Jair Messias Bolsonaro, who has arrived India to attend the Republic Day celebrations as chief guest, is
the President of which country?
[A] Argentina
[B] Brazil
[C] France
[D] Germany
Answer: Brazil
The President of Brazil, Jair Messias Bolsonaro recently arrived India along with his ministers,
parliamentarians and top businessmen, to grace the Indian Republic Day celebrations as the Chief guest.
India and Brazil are expected to sign as many as 15 agreements to enhance bilateral cooperation in the
fields of like oil and gas, mining and cyber security. Agreements on social security, on mutual legal
assistance in criminal matters on cooperation in bio-energy are also expected to be inked between the
countries. Bolsonaro became the President of Brazil in October 2018 and this is his first visit to India.
9. The International Children’s Film festival was recently inaugurated in which South Asian country, with the
theme ‘Under The World’?
[A] Sri Lanka
[B] Bangladesh

© 2020 GKToday | All Rights Reserved | https://www.gktoday.in 98


Current Affairs [PDF] - January 16-31, 2020

[C] Nepal
[D] India
Answer: Bangladesh
The International Children’s Film festival was recently inaugurated in the capital city of Bangladesh,
Dhaka. It is one of the biggest film festivals in the country and the only international festival which is
celebrated exclusively for children and young adults since 2008.
The festival is organized by the Children’s Film Society of Bangladesh and will feature 173 films from 39
countries. The 7 day-long festival is set to conclude on 31st of January. To mark the 100th birth
anniversary of the country’s founding President Sheikh Mujibur Rahman, ‘Khoka Jokhon Chhoto Chhilen’
movie was screened. The theme of this year’s festival is ‘Under The World’.
10. India has signed an agreement with which country to upgrade Ashuganj-Akhaura road into 4-lane
highway?
[A] Myanmar
[B] Bangladesh
[C] Sri Lanka
[D] Nepal
Answer: Bangladesh
India has recently signed an agreement with Bangladesh to upgrade 50.58 km road between Ashuganj
river port and Akhaura in Bangladesh into 4-lane highway. This is the part of $2 billion Line of Credit
extended by India to Bangladesh in 2016.
Out of the 50.58 km, a stretch of 39 km would be upgraded by Roads and Highways Department (RHD)
of Bangladesh and an Indian Company – Afcons Infrastructure Limited. The RHD and an India-
Bangladesh joint venture firm will take up the work of upgrading the remaining part of the road. The
project is expected to improve the road connectivity between the two countries and facilitate regional
connectivity between South and South-east Asian countries.
Current Affairs Quiz: January 28, 2020
1. Which legendary Indian sportsperson was awarded with the prestigious ‘Padma Vibhushan’ award
recently?
[A] Sachin Tendulkar
[B] Dhanraj Pillai
[C] M C Mary Kom
[D] Abhinav Bindra
Answer: M C Mary Kom
Six-time World Champion and Olympian boxer Mary Kom was conferred with the prestigious ‘Padma
Vibhushan’ award on the Republic day of the year 2020.
Seven eminent personalities were presented the ‘Padma Vibhushan’ award, which is the second highest
civilian award in India, only next to the ‘Bharat Ratna’. Other awardees are Former Union Ministers Arun
Jaitley, Sushma Swaraj and George Fernandes (posthumously), Singer Chhannulal Mishra of Uttar
Pradesh and former Mauritius prime minister Anerood Gugnauth.
2. How many personalities were selected for the ‘Padma Bhushan’ award, on the republic day 2020?
[A] 14
[B] 15
[C] 16
[D] 17

© 2020 GKToday | All Rights Reserved | https://www.gktoday.in 99


Current Affairs [PDF] - January 16-31, 2020

Answer: 16
Sixteen eminent personalities were selected for the ‘Padma Bhushan’ award, for their distinguished
service of high order. Out of the 141 total Padma awardees selected this year, there were seven Padma
Vibhushan, 16 Padma Bhushan and 118 Padma Shri Awardees.
Famous personalities who were selected for the ‘Padma Bhushan’ award includes, top-rated Badminton
Player PV Sindhu, Former Chief Minister of Goa- Manohar Parrikar (posthumous), father of modern legal
education in India, N R Madhava Menon and businessmen Anand Mahindra and Venu Srinivasan.
3. ‘The National Tourism Conference’ of the year 2020 was recently organised in which Indian city?
[A] Varanasi
[B] Konark
[C] Shimla
[D] Leh
Answer: Konark
The National Tourism Conference was recently hosted in the city of Konark, Odisha. The two-day
conference was jointly organized by the state government of Odisha and The Federation of Indian
Chambers of Commerce and Industry (FICCI).
Union Tourism Minister Prahalad Singh Patel attended the conference and announced that the Ministry
of tourism would fund the travel expenditure of the tourists, who visit 15 places in the country in a year.
Such tourists have to travel outside their home states and submit the photos on the ministry’s website.
A Memorandum of Intent (MoI) was signed between Odisha and Gujarat for cross-promotion of tourist
sites.
4. Mark Rutte, who was seen in news recently, is the Prime Minister of which country?
[A] Germany
[B] Netherlands
[C] France
[D] Italy
Answer: Netherlands
The Prime Minister of Netherlands Mark Rutte was recently seen in news, as he made the country’s first
apology for the persecution of Jews during the World War II.
He made this historic apology, first-of-its kind made by any government in this regard, during the
country’s annual Holocaust commemoration in Amsterdam, on the eve of the 75th anniversary of the
liberation of the Nazi death camp at Auschwitz. More than 1.1 million Jews out of the 1.4 million who
lived in Netherlands were killed in the Auschwitz concentration camp, at the outbreak of World War II.
5. Which city plays host to the annual event ‘Bharat Parv, 2020’ which celebrates the spirit of India?
[A] Mumbai
[B] New Delhi
[C] Ahmedabad
[D] Jaipur
Answer: New Delhi
The annual event ‘Bharat Parv, 2020’ is being celebrated from 26th to 31st January, 2020 in New Delhi.
The aim of the festival is to encourage Indians to visit different tourism destinations of India and to
encourage them with the spirit of ‘Dekho Apna Desh’.
The theme of this year’s festival is set as ‘Ek Bharat Shreshtha Bharat’ and ‘Celebrating 150 Years of
Mahatma Gandhi’. The six-day-event which is to held in front of Red Fort at Gyan Path and Red Fort

© 2020 GKToday | All Rights Reserved | https://www.gktoday.in 100


Current Affairs [PDF] - January 16-31, 2020

grounds in New Delhi has several attractions including performance by armed forces bands cultural
Performances from different regions of the country.
6. National Institute of Rural Development and Panchayati Raj (NIRDPR) has recently partnered with which
global organisation to establish a ‘Communication Resource Unit’?
[A] World Bank
[B] UNICEF
[C] UNESCO
[D] International Monetary Fund
Answer: UNICEF
National Institute of Rural Development and Panchayati Raj (NIRDPR), an autonomous organisation
under the Union Ministry of Rural Development, has recently partnered with UNICEF (United Nations
International Children’s Emergency Fund) to establish a ‘Communication Resource Unit’.
The unit will act as a mechanism to provide communication support to national flagship programmes
including POSHAN Abhiyaan, National Health Mission and Swachh Bharath Mission. The Communication
Resource Unit is currently operating in three states of Andhra Pradesh, Telangana and Karnataka, to
furnish communication support including capacity building and consultation, to different government
departments.
7. India’s first and biggest walk-through aviary with over 100 species of birds, was recently inaugurated in
which city?
[A] Mumbai
[B] Guwahati
[C] Bengaluru
[D] Thiruvananthapuram
Answer: Mumbai
The Chief Minister of Maharashtra, Uddhav Thackeray inaugurated the country’s first and biggest walk-
through aviary at the Veermata Jijabai Bhosale Udyan and Zoo commonly called as the Byculla zoo in
Mumbai.
The aviary comprises of an area of over 18,200 square feet and height of 44 feet. It will be home to over
100 different species of exotic and common birds, which will live in their natural environment. A small
bridge passes through the aviary which will be opened for the public to walk through and view the birds.
8. What is the name of the national award, that is given to a person for meritorious act of human nature in
saving the life of a person?
[A] Jeevan Raksha Padak
[B] Raksha Puraskar
[C] Jeevan Shakti Padak
[D] Bharat Jeevan Padak
Answer: Jeevan Raksha Padak
President Ram Nath Kovind has approved to confer the Jeevan Raksha Padak Series of Awards, 2019 to
54 persons. This includes Sarvottam Jeevan Raksha Padak to 7 persons, Uttam Jeevan Raksha Padak to
8 and Jeevan Raksha Padak to 39 persons. Five awards are to be given posthumously.
Jeevan Raksha Padak awards, a branch of the Ashoka Chakra series of Gallantry awards, were instituted
in the year 1961. The award is given for meritorious act of human nature in saving the life of a person in
incidents like accidents, electrocution, natural calamities or rescue operation in mines.
9. ‘Shiv Bhojan’ scheme, which is aimed at providing a meal to the poor at subsidised rates, was recently

© 2020 GKToday | All Rights Reserved | https://www.gktoday.in 101


Current Affairs [PDF] - January 16-31, 2020

launched in which state?


[A] Uttar Pradesh
[B] Bihar
[C] Maharashtra
[D] Karnataka
Answer: Maharashtra
The state government of Maharashtra launched the ‘ Shiv Bhojan’ scheme, which aims to provide a meal
to the poor for Rs 10. The scheme was launched on the occasion of the 71st Republic Day celebrations.
Under the scheme, thalis or lunch plates would be available to the people at designated centres in all
districts. Nearly 50 ‘Shiv-Bhojan’ outlets were set up in a pilot basis, which will be extended to other
parts of the state. The remaining fund needed for providing food will be given as grant to the district
collectorate by the state government.
10. Sher Singh Kukkal, who passed away recently, was a famous personality in which field?
[A] Politics
[B] Sports
[C] Art
[D] Business
Answer: Art
The 86-year-old veteran artist and sculptor Sher Singh Kukkal passed away after age-related illness in
New Delhi recently.
Sher Singh Kukkal was a noted artist with a great experience in several branches of fine arts including
drawing, painting, sculpture, printing and photography. He also taught arts at the Sainik School Satara.
The artist had won several national and international awards including the Rajiv Gandhi Foundation
Award and the Senior Fellowship in Visual Art Photography by Union Ministry of Culture during 2008-09.
Current Affairs Quiz: January 29, 2020
1. The Government of India and the state government of Assam recently signed a tripartite agreement with
which tribal group?
[A] Bodo
[B] Kuki
[C] Adi
[D] Nishi
Answer: Bodo
The Centre and the state government of Assam recently signed a tripartite agreement with National
Democratic Front of Bodoland (NDFB). The Bodo tribal group was demanding a separate state of
Bodoland from Assam for several years.
The All Bodo Students’ Union (ABSU) and another group called United Bodo People’s Organisation, which
led the insurgency, also signed the agreement. More than 1500 militants of NDFB are expected to
surrender with arms. Rs.1500 crore has been sanctioned by the government for implementing the accord
in next three years.
2. Which state assembly recently passed a resolution to abolish the state’s legislative council?
[A] Telangana
[B] Maharashtra
[C] Andhra Pradesh
[D] Karnataka

© 2020 GKToday | All Rights Reserved | https://www.gktoday.in 102


Current Affairs [PDF] - January 16-31, 2020

Answer: Andhra Pradesh


The state assembly of Andhra Pradesh recently passed a resolution to abolish the state’s legislative
council. The decision was supported by 133 legislators of the ruling YSR Congress Party.
This decision is taken at the backdrop of the constant opposition faced in the upper house of the
legislature, the Andhra Pradesh Legislative council. The main opposition party, Telugu Desam Party led
by Chandrababu Naidu has 27 seats out of 58 in the council. It recently sent two bills related to the
decentralization of Andhra Pradesh’s capital to a select committee for further deliberation, after it was
passed in the assembly.
3. What is the planned percentage of the government’s stake in Air India to be divested?
[A] 50%
[B] 60%
[C] 75%
[D] 100%
Answer: 100%
According to the recent bid document released by the Government, it would sell all its 100 per cent of its
equity share capital in Air India, which includes the Air India’s 100 per cent stake in AI Express Limited
and 50 per cent in Air India SATS Airport Services Private Limited.
The government has also announced that it would absorb 30 per cent more in debt and liabilities, than it
did in the previous year. March 17 has been set as the dead line for the submission of bids for the sale,
which is to be implemented through the open bidding route. The net worth for the interested entities is
also reduced to Rs.3500 Cr from the earlier limit of Rs.5000 Cr.
4. Which Indian city plays host to the third Global Potato Conclave of 2020?
[A] Mumbai
[B] Ahmedabad
[C] Gandhinagar
[D] Surat
Answer: Gandhinagar
The third Global Potato Conclave recently commenced in Gandhinagar, with the Indian Prime Minister
Narendra Modi’s address through video-conferencing.
Two Global Potato Conferences were organised in 1999 and 2008, one in every 10 years. The conclave
for this decade is being organised by the Indian Potato Association (IPA) in association with the Indian
Council of Agricultural Research (ICAR), Shimla based ICAR-Central Potato Research Institute and
International Potato Centre (CIP), Peru. The three-day global event will showcase the achievements in
potato research and to set a blueprint for the decade.
5. Which Indian ship-builder is to deliver the anti-submarine stealth warship INS Kavaratti to Indian Navy?
[A] Mazagon Dock Limited
[B] Cochin Shipyard Limited
[C] Hindustan Shipyard Limited
[D] Garden Reach Shipbuilders and Engineers
Answer: Garden Reach Shipbuilders and Engineers
Kolkata-based Defence PSU- Garden Reach Shipbuilders and Engineers is to deliver the anti-submarine
stealth warship, INS Kavaratti to the Indian Navy soon, as per the recent announcement.
Kavaratti is the last of the four Anti-Submarine Warfare (ASW) stealth corvettes built by the GRSE for the
Indian Navy under Project P28, the first three :’Kamorta’, ‘Kadmatt’ and ‘Kiltan’, named after islands in

© 2020 GKToday | All Rights Reserved | https://www.gktoday.in 103


Current Affairs [PDF] - January 16-31, 2020

the Lakshadweep archipelago, were delivered from 2014 to 2017. The new Kavaratti warship has 90 per
cent indigenous components and is expected to require lower maintenance. With all the trials being
completed, the ship is to be delivered by the month end.
6. The world’s smallest gold coin, which features Albert Einstein, is recently released by which country?
[A] USA
[B] Switzerland
[C] Germany
[D] France
Answer: Switzerland
Switzerland has recently minted the world’s smallest gold coin, which bears the famous picture of the
great Scientist Albert Einstein with his tongue out. The government-run Swissmint announced that the
world’s smallest coin has a diameter of 2.96-millimeter and weighs 0.063 grams. The value of the coin is
pegged at 1/4 Swiss francs or approximately Rs 18.50.
Einstein came to Switzerland at the age of 17 and completed his education in the country. He also
became a Swiss citizen in 1902. Another coin featuring the Swiss tennis star Roger Federer was also
minted.
7. Marjan Sarec, who was seen in news after his resignation, was the Prime Minister of which country?
[A] Slovenia
[B] Slovakia
[C] Serbia
[D] Croatia
Answer: Slovenia
The Prime Minister of Slovenia, Marjan Sarec was seen in news as he recently submitted his resignation
from the post to the country’s parliament. He also called for an early election as his minority government
could not pass important legislations. The next regular election is due to be held in the middle of 2022.
The ruling coalition of five parties, which assumed power in Late 2018, had only 43 out of 90 seats in
parliament. Before the announcement of the Prime Minister, the Finance Minister Andrej Bertoncelj also
submitted his resignation.
8. Who has been appointed as the chief executive of Indian Bank’s Association (IBA)?
[A] Rajnish Kumar
[B] V G Kannan
[C] Sunil Mehta
[D] Rakesh Sharma
Answer: Sunil Mehta
As per the recent announcement from the Indian Bank’s Association (IBA), Former Managing Director
and CEO of Punjab National Bank, Sunil Mehta has taken charge as the Chief Executive of IBA.
Sunil Mehta has recently retired as the MD and CEO of Punjab National Bank. He has previously held the
post of Chairman of Indian Banks Association, which was replaced by the head of State Bank of India,
Rajnish Kumar in October 2019. Now, Sunil Mehta has replaced VG Kannan, who assumed office on
December 31, 2019.
9. Which Indian theatre artist is to be conferred the prestigious French honour, Knight of the Order of Arts
and Letters?
[A] Naseeruddin Shah
[B] Sanjana Kapoor

© 2020 GKToday | All Rights Reserved | https://www.gktoday.in 104


Current Affairs [PDF] - January 16-31, 2020

[C] Girish Karnad


[D] Lillette Dubey
Answer: Sanjana Kapoor
Veteran theatre personality and the daughter of Bollywood star Shashi Kapoor, artist Sanjna Kapoor is to
be presented with the French honour, Knight of the Order of Arts and Letters. The award is conferred on
her for the outstanding contribution the artist has made to the theatre arts.
The Culture Minister of France, Franck Riester, who is currently on an official visit to India, is expected to
confer the prestigious award on Sanjna Kapoor in a proposed ceremony. Sanjana Kapoor acted in
several plays and revived the Prithvi Theatre in Mumbai, which was founded by her parents in tribute to
Prithviraj Kapoor. She also co-founded an organisation called Junoon, which increases the reach of
theatre and the arts.
10. Which Indian squash player became the runner-up in the Pittsburgh Open squash tournament?
[A] Joshna Chinappa
[B] Dipika Pallikal
[C] Saurav Ghosal
[D] Sunayna Kuruvilla
Answer: Saurav Ghosal
Ace Indian squash player Saurav Ghosal became the runner-up in the Pittsburgh Open squash
tournament after he lost to the top seed Egyptian player, Fares Dessouky in the final.
The second seed Saurav Ghosal lost 7-11, 4-11, 9-11 to the Egyptian player after he advanced to the
finals by defeating the fifth seed Egypt’s Omar Mossad in the hard-fought semi-final match, which lasted
for an hour and 13 minutes. Saurav has reached his career-high world ranking of 10 in April 2019.
Current Affairs Quiz: January 30, 2020
1. The Supreme Court of India recently permitted to introduce which animal to Indian habitat from Namibia?
[A] African elephant
[B] African Lion
[C] African Cheetah
[D] African Rhinoceros
Answer: African Cheetah
The Supreme Court of India recently permitted the Centre to introduce the ‘African Cheetahs’ to the
appropriate habitats in India. As the Asiatic cheetah, commonly called as the Indian Cheetah, has
become almost extinct, National Tiger Conservation Authority (NTCA) filed a petition, which sought
permission for introducing African Cheetahs to India.
The bench headed by the Chief Justice S A Bobde also set up a committee with three members from
Wildlife Institute of India, Ministry of Environment and Forests, which will guide the NTCA in this regard.
2. How many wetlands in India were recently added to the ‘Ramsar Site’ list?
[A] 8
[B] 10
[C] 12
[D] 14
Answer: 10
The Union Environment Minister Prakash Javadekar recently announced that 10 more Indian wetlands
have been declared as sites of international importance by Ramsar. The number of wetlands with
Ramsar site tag has now increased to a total of 37. The total surface area covered by the 37 Ramsar

© 2020 GKToday | All Rights Reserved | https://www.gktoday.in 105


Current Affairs [PDF] - January 16-31, 2020

sites is also increased more than 1 million hectares.


In the new declaration, Maharashtra gets the first Ramsar site tag for Nandur Madhameshwar. Punjab
added 3 more sites and now has a total of 6 sites while Uttar Pradesh added 6 new sites and now has 7
Ramsar sites. The Ramsar Convention was signed by the member countries to preserve the ecological
significance of their wetlands.
3. The tableau of which state has won the Best Tableaux awards for Republic Day Parade, 2020?
[A] Odisha
[B] Uttar Pradesh
[C] Gujarat
[D] Assam
Answer: Assam
The tableau of Assam with the theme ”Land of Unique Craftsmanship and Culture” was adjudged as the
winner of ‘Best Tableaux awards for Republic Day Parade, 2020’.
The tableaux of Odisha and Uttar Pradesh were jointly selected for the second place. While Odisha
displayed the famous Rukuna Ratha Yatra of Lord Lingaraja, Uttar Pradesh showcased the cultural and
religious tourism of the state with the theme ‘Sarva Dharma Sama Bhav’. The tableaux of National
Disaster Relief Force and Jal Shakti Mission which portrayed ‘Jal Jeevan Mission’ were chosen as the
best among the Ministries and Departments and Central Public Works Department got the Special Prize.
4. Who was recently appointed as the Ambassador of India to the United States?
[A] Harsh Vardhan Shringla
[B] Jai Shankar
[C] Taranjit Singh Sandhu
[D] Vijay Gokhale
Answer: Taranjit Singh Sandhu
Veteran diplomat Taranjit Singh Sandhu has been recently appointed as the next Indian Ambassador to
the United States. He replaces Harsh Vardhan Shringla, who was the previous Indian envoy to the US.
Taranjit Singh Sandhu is an Indian Foreign Service (IFS) officer of 1988-batch and is currently posted as
High Commissioner of India in Sri Lanka. Sandhu has earlier worked as the Deputy Chief of Mission at
Embassy of India in Washington D.C. for four years from 2013 to 2017. He has also served as the Joint
Secretary (United Nations) at the Ministry of External Affairs between 2009 and 2011.
5. The recently launched Bhuvan Panchayat web portal’s new version 3.0 uses the technology of which
organisation?
[A] Indian Space Research Organisation
[B] Defence Research and Development Organisation
[C] Hindustan Aeronautics Limited
[D] Bharat Electronics Limited
Answer: Indian Space Research Organisation
Union Minister of State Jitendra Singh recently launched the version 3.0 of Bhuvan Panchayat web
Portal, which will function with the help of satellite technology developed by the Indian Space Research
Organisation (ISRO).
The project is aimed at providing geo-spatial services to assist the process of developing Gram
Panchayats, undertaken by the Ministry of Panchayati Raj. The integrated high-resolution satellite data
will provide better visualisation and enable decentralised planning and monitoring of schemes at
panchayat levels.

© 2020 GKToday | All Rights Reserved | https://www.gktoday.in 106


Current Affairs [PDF] - January 16-31, 2020

6. Which Hindi word is chosen as the ‘Oxford Hindi Word of 2019’ by the Oxford University Press?
[A] Samriddhi
[B] Samvidhaan
[C] Sampriti
[D] Sammaan
Answer: Samvidhaan
The Oxford University Press has selected the hindi word ‘Samvidhaan’ as the Oxford Hindi Word of
2019’. Samvidhaan is the Hindi word which represents the meaning of constitution. It also denotes a
body of fundamental principles as per which a state or an organisation is to be governed.
A word or expression that has attracted the attention of large number of people and reflects the mood
and character of the past year is selected as the Oxford word. The Oxford Hindi Word of the Year is
chosen by the team of Oxford Dictionaries in India with the guidance from a panel of language experts.
7. Which Indian cueist won the title at the Senior National Billiards Championship, which was his career’s
33rd National title?
[A] Saurav Kothari
[B] Pankaj Advani
[C] Dhvaj Haria
[D] Geet Sethi
Answer: Pankaj Advani
World-class Indian cueist and 23-time world champion Pankaj Advani has recently clinched his 33rd
National title by winning the title at the Senior National Billiards Championship.
In the tournament held at Pune, Pankaj defeated his fellow cueist Sourav Kothari 5-2 in the finals. In the
semi-final held earlier, Pankaj dominated with a 5-0 victory over Dhvaj Haria of Gujarat. This is not only
the 33rd national title of Pankaj Advani, but this is also his 10th senior title. He will also participate in
the national snooker championship, which is scheduled in the same venue from February 3.
8. Union Social Justice Minister has recently inaugurated the Composite Regional Centre for empowerment
of persons with disabilities in which city?
[A] Hyderabad
[B] Port Blair
[C] Chennai
[D] Kochi
Answer: Port Blair
Union Minister for Social Justice and Empowerment Thaawar Chand Gehlot recently inaugurated the
Composite Regional Centre (CRC) in the city of Port Blair, Andaman & Nicobar.
The Composite Regional Centre is aimed at providing skill development, rehabilitation and
empowerment of persons with disabilities in the local region. The centres are set under Ministry of
Social Justice and Empowerment to create infrastructure facilities and resources for persons with
disabilities. In India, the CRCs are located in several other cities including Bhopal, Lucknow, Patna,
Guwahati and Ahmedabad.
9. The skill development centre to train tribals, named ‘Sardar Valabhbhai Patel Centre for Empowerment
and Livelihoods’, is to be set up in which state?
[A] Uttar Pradesh
[B] Gujarat
[C] Rajasthan

© 2020 GKToday | All Rights Reserved | https://www.gktoday.in 107


Current Affairs [PDF] - January 16-31, 2020

[D] Chhattisgarh
Answer: Gujarat
The foundation stone for the skill development centre to train tribals of Kevadiya village in Gujarat was
laid by the Union External Affairs Minister S Jaishankar, recently. The centre is named Sardar Valabhbhai
Patel Centre for Empowerment and Livelihoods. It is a joint initiative of Gujarat government owned
Sardar Sarovar Narmada Nigam Ltd and a Telangana-based foundation.
The world’s tallest colossal statue, the 182-metre tall Statue of Unity is located in the Kevadiya village,
which is situated in the district of Narmada, Gujarat. As the statue attracts nearly 17000 visitors daily,
the local tribal youth will be trained in tourism and hospitality management.
10. Who was recently appointed as the Foreign Secretary of India?
[A] Harsh Vardhan Shringla
[B] Jai Shankar
[C] Vijay Gokhale
[D] Ruchi Ghanashyam
Answer: Harsh Vardhan Shringla
India’s former ambassador to the United States and Senior diplomat Harsh Vardhan Shringla recently
assumed charge as the next Foreign Secretary of India.
He has succeeded Vijay Gokhale to take charge in the said post for a fixed two-year term. Harsh
Vardhan Shringla is an Indian Foreign Service (IFS) officer of the 1984 batch. Shringla had successfully
hosted the ‘Howdy Modi’ event in Houston, US in September 2019. He had also served as the Indian High
Commissioner to Bangladesh, when he played an instrumental role in the historic India-Bangladesh Land
Boundary Agreement 2015.
Current Affairs Quiz: January 31, 2020
1. Which district has topped the list of aspirational districts for the month of December, released by NITI
Aayog?
[A] Sahibganj (Jharkhand)
[B] Hailakandi(Assam)
[C] Belangir (Odisha)
[D] Chandauli (Uttar Pradesh)
Answer: Chandauli (Uttar Pradesh)
The think tank to Indian Government, NITI Aayog has ranked the aspirational districts on the basis of
incremental progress made across six developmental areas in December 2019.
As per the list, Chandauli district of Uttar Pradesh has been placed in the top position, followed by
Belangir district of Odisha. Andhra Pradesh’s YSR district is in the third place. The six developmental
areas which are taken into account are: Health and nutrition, agriculture and water resources, financial
inclusion, education, skill development and infrastructure.
2. What is the upper limit for legal abortion, as per the Medical Termination of Pregnancy (Amendment) Bill,
recently approved by the Cabinet?
[A] 20 weeks
[B] 24 weeks
[C] 28 weeks
[D] 32 weeks
Answer: 24 weeks
The Union Cabinet has recently approved the ‘Medical Termination of Pregnancy (Amendment) Bill, 2020

© 2020 GKToday | All Rights Reserved | https://www.gktoday.in 108


Current Affairs [PDF] - January 16-31, 2020

and increased the upper limit for legal abortion to 24 weeks. Prior to the amendment, the upper limit was
only 20 weeks.
The existing act of ‘Medical Termination of Pregnancy Act, 1971’ has been amended and the ‘Medical
Termination of Pregnancy (Amendment) Bill, 2020’, which was drafted by the Union Ministry of Health
and Family Welfare was approved by the Cabinet. The bill is expected to be introduced in the upcoming
session of the Parliament.
3. In which city, the Indian Coast Guard has recently commissioned its ‘C-448’ high-speed boat?
[A] Kochi
[B] Vishakhapatnam
[C] Mangaluru
[D] Chennai
Answer: Mangaluru
The Indian Coast Guard has recently commissioned its ‘C-448’ high-speed boat at Mangaluru, Karnataka.
The Chief Secretary of Karnataka attended the ceremony of commissioning the Coast Guard vessel.
C-448 is a water-jet-propelled interceptor vessel, which is set to be deployed in for patrolling and rescue
operations off the coast of Mangaluru. The boat also comprises of world-class navigation and
communication equipment systems, which helps in search operations and maritime surveillance.
4. What was Tushar Kanjilal, the famous personality who passed away recently in Kolkata?
[A] Sportsperson
[B] Social worker
[C] Politician
[D] Musician
Answer: Social worker
The 85-year-old veteran social worker and educationist Tushar Kanjilal passed away recently in Kolkata.
He was the recipient of the prestigious ‘Padma Shri’, the fourth highest civilian honour of India, in 1986.
Fondly called as ‘Mastermoshai’, he worked as the head of a high-school in Rangabelia village of
Kolkatta and strived for promoting education among the local population. He also authored a book titled
‘Who Killed the Sunderbans?’, which is based on the issue of destruction of the Sunderbans’ mangrove
forests.
5. What is the theme of the World Sustainable Development Summit, 2020?
[A] Towards 2030 Goals: Making the Decade Count
[B] Sustainable Energy across the world
[C] Attaining SDG before 2030
[D] Energy and Resources for all
Answer: Towards 2030 Goals: Making the Decade Count
The World Sustainable Development Summit (WSDS) is the annual flagship event organised by The
Energy and Resources Institute (TERI). This year, the global summit is scheduled to be held at the India
Habitat Centre, New Delhi.
The three-day event was inaugurated on January 29, 2020 with the theme, ‘Towards 2030 Goals: Making
the Decade Count’. The summit has been organised for 18 years from 2001, to provide a platform for
global leaders to discuss climatic issues. This year, the event focusses on the global efforts to meet the
Sustainable Development Goals of the United Nations.
6. The Surajkund International Crafts Mela is the flagship annual event organised by which state?
[A] Bihar

© 2020 GKToday | All Rights Reserved | https://www.gktoday.in 109


Current Affairs [PDF] - January 16-31, 2020

[B] Haryana
[C] Himachal Pradesh
[D] Assam
Answer: Haryana
The 34th Surajkund International Crafts Mela is set to be held from February 1 to 16, 2020, at Surajkund,
Haryana. It is an annual flagship event organised by the state government of Haryana.
This year, Himachal Pradesh is selected as the theme state for the festival. The Chief Minister of
Himachal Pradesh, Jai Ram Thakur attended the inaugural ceremony of the Mela. Various cultural
programmes and exhibitions are arranged to promote the tourism of the state. This will enable the state
to showcase its tourism potential and attract more people to the state.
7. As per the recent survey of Transparency International, which state has been ranked as the top state in
budget formulation practices?
[A] Odisha
[B] Andhra Pradesh
[C] Assam
[D] West Bengal
Answer: Assam
According to the recent survey conducted the international NGO Transparency International, Indian state
Assam has been ranked as the top state in terms of best practices followed during budget formulation.
The survey considered four parameters for ranking the states, which are: public disclosure, budgetary
process, post budget fiscal management and efforts to make budget more transparent and citizen
friendly. The survey also revealed that the states which are placed at the bottom of the list were Goa,
Maharashtra and Punjab.
8. Which Indian has been recently appointed as the UN Resident Coordinator in Thailand?
[A] Harsh Vardhan Shringla
[B] Gita Sabharwal
[C] Jai Shankar
[D] Taranjit Singh Sandhu
Answer: Gita Sabharwal
Gita Sabharwal of India has been recently appointed as the United Nations Resident Coordinator in
Thailand, with the approval of the host country.
Gita Sabharwal was serving as the Peace-building and Development Adviser representing the United
Nations, in Sri Lanka. The UN Resident Coordinators are the representatives of the UN Development
system and leaders of the UN country teams. They also support to member countries in achieving the
2030 Sustainable Development Goals (SDG) targets.
9. Amazon India recently partnered with Eastern Railways, to set up a pick-up kiosk in which railway station?
[A] Dum Dum junction
[B] Bidhan Nagar
[C] Sealdah
[D] Durga Nagar
Answer: Sealdah
e-Commerce giant, Amazon India recently partnered with Eastern Railways, to set up a pick-up kiosk in
Sealdah railway station in Kolkata.
By selecting the railway station as their pick-up location while ordering their products in the company’s

© 2020 GKToday | All Rights Reserved | https://www.gktoday.in 110


Current Affairs [PDF] - January 16-31, 2020

website or application, the customers can pick up their packages conveniently at the station. In 2019,
Amazon successfully launched pick-up kiosks at four railway stations in Mumbai by partnering with
Indian Railways.
10. Which international smartphone firm recently signed an MoU with IIT- Hyderabad for Research and
Development projects?
[A] Xiaomi
[B] OPPO
[C] Huawei
[D] Vivo
Answer: OPPO
The international smartphone firm OPPO recently signed an MoU with the Indian Institute of Technology
(IIT)- Hyderabad for collaborating in Research and Development projects in emerging technologies.
As per the agreement signed between the R&D Department of the IIT and Head of OPPO India, joint
research and development for multiple projects will be undertaken for a period of two years, in
technologies such as Image processing, 5G networks and Artificial Intelligence. The partnership also
aims to create industry-ready and skilled workforce from the institution.

You might also like